You are on page 1of 207

2018

Prometric Exam book


The index edition

‫تم جمع األسئله في أقسام متعدده حسب المواضيع و األمراض‬


‫الفهرس مرتب حسب الترتيب األبجدي‬

dr.Hanan Shamse ‫تم بعمل التقسيم‬


dr.Ibrahim Najem ‫تنسيق الكتاب‬

Dr.najem , Dr. Rua'a , Dr. Ashraf Second Edition


Dr. Asmaa , Dr.shamse, Dr. Nesreen
Index
parts Page
Acanthosis Nigricans 3
Acne 4
Basic Dermatology &histology 12
Bullus Diseases 21
Connective Tissues Disesases 32
Cosmatology & LASER 41
Drugs 49
Eczema,Dermatitis & pruritis 66
Erythema 75
Fruits In Medicine 77
GenoDermatoses & Ichthyosis 79
Glands Diseases 90
Metabolic, Granulomas, Infeltrations, Vitamines & FDE 93
Hair 106
Hypersensetivity ,Urticaria&Angioedema 112
Infection 116
Internal medicine 137
Kaposi Sarcoma 138
Melanoma 140
Mycosis fungoids 143
Miscelaneus 145
Nail 147
Other Skin Cancers 151
Pigments Diseases 163
Pregnancy Dermatoses 170
Psoriasis & Papulosquamous Diseases 172
SCC 183
Sexualy transmitted Diseses 185
Surgery of the Skin & Local Aneasthesia 195
Tongue and Mucous Membranes 197
Vascular 199

2
Acanthosis Nigricans
931-Which of the following is most commonly seen in patients with acanthosis nigricans?
o An underlying internal cancer.
o An underlying non-hodgkin
o Internal malignancy.
o DM.
 Insulin resistance.

From the questions in page 83 :

167-Acanthosis nigricans: adenocarcinoma of the stomach


168-Acanthosis nigricans: pruritus (NOT in erythema gyratum repens)
169-Acanthosis nigricans:insulin resistance

From the questions in page 83 :

94-Most common cause of acanthosis nigricans- obesity

3
Acne
99-Contraindications to oral retinoids include all except
o Pregnancy
o Patients at risk for liver toxicity
o Chronic use in children
o Concomitant use of methotrexate and oral retinoids
o Aphakia
100-The difference of side effect of etretinate and isotretinoin is that
o Isotretinoin procedure
o Facial dryness + redness of lips
o Alopecia

208-Primary lesions of acne include:-


o Papules
o Pustules
o Comedones
 All of the above
209-The pathognomonic lesion of acne vulgaris is the
o Papule
o Comedone
o Pustule
o Cyst
210-The comedones are seen in
o Acne
o Periorbital ages skin
o Radiated skin
o All of the above
211-Small papule with invisible opening
o White comedone
212-Sebum function all of following EXCEPT
o Immune regality modations
 Vit.E source from skin
o Protect from any microbiocal pathology
o Immuno preventive barrier
213-Unique composition of human sebum is
o Cholesterol is more than wax esters
o Squalene is more than cholestrol
214-Sebum production increase if
o Squalene is more than cholesterol
o Cholesterol is more than Squalene
o Wax ester is more than Squalene
215-Topical treatment in acne vulgaris that decrees sebum is :
 Clindamycin
o Erythromycin
o Benzoyl peroxide
o Azelaic acid

4
216-Which of the following topical agents has effect of reducing sebum production ?
 Topical clindamycin plus nicotinamide
o Topical erythromycin plus zinc
o Topical benzol peroxide
o Topical azelaic acid
217-The follicles in which acne lesions develop generally contain
o Only sebum
o No hair
o A villus hair
o A terminal hair
o Only keratin
218-Acne venenata most common cause
o Perfumes
o Halogenated hydrocarbon
o Cutting non absorbable oil
o Compliment
219-Acne venenata most common cause
o Cosmetic
220-Treatment of acne venenata
o Topical retinoid
o Dapson
o Doxyclin
o Pencellin.
221-Acne conglobata all true EXCEPT:-
o More in male than in female
 Absence of comedones
o Hidardentis suppurativa may associate it
o Most common on posterior neck & back
222-Acne conglobata is best treated with:-
o Topical Tetracycline's
o Systemic Tetracycline's
o Ampicillin
o None of the above
223-Teenager male presented with severe inflammatory nodulocysistic lesions on face, the most probable
diagnosis is:-
o Acne vulgaris
o Acne conglobata
o Acne necrotica
o Acne rosacea
o Drug induced acne
224-All are true about acne fulminants EXCEPT:-
o Fever
o Leucocytosis
o Arthritis
o Lesions contains osteolytic areas
o Gram- ve infection
225-Acne fulminants can be presented by all EXCEPT:-
o Fever
 Hemoptysis
o Polyartheritis
o Leucocytosis

5
226-For acne fulminans:
o Arthralgia
o Fever
o Leukocytosis
o Malaise
o All of the above
227-Monomorphic eruption on sun exposure:-
o Acne tropica
o Acne aestivalis
o Acne cosmotica
o Acne venenata
228-Drug induced acne is characterized by:-
o Cause nodule or cyst
 Monomorphic
o Mainly on the face
o Has connection
229-Systemic steroid acne commonly seen on
o Face
o Back
o Abdomen
o Buttocks
o Thigh
230-Premenstrual acne is mainly mediated by
o Estrogen
o Progesterone
o DHT
o Glucocorticoids
o None of the above
231-Side effects of Isotretenoin include:-
o Chelitis
o Hair fall
o Diarrhea
o Pseudotumor cerebri
 All of the above
232-Long term side effect of isotretinoin
o Skeletal hyperostiosis
o Cirrhosis
233-The most common well established side effect of oral retinoid is:-
o Exfoliative dermatitis
o Erythroderma
o Alopecia areata
o Optic abnormalities
o Teratogenicity
234-Treatment of Acne recalcitrant
o Sulfur high dose
o Estrogen
o Dapsone
o Prednisone
 All of the above
235-We must not use this drug in excoriated acne:
o Benzyl peroxide

6
o Tetracyclines
o Clindamycin
236-Treatment of scared acne which type of laser
o Diode laser
o Nd-yag 1064nm laser
o Erbium laser
237-Ice-pick acne scars are best treated by
o Microdermabrasion
o TCA peeling 50%
o Eryag laser therapy
o Punch replacement grafting
o Topical tretinoin
238-Dermbrasion is used to treat acne scar in the:-
o Face
o Trunk
o Shoulder
o Forearms
239-Rosacea can have all EXCEPT:-
 Comedon
o Follicular papules
o Telangectasia
o Pustules
240-Rosacea cause all EXCEPT:-
o Conjunctivitis
o Sclera
o Blepharitis
o Iritis
o Retinitis
241-Rhinophyma is the complication of:-
o Acne Rosacea
o Acne vulgaris
o Hidardentis suppurativa

865-Patient with soft papule , central opening , localized on the face , what is the treatment(commedons)
o Surgical excision
o Tetracycline
o Intralisonal steroids
o Reassurance

932-All for TTT of acne EXCEPT:


 Pimozide
o Tetracycline
o Tretonine
o Vit A

961-Female patient with acne treated with doxycycline 100 for 2 months then develop papulopustular and
nodular lesion (kliebsiella) what is diagnosis:
o Drug eruption from doxy

7
o Acne conglobate
 Acne gram -ve follicultis

965-Hidradenitis suppurativa all true except


 Should be ttt with intralisional antibiotics for 2 w
o It is a chornic inflamatoin of apocrine sweet gland
966-Regarding fox foradyce what is true
o Female=male
o Asymptomatic
968-Fox fordyce disease is:
o Increasing with pregnancy
 Decreasing with pregnancy
o Disorder of eccrine gland

985-Female pt developed lesions on the cheeck & nose and diagnosed as rosacea. Rx is:
o Amoxacillin
 Tetracycline

996-32 years old male with active nodulocystic acne and skin type 4 physical treatment after medical
treatment
o Dermabrasion best modality after medical treatment
o Dermabrasion, chemicalpeeling
 Intralesion steroid
o Ch.peeling dermbrsion and intralesion steroid
o Dermabrasion scar revision

1002-Fiddler's neck in violin players due to:


 Acne mechanica
o Allergic contact dermatitis
Irritant contact dermatitis

1059-The joint most frequently affected in acne fulminans is the :


o Elbow
o Intervertebral
 Sterno clavicular
o Sacroiliac

1080-Error in the pathogenesis of rosy (? Acne rosacea)


o Damage connective tissue
o About vessel filtrate
 Infection helikobakatr
o There effective vessels

1093-Patient has bromide acne

8
 Be in the form of papules & pustule
o There is typical
 The most important no comedon

1241-The best time for complete improvement of comedones by tretonin is


o 1-2w
o 2-4w
 4-12w(comedon)
o 6months(acne)

1242-Lesion on the thight show dimpling on squeezing between thumb what most possible
histopathology discharging nodules since 5 years in the axilla; inguinal and groin
o Acne vulgaris
 Acne inversa
o Axillary scrofluderma
o Actinomycosis

1247-All are associated with acne fulminans EXCEPT :


 Rapid response to antibiotics
o Male more than female
o Ulcerative lesions
o Leukaemic lesions
o High fever

1270-A young person is having comedones and papulo-pustular acne over face and trunk and back. How
will you manage the patient?
o Topical retinoic acid
 Oral doxycycline + topical retinoic acid
o Topical clindamycin
o Oral retinoic acid

1281-A 19-year old male presents with several comedones, papules and pustules on face and trunk. The
appropriate drug of choice for the patient would be
o Topical retinoic acid
 Topical retinoic acid + Oral doxycycline
o Topical clindamycin
o Topical azithromycin

1312-Worst prgnosis for acne conglabata


 Neck
o Forheade
o Chin
o Check

1381-femal patient with lesion in axilla, as fissures, pits, plaques and scarring. the diagnosis;

9
 acne inversa

1398-These are no documented studies that some foods, chocolate, jellyfish can share in the production of
acne:-
o True
o False

1398-Acne vulgaris can occur in normal sized sebaceous gland with normal sebum secretion
o True
o False

1400-Most of acne patients don't seek medical


Treatment
o True
o False
1401-Small multiple papules are characteristic in steroid acne
o True
o False
1402-Benzyl peroxide is powerful antibacterial as it bacterial count & transformation of triglycerides—
o True
o False
1403-As a group sebaceous gland not in size & no in sebum production in patients of acne than those
without acne:-
o True
o False
1404-Apocrine glands in human anogenital regions are small and atrophic and not produce smell:-
o True
o False
1405-Sebum amount affect acne:-
o True
o False
1406-Acne may extend as far as buttocks
o True
o False
1407-Acne in an 12 month old infant usually indicates endocrinal abnormalities
o True
o False
1408-In normal persons apocrine sweat glands have no odor:-
o True
o False
1409-Acne vulgaris predispose to Rosacea

10
o True
o False
1410-Rosacea is effectively treated by metronidazole
o True
o False
1411-Rhinophyma is a precancerous condition
o True
o False

1417- The difference between side effects of etretinate & isotretinoin is that isotretinoin cause facial
dryness & redness of lips
o True
o False

1476-Steroids usually used in treatment of acne


o True
o False

1482-Isotretenoin is used in treatment in comedogenic acne


o True
o False

1529-Drugs cause acne form eruptions are:- Lithium, Hydantion, Steroid, Androgen, Contraceptive
pills.
1530-Retin A in acne treatment give approving after:- 6 to 8 weeks.
1531-Retinoid not given to patient with:- hyperlipidemia.
1532-Gram-ve folliculitis is classified as different from acne vulgaris mainly because of:-its flora and
the absence of comidones.

1539-In acne vulgaris, the precursor of large inflammation is:- the white head.

From the questions in page 84 :

115- DOC for rosacea- Metronidazole (topical), Doxy (Oral)

11
Basic Dermatology &histology

1. Skin layer which prevent body from water loss is


o Stratum corneum
o Stratum granulosum
o Stratum spinosum
o Stratum basale
o All of the above
2. Stratum lucidum mostly apparent in :
o Scalp
o Back
o Palm
o Face
3. Mucous membrane generally is more permeable than skin because it lacks
o Keratin
o A surface lipid film
o A stratum corneum
o Pilosebaceous follicles
o All of the above
4. Time which based cell needs to reach stratum corneum is:-
o 2 weeks
o 28 days
o 35 days
o 10 days
5. The weakest part of BMZ is:-
o Lamina lucida
o Lamina densa
o Anchoring fibrils
o Hemidesmosomes
6. Papillary dermis differs from reticular in
o Reticular fibers
o Collagen fibers
o Elastic fibers
o Collagen and elastic fiber
7. Normally the human dermis is thickest on the
o Scalp
o Back
o Thigh
o Palm
o Sole
8. Mechanical strength
o Dermis
o Epidermal
9. Causes of mechanical traction of the skin
o Epidermis
o Dermis
o Subcutaneous
o Fat

12
10. Basal cell layer one layer normally but in glabrous skin and hyperproliferative lesion may reach
o 2 layer
o 3 layer
o 4 layer
o 5 layer
11. Normally most of the epidermal cell replication cycle is occupied by
o GI phase
o S phase
o G2 phase
o M phase
o None all the above
12. Separation of reticular and papillary dermis is seen by arrangement of
o Ground substance
o Collagen
o Reticulum
o Elastin
 B&d
13. In normal skin , transient-amplifying cells are found in the :
 Basal layer
o Suprabasal layers
o Basal and suprabasal lauers
o Dermis
14. About nodule all are false except:-
o Differ from papule only in size
o Heals with scarring
 Palpable circumscribed solid lesion
o Heals rapidly by intralesional steroids
15. Whish of these keratinis is characteristic of the coronoid lamellae ?
o 16
o 14
o 7
o 3,12
16. Which of these keratin pairs are characteristic of differentiated epidermal keratinocytes ?
o 8 and 18
o 3 and 12
o 6 and 16
 1 and 10
17. Collagen of human mostly
o Type1
o Type 2
o Type3
o Type4
18. ↑↑Proliferation of keratinocytes by
o IL-1
o IL-5
o Catecholamine
o Acetylcholine
19. Description of cell binuclear rounded cytoplasmic
o Granular
o Merkel cell

13
o Melanocyt
o Ls
o Mast cell
20. Cells has a halo surronded it
o Mast
 Merkel cell
o Mellanocyte cell
o Langrhans
21. Cells has a halo surrounds it in H/P
o Mast cells
 Merckle cells
o Melanocyte cells
o Langerhans cells
22. The definitive organelle of the melanocyte is the melanosome, and the definitive organelle in the
melanophages is
o Premelanosome
o Melanophore
o Mitochondrion
o Lysosome
o None of the above
23. Melanocyte destruction due to cooling occurs at following dergrees
o -4 _ -7
o -1 _ -5
24. Melanocytes in the oral mucosa:-
o Can seldom be found,
o Are normally inactive,
o Lack dendrites,
o Occur only in laminal propria.
 None of above
25. Parakeratosis is not typical of
o Psoriasis
o Pityriasis rosea
o Subacute eczema
o Lichen planus
o Seborrehic dermatitis
26. Fluid accumulation within epidermal spaces due to rupture & death of epithelial cells is called:-
 Cytoslysis
o Spongiosis
o Acantholysis
27. Which of the following lack desmosomes ?
o Keratynocytes of the granular
o Langerhans cells
o Melanocytes
o Keratinocytes of basal cell layer
o B and C
28. Histological epidermal structure which responsible for prevention of cohesion of epidermal cells and
acantholysis and blister formation in vesico- bulious dermatosis is :
o Epidermal cell collisions
o Intercellular substances
o Desmosomes

14
29. Histological epidermal structure which is responsible for prevention of loss of coherence between
epidermal cells:-
o Intercellular substance
o Desmosomes
o Lamina lucida
o Collagen fibers
30. The basic morphologic patterns of intraepidermal vesicle formation are:-
o Spongiosis
o Acantholysis
o Cytosis
 All of the above
31. Spongiosis in dermatitis
o Intact desmosomes but look stretched
o Intact desmosomes that do not seem stretched
o Acantholysis
32. Vesicule occurs histopathology by –
o Loss of cohesion of desmosomes leading to acantholysis
o Defect in basement member- bullae
33. A typical early (about 8 hours) response to epidermal injury is
o Glycogen depletion
o Glycogen accumulation
o Shrinkage of basal cells
 Increased numbers of mitoses
34. Langerhans's cells all except
o Play an important role in antigen presentation
o By Hx & E stain they appear as high-level brown cells
o They express class II MHC antigens
o They circulate through the epidermis, the dermis, lymphatics and lymph nodes
o Corticosteroids decrease their surface markers
35. One of the following vehicles doesn’t suit an acute skin condition:-
o Powders
o Solutions
o Ointments
o Lotions
40-Normal human epidermal glucose levels in vivo are
o About equal to blood levels
o About 30% to 60% of the blood level
o About 10% of the blood level
o Zero
o Not exactly known

52- Epidermal infiltration by pustule of pmls (Munro microabsceses)occurs in:-


o Pustular psoriasis
o Reiter's syndrome
o Impetigo herpetiform
o Acrodermatitis continua (Dermatitis Repens)
 All of the above

682-The vasomotor effect of local heating of the skin is


o Vasoconstriction

15
o Vasodilation
o Obscured by the precedence of sympathetic vasomotor control
o Negligible at ordinary encountered temperatures
o Dependent upon intact sympathetic innervations

890-Extrusion of dermal content through the epidermis is called


 Transepidermal elimination

919-What is true about protein C?


 Has antithrombotic effect
o Inhibit protein S

939-Cyto skeletal structures


 Tonofilament
o Desmosomes
o Odland bodies
o Hyalokeratine granules

1009-Painful well circumscribed nodule or papule in the thigh which is compressible is mainly:
 Dermatofibroma
o Cholingenoma
o Keloid

1045-Anchoring filament
 Anchoring fibril:type VII collagen lamina densa
 Anchoring filament:antigen2 (type XVII collagen) lamina lucida

1067-Bacel cell is a single layer naturally and when diseases are proliferating is
o 2
 3
o 4
o 5

1116-Percentage of keratinocytes in epidermis:


 90% (80-85%)
o 20%
o 50%
o 75%

1140-The function of laminin:


 Connection
o Anchoring
o Cell adhesion

16
o Transmission
o Immunoregulator

1166-The skin Langerhans cell what is right...


 Originated from neural crest.
o Do not contain tennis racket granules.
o Are 80 cells/mm2.
o Are not dendritic.
o Are basal.

1177-Conversion of testestrone to dihydrotestestrone ccures in females in"


 Sebaceous gland
o Ovaries
o Liver
o Breast
o Intestine

1178-First appearance of hair in fetus:


o 4th week
o 9thweek
o 11thweek
 12th week

1183-Component in basement membrane :


Collagen
 Laminin
o Fibronectin

1203-A cell has a smooth surface and normal looking cytoplasm is


 Merkel cell

1222-Defention of perforatin disease


 Extrusion of dermal materials through the epidermis with minimal damage

1223-All cells are present in epidermis EXCEPT


 Mast
o Merkel cell
o Mellanocyte cell
o Langrhans
1224-Keratin in basal layers:
 14&5

17
1252-Enzye in cell envelope :
 Transglutaminase Loricri

1273-Spongiosis due to:


o Intracellular edema
 Intercellular edema
o Appotosis of cell

1279-Cytoskeleton include all of the following EXCEPT:


o Tonofilament
o Actin
o Tubulin
 Filaggrin
1280-Function of cyto skeleton except
o Maintain shape
o Transport organelles
o Move chromosome
 Protect from toxin damage

1285-What is normal flora of the skin:


o Staphylococcus aureus
 Staphylococcus epidermidis
o Micricone
o Corynebacterium

1292-keratin 19
o Basal cell of mucous membrane
 Bulge of hair follicle

1293-Stemcells of hair follicle present in


 Bulge of hair follicle

1402-Apocrine glands in human anogenital regions are small and atrophic and not produce smell:-
o True
o False

1408-In normal persons apocrine sweat glands have no odor:-


o True
False

1414-Conversion of testosterone to the active form dihydrotestosterone occurs in the target organs
(prostrate, vas deferens, seminal vesicles & epididymis under effect of 5:-alpha:-reductase enzyme)
o True
o False

18
1427-Macule is only change in color
o True
o False
1428-Pustule is a primary skin lesion
o True
o False
1429-Vesicle is a pus containing epidermal cyst less than 0.5 mm
o True
False
1430-Nodule differs from papule in depth & size
o True
o False
1431-Crust is usually honey colored
o True
False

1432-Wheel is the characteristic lesion in urticaria


o True
 False

1433- Wheel occurs only in urticaria


 True
o False

1434-Fissure is a breach in the skin without loss of substance


o True
o False
1435-Lichenification is a feature of chronic contact dermatitis
o True
o False

1447-Acantholysis is a loss of the coherence between epidermal cells


o True
False

1460-Isomorphic phenomenon is another term for keener's phenomenon


o True
o False
1547-Desmosome Complex is:- coherence between epidermal cells.
1552-Normal sebum content:- Waxester + Squalene + Cholesterol Esters (Wsce) ‫ويسكي‬
1553-Testosterone is converted to dihydrotestosterone in:- target organs.

1558-Which sensation is lost first:-


Temperature then light touch then pain then deep touch.

1579-The absence of light below 290nm at the earth surface from solar radiation is due to:- atmosphere
ozone and other gases.

1580-The epidermal turn over rate on the scalp in dandruff is:- increased.
1581-Cell envelope of corneacyte – loricrin
1582-Langerhans cell probably functions as:- contact allergen trap

19
1584-The following organisms does not commonly colonize healthy skin:- candida albicans

1594-Normal sebum content:- Waxester + Squalene + Cholesterol Esters (Wsce) ‫ويسكي‬


1598-Turnover time of sebaceous gland cells as determined in radioactive thymidine update studies is
about:- 7 days.

1610- Anchoning filament presnt in:- lamina lucida

1618-Cerumem consists:- sebum and apocrine secretion.


1624-Epidermal cleft opened and the cells die:- Cytolysis.
1639-Lamina fibroreticularis=lamina densa

1643-Fox for dyce disease is disorder in:- Eccrine gland


1644-Fox for dyce spots is disorder in:- Sebasceous gland
1645-Lamina fibroreticularis=lamina densa

From the questions in page 78 :

1-Lamina lucida: weakest BMZ layer.

43-Most common layer of epidermis for lamellar body presence- granular layer

From the questions in page 82 :

44-Most common layer for synthesis of vitamin D ( Also same answer for presence of
langerhans cells)- spinous layer

20
Bullus Diseases
49-Each of the following are the nikolesky sign except:
o Pemphigus
o Pemphigoid
o H. Simplex
o Insect bite
50-Positive nikolysky sign found in:
o B.pemphegoid.
o P .vulgaris.
51-Each of the following are +ve Nicolesky's sign EXCEPT:-
o Pemphigus vulgaris
o Herpes simplex
o Pemphigoid
o S.S.S.S

432-Staph toxin in (pemphigus vulgaris) because it


o Dsg I
o Dsg III
o Dsc I
o Dsc III
433-Regarding pemphigus vulgaris all are true EXCEPT:-
o Immune complexes are seen in the intercellular spaces
 Subepidermal bullae formation
o Commonly involves mucous membranes
o IgG is found in both involved & clinically free skin
434-Paraneoplastic pemphigus is characterized by the presence of IgG antibodies against of the following
antigens ?
o Desmocollin 1
 Desmoplakin 1
o Desmoglein 3
o Desmocollin 3
435-Oral lesion in pemphigus vulgaris:-
o 10%
o 30%
o 20%
o 50%
436-All of the following diseases, there are deposition of immunoglbulin at BMZ EXCEPT:-
o Pemphigus vulgaris
o Dermatitis herpetiform
o Bullous pemphigoid
o Herpes gestation
437-The worst prognosis is expected in:-
o Bullous pemphigoid
o Pemphigus foliaceous
o Pemphigus vegetans
o Dermatitis herpetiform
o Herpes (pemphigoid) gestations
438-Igg deposition occurs at BMZ in all of the following EXCEPT:
 Pemphigus vulgaris

21
o Dermatitis herpetiform
o Bullous pemphigoid
o Herpes gestation
o EBA
439-Pemphigus foliaceous:-
o Have no racial base
o More in Jewish
o Commonly have mucosal lesion
o All of above
440-All the following lesions are present in Pemphigus foliaceous EXCEPT –
o Mucosal lesions.
441-P. Foliaceous like lesions on chest with erythema and scaling on face we can give all EXCEPT
o Cyclophosphamid
o Azathioprine
o Dapsone
o Thalidomide
442-A-35-year-old patient with pemphigus vulgaris is being treated with prednisalone 40 mg daily for last
6 months with inadequate control of disease activity. Which of the following adjuvant would be most
effective for this patient?
o Methotrexate
o Cyclophosphamide
o Colchicine
o Cyclosparin A
443-Patient with pemphigus not responds to steroid which drug can be used adjuvant
o Cyclophosamid, azathioprine
 Cyclosporine
o Methotraxate
o Gold(not used)
o Dapsone
444-Treatment of choice for treating IgA pemphigus is
o Colchicines
o Potent topical steroids
o Minocycline
o Dapsone
o Nicotinamide with tetracycline
445-The immunoglobulin precipitated in paraneoplastic pemphigus
o IgA
o IgG
o IgG and C3
o IgM
o All of the above
446-Of the following, the malignancy that has been associated most frequently with paraneoplastic
pemphigus:-
o Gastric carcinoma
o Hepatoma
o Malignant carcinoid
o Chronic lymphocytic leukemia
o Adenocarcinoma of the breast
447-Immunoglobulin in bullous pemphighoid are located in
o Desmosome
o Hemidesmosome

22
o Anchoring fibrils
o Anchoring fibers
448-The sites of immune globulins in bullous diseases
o Inter cellular and liner basement membrane deposition of igg mainly
449-Immunoglobulin's in bullous pemphigoid are:-
o Igg & C3
o Igm
o Iga
450-A case of violacous hyperpigmented patch over lower leg after and he developed tense bullae over
norma skin and away from it… DIF is positive what is the diagnosis
o Bullous lichen planus
 Lichen planus pemphigoides
451-Treatment of bullous pemphigoid
o Azathioprine
o Cyclosporine
o Cyclophosphamid
o Corticosteroid
452-The treatment of cicatricial pemphigoid include:
o Predinsolone
o Dapsone
o Predisolone+cyclophosfamide.
o Predinsolone+ azathioprine.
o All above.
453-Most steroid sparing drug used in bullous pemphigoid
o Mycophenolat mophetyl
o Azathioprine
o Ritiximab
o Mtx
454-What is the best cytotoxic drug can be used in bullous pemphigoid patient. A severe ocular damage
o Cs
o Azathioprine
o Cyclophosphamide
o Infliximab
455-Herpes gestation what is TRUE
o Cannot occur without pregnancy
 HG antigen is always found in DEJ
456-Pemphigus gestationis is deposition of whish ig in dermoepidermal junction by immunofluorescence:
o Ig A
o Ig G
o Ig M
o C3
o Ig C4
457-HG:-
o Occurs during pregnancy only
o Carry significant effect on mother morbidity and infant morbidity and mortality
o Differs completely than bullous pemphigoid
o Immunoglobulin deposits at Lamia Lucida
458-H. Gestations what is TRUE:-
 HG factor present in the lamina lucida.
o HG morbid to the mother
o HG morbid to the fetus

23
o HG morbid to both mother and fetus.
459-Herpes gestation : all EXCEPT
o Occurs only during pregnancy
o Occurs first time in multipara only
o Completely different in histopathology from bullous pemphigoid
 All of the above
o None of the above
460-One of the following differentiates H.G from PUPPP:-
o PUPPP occurs in the 1st trimester
o PUPPP occurs in the last trimester
o PUPPP has intense pruritis
o PUPPP has urticarial plaques & papules
461-A30-year-old patient who is 6 months pregnant presents with multiple urticated papules and plaques
and tense bullae on the obdomen and legs. Itching is severe . What would skin biobsy from the bulla show
?
o Intrapidermal split with Tzanck cells
o Subcorneal split with numerous red blood cells
o Papillary dermal neutrophilic microabcesses
 Subepidermal split with eosinophils
462-The treatment of herpes gestation
o Topical steroids
o Systemic steroids
o Anti histamine
463-Treatment of Cicatricial pemphigoid
o Prednisolone
o Prednisolone + azathioprine
o Prednisolone + Cyclophosphamide
o Dapsone
 All of above
464-All have IgG deposition along BMZ by DIF EXCEPT:-
o BP (Bullous Pemphigoid)
o SLE (Systemic Lupus Erythematosis)
o EBA (Epidermolysis bullose Acquired)
 DH (Dermatitis Herpetiform)
465-Male with severely pruritic papules and tense vesicles on the back and shoulder and tense urticated
plaque on abdomen what is probable IG if no oral lesion (DH)
o Granular Ig A
o IgG
o C3
466-The most reliable test for diagnosis of dermatitis herpetiform is:-
o Hematoxylin/eosin stain of perilesional skin
o Indirect immunofluorescence demonstrating IgA antibodies to reticulin
o Immunoblotting study demonstrating presence of gluten antigen in skin
o Direct immune fluorescent staining of perilesional skin showing dermal papillary IgA deposits
o Detection of HLA-Qw2 antigen
467-Best reliable test for DH diagnosis is –
o Direct immunofluorescence
o Of non-lesional skin
468-All are true regarding DH EXCEPT:-
 Good response to steroids

24
o Symmetrical lesions
o Immune florescence show iga antibodies in uninvolved skin
o Severely pruritic
469-In patient with dermatitis herpetiformis may be treated with sulfapyradine 1-1.5 g/day if:
o Dapsone is not tolerated.
o In elderly patient.
o In patient with cardiac disease.
 All above.
470- 4 years old boy presented by 2 months duration of Perioral & napkin annular vesicular eruption. The
most probable diagnosis is:-
o IgA bullous dermatosis
o Impetigo contagiosum
o Contact dermatitis
o Chronic herpes simplex
471-IgA linear dermatosis:-
o Associated with gluten enteropathy
 Dapsone is needed in treatment
o Carry bad prognosis
o Bullous formation uncommon
472-In linear iga bullous disorder , what is the initial dose of Dapsone in mg/kg?
o 0.25
o 0.5
o 1.0
o 2.0
473-Chronic bullous of childhood dermatosis closely associated with:
o HLA – B27
o HLA _B13
o HLA – B17
o HLA – DR3
o HLA – DR4
474-Chronic bullous dermatosis of childhood (CBDC) is closely associated with:-
o HLA-B27
o HLA-B13
o HLA-B8
o HLA-DR4
475-Treatment of choice of CHDC is:-
o Dapsone
o Corticosteroids
o Sulphapyridine
o Plasmapharesis
476-The organism isolated from subcorneal pustular dermatosis is:-
o Staphylococci
o Streptococci
o Mycobacterium tuberculosis
o None of the above
477-Subcorneal pustular dermatosis is best treated by
o Tetracycline
o Erythromycin
o Isotretinoin orally
o Dapsone
o Topical corticosteroids

25
478-The underlying etiology of Hailey-Hailey disease is a mutation in the
o ATP 2A2 gene
o ATP 2C1 gene
o Desmoglein 1 gene
o Desmoglein 3 gene
o Desmoplakin 1 gene
479-Differential diagnosis of H & H disease is:-
o Impetigo
o Pemphigus vulgaris
o Darier's disease
o Pemphigus vegetans
o All of the above
480-Treatment of hailey - hailey disease :
o Steroid
 Dapson
o Azathioprine
481-Haily-Haily disease treated by ‫كله صح بس بالترتيب‬
 Steroid
 Antibiotic
 Dapson
482-Treatment of grover disease (transient acantholytic dermatosis)
 Isotritinoin for systemic
 Steroid for topical
o Dapson
483-Pt. Photo sensitivity malar rash and have develop lesion like pemphigus have sterile pustule in palm
*sole which not you give
o Acetritin
o Azathioprin
o Cyclophosphamide
484-Female patient with recurrent malodorous lesions around her neck, in axilla, groin histopathology
reveals familial benign pemphigus what is best ttt:
o Dapson
o Antiherpetic/anti- infective drugs
o Steroid
485-The inheritance of benign familial pemphigus is
o Autosomal recessive
o Autosomal dominant
o Autosomal dominant with variable penetrance
o X-linked dominant
o X-linked recessive
486-EBS (simplex) defect
o Keratins 5 and 14
o Collagin
o Hemidosoms oto hmunity
487-EBA is commonly characterized by:-
o Vesicles & bullae following trauma
o Tends to occur in frictional skin areas
o Mucosal lesions are common
 All of the above
488-The clinical presentations of EBA include:-

26
o The "classical" presentation
o A bullous pemphigoid like presentation
o A cicatricial pemphigoid like presentation
o All of the above
o None of the above
489-A 6 – year- old boy with dystrophic epidermolysis bullosa with recurrent blistering is treated with
low dose of isotretinoin for controlling his disease and reducing the risk of squamous cell carcinoma.
What is the most likely cutaneous complication that would occur on increasing the dose of the drug
o Mechanical fragility
o Photosensitivity
o Xerosis
o Ulceration of the lesions

917-Most fatal of those is:


o Bullos pemphgoid
o Pemphigus foliaceus
 Pemphigus vegetans
o Herpes gestations
o Dermatitis herpitiforms

926-Adjuvant to corricostiroid in PV :
o Cyclophosphamide
o Methotrexate
o Cyclosporine
o Colchicines
 Mycophenolate mofetil or azathioprine

946-An elderly man + intense itching urticarial plaque lesions + tense bubbles on the abdomen and nooral
lesions what is the
o Desmoglin 1
o Desmoglin 3
BPA 180

948-The best treatment for dermatitis herpetiformis


o Steroids
 Dapsone
o Astrin
949-A case of dermatitis herpitiformes what symptom improves with gluten free diet and not with dapson
 Intestinal velli atrophy

948-The best treatment for dermatitis herpetiformis


o Steroids
 Dapsone
o Astrin
949-A case of dermatitis herpitiformes what symptom improves with gluten free diet and not with dapson
 Intestinal velli atrophy

27
982-Blistering skin rash is a feature of the following dermatoses EXCEPT:
o Erythema herpiticum
o Erythema multiforme
o Sulphonamide allergy
 Erythema nodosum

1003-Pemphigus like antibodies are found in all except


 Thermal burn
o TEN
o Bullous SLE
o Erythema multiforme

1065-Best reliable test for DH diagnosisis


 Skin biopsy
 Immunofluorescence

1126-Patient with grouped vesicles and DIF show IgA deposits in epidermis:
o P. Erythematosus
 P. Herpetiformis(igG in epidermis)
o Dermatitis herpetiformis(igA in BM)
o Pemphigus-lupus overlap

1180-The most common neoplasm accompanying paraneoplastic pemphigus is:


 Chronic lymphatcytic leukemia
o Hodjkins lymphoma
o Adenocarcinoma
 Non-hodjkins lymphoma‫لو موجودة‬

1182-Cutaneous manifestation of Darier's disease may be exacerbated by: -


o Dapsoneo
o Betablockers
 Lithium
o Thiazides
o Antimalarial

1229-Patient with painful oral lesion ,skin lesion are suprabasal acantholysis, what not in this disease
(PV):
o Dsg3
o Dsg1
o Lectin
Bpag2

1240-IgA antiidodies to tissue trasglutaminase diffrentiate this disease from other groups
o DH
o Linear IgA disease

28
o Bullous SLE
o EBA

1268-Patient with bilateral flaccid axillary bullae.


 Pemphigus vulgaris
1331-Figure : small vesicles on hand and forearm of baby the histopathology shows deposition of igA at
dermoepidermal with subepiderma bullae with eosinophils and neutrophils what is diagnosis:
 Ch. Bullous dermatosis of childhood

1332-Child with vesicles in genials; chest; diagnosis


 IgA dermatosis

1495-Tzank smear is used to diagnose bullous pemphigoid


o True
o False
1496-Steroids play no role in treatment of bullous pemphigoid
o True
o False
1497-H.G may occur with the 1st menstrual cycle
o True
o False
1498-H.G occurs in the 3rd trimester
o True
o False
1499-Gluten free diet in iga dermatosis improves the patient condition:-
o True
o False
1500-Iga deposits in mucosa of DH not affected with treatment:-
o True
o False
1501-Best reliable diagnosis of DH is DIF of non lesional skin
o True
o False
1502-Dapsone is the drug of choice D.H
o True
o False

29
1503-Result in involved skin as well as uninvolved skin in D.H
o True
o False
1504-Gluten free diet is rapidly restore D.H
)‫ شهر‬21-5( ‫بعد سنة‬
o True
o False

1505-D.H like psoriasis, tend to occur in extensor surfaces


o True
o False

1506-D.H is more common in male


o True
o False

1507-D.H is common in old age


o True
o False

1508-D.H. is pruritic
o True
o False

1509-Jugenal biopsy shows characteristic findings with D. H


o True
o False
1510-In Cicatricial pemphigoid, iga is present against lamina lucida
o True
o False
1510-Oral lesions are common in bullous pemphigoid
o True
o False
1511-Pemphigus vegetans is usually localized in intertrigenous areas
o True
o False
1512-Pemphigus vulgaris may result in hyperpigmentation
o True
o False
1513-The type of pemphigus that have racial factor is pemphigus vulgaris
o True
o False
1514-Bullous pemphigoid is common at age 65-80
o True
o False
1515-In pemphigus vulgaris prednisolone is given at a dose of 100-200 mg/day
o True
o False
1516-Iry lesion in pemphigus vulgaris is a flaccid bullae on an erythematous base
o True

30
o False
1517-Steroids are contraindicated in herpes gestation
o True
o False

1570. The best reliable test for diagnosis of DH is:- DIF.

1624-All are right about Pemphigus folliacious except:- Mucosal lesion.

From the questions in page 80 :

122-CBCD “chronic bullous disease of childhood” HLA-B8


123-CBCD: granular iga (both normal and affected skin)
124-DH: Spontaneous remission (10%)
125-DH: Dapsone (ttt of choice)
126-DH: Sulphapyridine 1-1.5g/d (in dapsone intolerance)
127-HG:2nd trimester (onset)
128-HG: C3, igg (vs. PUPPP)
129-HG: morbidity/motality(vs.PUPPP)
130-BP: igg,C3 (immunoreactants)
130-BP: incidence of underlying cancer NOT ↑↑
130-SCPD (subcorneal pustular dermatosis) best treatment dapsone
130-EM (erythema multiform):H.S, orf, suiphonamides (major causes)
130-EM: deposits around superficial blood vessels
130-Darrier: onychorrhexis

From the questions in page 82 :

30) Most common type of pemphigus- pemphigus vulgaris


31) Rarest type of pemphigus- pemphigus vegetans
32) DOC for Dermatiis herpetiformis- Dapsone

34) Most common cause of non bullous impetigo- strepto> staph


35) Most common cause of bullous impetigo- staph

31
Connective Tissues Disesases
FANA Diagnosis
1- Peripheral SLE
2- Homogeneous Drug
3- Nucleolar Scleroderma
4- Speckled MCTD

=================================================
Polymyositis V/S Dermatomyositis V/S Inclusion Body Myositis
=================================================
►Affects Males > Females: Inclusion Body Myositis, In contrast to the other 2.
►Associated with Parasites (protozoa, cestodes, nematodes), tropical and bacterial
myositis:: Polymyositis
►Associated with malignancy:: Dermatomyositis (15%)
►Familial association:: Inclusion Body Myositis(in some cases)
►Systemic autoimmune diseases:: Frequently with Polymyositis and less frequently with the
other 2.
►Not associated with Drugs:: Inclusion Body Myositis. Drugs include penicillamine
(dermatomyositis and polymyositis), zidovudine (polymyositis), and contaminated tryptophan
(dermatomyositis-like illness).
►In contrast to polymyositis and dermatomyositis, where facial muscles are typically spared
(Dalakas 2010a; 2010b), mild facial weakness is very common in inclusion-body myositis
►Rash or calcinosis associated with:: Dermatomyositis
►Pattern of weakness
•Proximal muscles > Distal Muscles: In case of Polymyositis and Dermatomyositis
•Distal muscles > Proximal Muscles: Inclusion Body Myositis
►Responds to IVIG:: Dermatomyositis
►Muscle biopsy findings::
•"Primary" inflammation with the CD8/MHC-I complex and no vacuoles::Polymyositis
•Perifascicular, perimysial, or perivascular infiltrates, perifascicular atrophy::Dermatomyositis
•Primary inflammation with CD8/MHC-I complex; vacuolated fibers with -amyloid deposits;
cytochrome oxygenase–negative fibers; signs of chronic myopathy:: Inclusion Body Myositis

490-Different between DLE & SLE is


o Atrophy
o Photosensitivity
o Erythema
o Scaling
o Telangectasia
491-The most important histological features of DLE, for diagnosis is
o Hyperkeratosis
o Malpighian layer atrophy
o Liquefaction degeneration of the basal cell layer
o Patchy lymphocytic infiltrate
o Endothelial proliferation
492-Diagnosis of DLE is confirmed by:-

32
o ESR
 Skin biopsy
o Photopatch test
493-Complications of scalp DLE include all EXCEPT
o Ulceration
o Koebnerization
o Metastatic squamous cell carcinoma
o Cosmetic disfigurement
o Premature canities
494-Which of the following clinical findings distinguishes scalp DLE from follicular LP?
o A positive pull test result for anagen hairs
o Response to topical corticosteroids
o Development of lesions in non-scalp areas
o Erythema and scale affecting the center of the lesion in active disease
o All of the above
495-Drug induces SCLE
 Hydrachlorothiazid
o Hydralazin
o Hydantoin
o Procaine
496-All are manifestations of SLE except:-
o Malar flush
o Photosensitivity
 Telangectasia
o Chronic discoid lesions
497-Drug which most induce SLE
o Penicillin
o Procainamide
o Tetracycline
o Chlore hexidine
498-Drug most proven causes SLE
o Penicillamine
o Isoniazide
499--In SLE, the most important pathologic feature in connective tissue is
o Inflammatory infiltrate
o Edema
o Fibrosis
o Fibrinoid change
o Proliferation of fibroblasts
500-The cutaneous marker of high risk in SLE is –
o Oral ulcers
o Leg ulcers
o Development of multiple dermatofibroma lesions
o Levido reticularis
501-%of oral lesion in SLE
o 7%
o 8%
o 10%
o 20%
502-Most common haematologic finding in SLE is
o Lymphopenia

33
o Thrompcytopenia
o Leucopenia
o Mild normocytic normochromic anaemia
o Haemolytic anaemia
503-To assure high sensitivity in taking skin biopsy for immunofluorescence in SLE you have to avoid
the following site
o Expose skin sites
 Sites that are intensively treated
504-Positive lupus band- test in the normal sun exposed, occurs with :
o DLE
o SLE
o Pemphigus
o Scleroderma.
505-SLE all are false EXCEPT:-
o C3, C4 ↑↑
o No ANA +ve
o Photosensitivity
o Strawberry tongue
506-Severity of SLE is gauged
o Leukocyte
o Cd4
o Ss dna ‫ للخطورة‬risk
o Ds dna ‫ للمراقبة‬monitoring
507-Good prognosis is SLE depends on:-
o Liver affection
o Renal affection
o Lung affection
CNS affection
508-One of the most important factors in determining the prognosis of SLE ?
o Initial ANA titer
o Severity of arthritis
o Sex of patient
o Degree of renal involvement
o Leukocyte count
509-The renal manifestation most often associated with a poor prognosis in SLE is
o Glycosuria
o Proteinuria
o Pyuria
o Nephrotic syndrome
o None of the above
510-SLE in childhood
o Is more common in boys than girls
o Seldom manifests cutaneous lesions
o Often has negative fluorescent ANA tests
o Has a worse prognosis than in adults
o Does not occur
511-A patient with photosensitivity, facial rash and investigation showed +ve anti dsdna, ANA. Best
treatment to start with is
o Preden`isolone
o Hydroxychloroquine
512-The pattern of nuclear antibody that shows speckled pattern seen in:

34
o SLE.
o Scleroderma.
o Dermatomyositis.
o Mixed connective tissue disease.
513-One of the following statements is true:-
 Anti-ndna+ve in SLE patient means active nephritis
o Anti-Jolis+ve in cancer associated myositis
o Anti-nucleohistone are +ve in drug included SCLE
o Anti ss- DNA+ve means systemic sclerosis
514-All true about morphea EXCEPT:-
o Hypopigmentation
o Thickening of the skin
 Asymptomatic
515-Female patient have multiple skin lesions on abdomen and face which is hyper pigmented,
shiny,devoid of hair,indurated. Histopathology reveals collagen deposit in the dermis what do u suspect
(morphea):
o 25% of pt. Have anti ss DNA
o 50% have anti-cardolipin
516-The most common monoclonal gammopathy associated with scleroderma is:-
o Iga
o Igm
o Igg kappa
o Igg Lambda
o Ige
517-The histologic alterations in scleroderma typically begin
o In papillary dermis
o In reticular dermis above eccrine glands
o In perivascular collagen
o In the perappendageal collage
o In the dermal-subcutaneous junction area
518-Visceral affection of progressive systemic sclerosis (PSS) involve:-
o CVS
 GIT
o Lung
o Dihydrotestosterone
519-Most common organ involved in Scleroderma is
 Esophagus
o Colon
o Kidney
o Heart
520-Scleroderma all EXCEPT:
o Causes calcification of the skin
 Shows periorbital lesion
o Scl 70
o Raynaunds phenomenon is common than is SLE
521-About systemic sclerosis all are true EXCEPT:-
 Hepatosplenomegaly
o Swollen hands
o Hypopigmentation
o Atrophy of terminal phalanges

35
522-Heliotrope eyelids are seen in:-
o DLE
o SLD
o Dermatomyositis
o Scleroderma
o Rheumatoid Arthritis
523-Histopathology of Dermatomyositis all except
o Acanthosis
o Hydropic degeneration of basal cell layer
o Flattened epidermis
o Accumulation of acid mucopolysaccharides in the dermis
o Edema of the upper dermis
524-Pseudoscleroderma can be due to all EXCEPT
o Eosinophlic facilities
o Bronchial carcinoma
o Hypothyroidism
o Bleomycin
 Histidinemia
525-Scleredema subside after
o 1-2 year
o 4-5 years
o 8-10 years
526-Child had indurated swelling on neck upper trunk after pharyngittis when they consider resolved
(scleredema adultorum of buschke)
o 1-2 year
o 4-5 years
o 8-10 years
o 6-12 monthes
527-Eosinophic Fasciitis (Shulman syndrome) treatment
o Isotritnoin
o Corticosteroid
o Retinoid
o Antibiotic
o Cytotoxic drugs
528-Treatment for eosinophilic fasciitis
o Pencillamine
o Prednisolone
o Melphalan
529-Juvenile hyaline fibromatosis is characterized by
o An autosomal dominant inheritance
o Large cutaneous nodules, gingival hypertrophy and joint contracture
o The collagen present is type IV
o The main glycosominoglycan present in tissue is hyaluronan
All of the above

880-To assure high sensitivity in diagnosis of SLE by immunofluorescence you shold take skin biobsy
from
o Sun exposed involved sites
o Sun hidden involved sites

36
 Uninvolved sites sun exposed(diagnosis)
o Uninvolved sites sun unexposed(prognosis)
887-Heliotrobe erythema accurse with
o LE
 Dermatomyositis
898-Scleroderma can be associated :
 GIT problems
o Lung

905-Calcinosis, raynaud phenomenon, esophageal dysmotility, sclerodactyly, and telangiectasia ?


 Crest syndrome.

906-Dysplastic nevus syndrome (AD)all of following are true except:


 Autosomal recessive

907-Patient take steroid for treatment of dermatomyocitis he developed muscle pain take 40mg
cortisone+15mg mitotrixat since week the myopathy become worst and enzyme of muscle is normal the
probable cause
 Steroid induced myopathy
o Aging
o Mitotrixat
o Worst myopathy

979-Prognostic for SLE is


 Antidnadoblestand
o Ro-ssa

981-Parameters for type IV skin patient for hair removal with nd-yag laser-
 30 j/cm and with pulse width 50ms .

1001-Case of SLE 1st line of TTT


o Cs
 Hydroxychloroqiune
o Thalidomide
o Mtx

1022-RAST
 Detect IgE

1045-Antiro antihistone anti DNA


 SLE because of drug

37
1062-Drug which can cause SLE
o Tetracycline
o Isoniazid
o Pencillin
o Penicillamine the most
 All of the above

1064-Scleroderma
o Ig E
o Ig A
o Ig M
 IgG kappa

1088-Child has a tumor on the left shoulder, two years old, what is the best measure(Scleredema)
 Control(reassurance)
o X-ray
o Laser
o Surgery

1097-What is true regarding SLE except


o Photosensitivity may be one of the manifestations of
o Complement high
o Alarm regard to renal injury

1216-Gottron's sign is present in


 Dermatomyositis

1226-Pattern of nuclear antibodies in SLE:


 Spickles
o Nuclear

1248-Patient with photosensitivity , malar rash ( red, scaly ) developed generalized p foliaceus like lesion
, best investigation to confirm diagnosis :
o Skin biopsy for H/P
 Immunoflurocence staining
Peripheral eosinoph

1256-ESRD with hyperkeratotic papules, biopsy showed: transepidermal elimination of collagen, best
TTT:
o Emollient
 Steroids
1283-Earliest feature of tuberous sclerosis is ?
 Ash leaf spot
o Shagreen patch

38
o Facial angiofibroma
o Gingival fibroma
1289-The drug with the least side effects for the treatment of SLE is:
o NSAIDs
o Methotrexate
o Corticosteroid
o Hydroxychloroquin

1298-Red lunula in SLE


 Related to periungual erythema or chilblain lupus
o Land mark in SLE

1308-SLE with pemphgus


 Azathioprine,cyclophosphamide

1365-picture of old female has red face…a case of breast cancer since 2 years…recently develop red face
is it?
o cut LE
o chilblan LE
o Lupus timdus
 dermatomyositis

1415-Scleroderma 1st manifestation is swelling of hands & feet


o True
o False
o
1449-SLE may associated with peripheral neuropathy
o True
o False

1450-Dermatomyositis have malignant association in children


o True
o False

1451-En coup de saber is the linear variety of morphea


o True
o False

1462-Heliotrope rash & gottron's papules are the characteristic cutaneous lesions in dermatomyositis
o True
o False
o

1477-In dermatomyositis there is distal symmetrical muscle weakness

39
o True
o False
1478- In systemic sclerosis there is nail changes
o True
o False

1481-RAST is the best test for allergic dermatitis


o True
o False

1626-Drugs induce SLE: Hydralazine, Isoniazid, Procainamide.

From the questions in page 80 :

136-Behcet’s disease: young + (peak incidence)


137-Behcet’s disease Topical steroids (NOT effective ttt)
138-Behcet’s disease: Urethritis (NOT a manifestation, vs Reiter’s)
139-Reiter’s syndrome: HLA-B27
140-Pathergy test: pyoderma gangrenosum + sweet syndrome + Behcet’s
141-Acute febrile neutrophilic dermatosis: Myeloproliferative(major cause)
142-PG (pyoderma gangrenosum):Psoriasiform lesion NOT present
143-SLE:DIF high sensitivity (areas NOT intensely treated)
144-SLE:Skin lesions: Livedo(patient at risk)
145-SLE: Kidney functions (best prognostic factor)
146-SLE: ndna(active with severe nephritis)
147-DLE: lupus band test NOT positive (positive in normal sun-expposed)
148-DLE: DIF high sensitivity (areas NOT intensely treated)
149-DLE: DIF high specificity (areas NOT sun-exposed)
150-MCTD: (Mixed connective tissue disease):specified nuclear igg.
151-SS:(systemic sclerosis)GIT(most affected organ)
152-SS: igg (predominant Ig)
153-Sjorgen-Larsson: Anterior congenital cataract is NOT manifestation
154-Inflammatory bowel disease; skin affection: PG, SCPD,EED
155-Crohn’s disease :oral lesions (the more common ones)

40
Cosmatology & LASER

37-Epidermis of the facial skin in aged persons is characterized by all except


o Thinning of the malpighian layer
o Reduced number of functioning melanocytes
o Glycogen accumulation
o More marked involutional changes than are seen in non-light exposed skin
o Absence of stratum granulosum

69-Patient treated with PUVA should be examine by:


o Neurologist.
o Ophthalmologist.
o Git specialist.
70-The earth rays contains most concentration of UVA
o 10am – 2pm
o 11am – 1pm
o 10am – 3pm
o 11am – 4pm
o 11am – 2pm

70-The earth rays contains most concentration of UVA


o 10am – 2pm
o 11am – 1pm
o 10am – 3pm
o 11am – 4pm
o 11am – 2pm

71-Immediate pigment darkening (IPD) is produced and maintained by:-


o Visible light
o UVB
o UVA
o UVC
o All
72-Immediate tanning reaction occurs with –
o UVA
o UVB
73-Immediate tanning:
o 320-340
o 320-400
o 290-320
74-Immediate tanning:
o 8-12 hours
 1-6 hours
o 12-24 hours
o 24-36 hours

41
75-Sun burn peaks
o 4-6 hours after exposure
o 24 hours after
76-Psoralene reaches the skin after oral administration within:-
 1-2 hours
o 3-4 hours
o 10-12 hours
o 20-24 hours
77-Patient , whight 70 kg , for PUVA treatment what the dose of psoralin :
o 20mg befor1h
 40mg befor2h
o 20mg befor2h
o 40mg befor1h

o 78-Skin diseases responsive to PUVA therapy include all except


o Chronic hand eczema
o Sarcoidosis
o Urticaria pigmentosa
o Solar urticaria
o Actinic keratosis

79-The following dermatosis can be aggravated by exposure to UVR all EXCEPT


o Acne vulgaris
o Atopic dermatitis
o Lichen planus
o Pemphigus foliaceous
o Lichen sclerosus et atrophicus

80-UVB tanning appear after :


o 1 houre
o 24 hours
 48 hours
o 72 hours

81-UVB tanning appear after early hours delayed UVB tanning appears after -
o 36 hours
o 24 hours
o 72 hours
o 48 hours

82-Patient with psoriasis take UVB and not respond which following drugs :
o Acitritin
o Methotraxate
o Cyclosporine
83-UVB alleviates:-
o Pruritis with biliary cirrhosis
o Pruritis with diabetes
o Pruritis of hemodialysis
84-The highest SPF available at this time is
o 15

42
o 19
o 30
o 39
o 50
85-The best sun cream (sunscreen )
o Paba (para-aminobenzoic acid)
o Titanium

96-Dermatologic side effects of radiotherapy include all except


o Pigmentary changes
o Permanent alopecia
o Fibrosis
o Radionecrosis
 Acneiform eruption

86-For prophylactic from infection , before procedure , the antibiotic should be given
o 6 hours
o 14 hours
o 24 hours
87-High risk patient for cardiac diseases scheduled for operation (cosmotic) should recive antibiotic
(prphylactic).To avoid skin infection:-
o 1 hour before surgery
o At time of surgery
o 6 hours after surgery
1 day after surger

867-Which of the following can use UV:


o SCC
o Nodular BCC
o Superficial BCC
o Kerato acanthoma

43
821-Preferred laser used in treatment of port wine nevus in children is:-
o Nd-Yag laser
o 565 pulsed dye laser
o 585 pulsed dye laser
o Argon laser
822-The wavelength of the Excimer laser that can be used in the treatment of vitilligo is
o 755 nm
o 311 nm
o 585 nm
o 308 nm
o 1064 nm
823-The "gold standard" laser for laser-assisted hair removal in people with skin types I-III is
 Alexandrite laser
o Ruby laser
o Diode laser
o Nd: Yag laser
o Co2 laser
824-The Q-switched ruby laser delivers:-
o UV light
o Green light
o Yellow light
o Infrared light
o Red light
825-Color of q switched ruby:
o Infrared
o Red
o Green
826-Patient with keloid residual after treatment with intralesional steroid best laser
o CO2
o 585 dye laser
o IPL
o Fraxel
827-Magnolian spot best ttt
o Q switched ruby
o PDL
o Co2 laser
o IPL
828-Nevus of OTA best treatment
o Q-switched Nd:YAG(532)
o Q- switched ruby
o IPL
Long pulse

933-Treatment of 3y pt with mongolion spot on his back :


 No treatment
o Q switched nd-yag
o Alexandrite

934-SPF 15 definition
 Needs 15 fold more UVB to cause erythema

44
o Needs 15 fold more UVA to cause erythema
o Needs 15 fold more UVB and UVA to cause erythema
o Needs 15 min of UVB to get erythema

976-Psoralin ultravioletray A (PUVA) all of the following are true except:


o Useful in vitiligo
o Contraindicated in SLE
 Used to treat some childhood intractable dermatosis
o Increase the risk of basal and squamous cell cancer

987-Delayed inflammatory erethema UVB


o 1 to 6hrs
o 6 to 12hrs
 12 to 24hrs
o 18 to 36hrs

999-Best adjuvant treatment with NB-UVB for plaque psoriasis


o Methotrixate
o Cyclosporine
 Acitretine
o Hydroxylurea

1018-Immunotherapy
o Radiotherapy

1047-Immediate tanning caused by:


o 320-340
 320-400
o 290-320
1048-The wave length of UVA2 is
 320-340
o 340-380
o 340-400
o 380-420
1049-Wave length of UVB
o 320 - 425 nm
 290 - 320 nm
o 311 - 312 nm
o 400-760nm
1050-Difference between UVA1 and UVB is:
 UVA1 goes deep in dermis than UVB
1051-Site to detect minimal erythematous dose
o Face
 Abdomen
o Leg
o Neck

45
o Arm&dorsum of the hand

1063-Red tattoo which laeser


o Q switch alexandrite
o Q switch rupy
 Double frequency ND-YAG 532nm
o Pulsday

1066-All are used for hair removal,all are true EXCEPT


o The shorter long of waves more target melanin
o The long duration makes more heat on skin
o The presence of edema follicular and redness disappear
o The presence of pigmentation and hypopigmentation relieved after 6 months

1113-The absence of light below 290nm at the earth surface from solar radiation is due to:-
atmosphere ozone and other gases

The stratosphere starts just above the troposphere and extends to 50 kilometers (31 miles) high. The ozone layer,
which absorbs and scatters the solar ultraviolet radiation, is in this layer.

1119-About hair removal with Alexandride ,Nd :Yag ,Ruby what is wrong
o Perifollicular edema and erythema are the commenest side effect
o Hypo or hyperpigmentation will subside after 6 monthes
o The shorter wave length the more destructed melanin
 Increase pulse duration will reduce number of hair affected

1122-Delayed erythema caused by UVB develops with a peak of after :


o 6-12 hrs
 12-24hrs
o 24-48hrs

1139-Patients with end stage chronic renal failure (CRF) suffering from hyperkeratotic papules on biopsy
there was transepidermal elimination of collagen. What the safest treatment:
o Steroid
o Emollient
 NB UVB
o Allopurinol
o Isotretinoin

1198-Boiling degree of liquid nitrogen


 −195.79

1253-Pt with facial furrows and exagerated nasolabial fold. What is TTT?

46
 Botulinum neurotoxin injection

1282-After oral injection of 8-methyloxypsoralen, how long an interval should be allowed before UVA
radiation is given in PUVA treatment?
o 1 hour
 2 hours
o 3 hours
o 4 hours

1297-Woman ephelides (Freckles) on face


 Q-switched ndyag 532
o Co2 laser
o Erbium glass

1310-Re-puva therapy
 Oral administration of a retinoid derivative and systemic PUVA treatment

1315-phototherapy in neonatal hyperbilirubinemia


 460-490 nm

1397-Erythematosus minor dose is the dose of UVB which leads to erythema after application on the
terminal part
 True

1413-Immediate tanning occurs due to UVA while delayed tanning occurs due to UVB:-
o True
o False

1418-PUVA is the treatment of choice in PLE:-


o True
o False

1419-70-80% of total life dose of solar radiation is received before the age of 20 years
o True
o False

1420-MED is the minimal dose of UVR that induce Erythema & is measured by mj/cm or j/m
o True
o False

1422-Polyester clothes reflect more UVR so it is used more than cotton clothes in hot climate
o True
o False

1523-Penetration of the skin by UV light is:- greater for longer wavelengths than for shorter
wavelengths.

47
1523-Delayed UVB tanning appear after:- 72 hours.
1523-Immediate UVA tanning appear after few min and fade within 6-8 hours
1523-UAV1 range:- 340-400 nm.
1523-UVA2 range:- 320-340 nm.
1523-UVB range:- 290-320nm.
1536-Common complication of cryotherapy:- hyperpigmentation but serious complication is scar

1538-Common complication of cryotherapy:- hyperpigmentation but serious complication is scar.

1581-The absence of light below 290nm at the earth surface from solar radiation is due to:- atmosphere
ozone and other gases
1595-The photosensitizing action spectrum of the furocoumarines is:- 300-360.

1630-Dose of 8MOP in patent 70 kg before PUVA:- 70 X 0.4 :- 28 mg.

1631-The usual site to detecting minimal erythematous lesion:- Abdomen.

From the questions in page 79 :

63-Solar radiation: vitamin D synthesis, psychological well-being, healthy skin


64-UVA: striking earth surface 150 folds UVB
65-UVA: maximum erythemogenically 9-1mm
66-UVA: quick tanning (oxidizes preformed melanin)
67-UVA: wrinkles (fibroblast injury, mild dermis)
68-PUVA: psoriasis, atopic eczema, pityriasis rosea, pruritus with hemodialysis (uses)
69-PUVA: sunburn i.e erytherma, pain commonest side effect
70-UVB (acute)erythema,12-24 h
71-UVB-delayed tanning,72 h new melanin formation
72-UVB (chronic)aging, cancer

48
Drugs

Cytochrome P450 inducers (↓metabolism) Cytochrome P450 inhibitors (↑metabolism)


 Phenobarbitone ↓↓  Fluconazol ↑↑
 Phenytoin  Itraconazol
 Rifampin  Ketoconazol
 Grisofulvin  Cimitidine
 Cortisol  Erythromycin
 Carbamazepine (Tegratol)  Miconazol
 Clarithromycin
Drugs ↓ Cyclosporine A blood level Drugs ↑ Cyclosporine A blood level
 Phenytoin ↓↓  Fluconazol ↑↑
 Rifampin  Itraconazol
 Carbamazepine (Tegratol)  Ketoconazol
 Methylprednisolone
 Allopurine
 Erythromycin
 Clarithromycin

99-Contraindications to oral retinoids include all except


o Pregnancy
o Patients at risk for liver toxicity
o Chronic use in children
o Concomitant use of methotrexate and oral retinoids
o Aphakia
100-The difference of side effect of etretinate and isotretinoin is that
o Isotretinoin procedure
o Facial dryness + redness of lips
o Alopecia
101-Use of minoxidil for 96 day will result in:
o Increase hair thickness &weight
o Increase hair length & decrease weight
o Increse hair thickend only
102-Acyclovir-resistant herpes virus infection is best treated by
o Cidofovir
o Indinaivir
o Granciclovir
o Foscarnet
o Lamivudine

49
103-Penicillamine induce :
o Sle
o Dermatomyositis
o Pimphigus
o Polymyositis
o All above
104-Patient who is patch test +ve to ethylendiamine group of antihistamines should avoid
o Hydroxyzine (Atarax)
o Chlorophenramine
o Terfenadine
o Diphenhydramine
105-Dose of nifedipine in treatment of Raynaud phenomena
o 10 mg twice daily
o 20-30 daily
 30-40 daily
o 10-80 daily
106-The administration of erythromycin can even the blood level of:-
o Theophylline
o Oral contraceptives
o Hydrochlorothiazide
o Insulin
o 13-cls retinoic acid
107-Erythromycin toxicity if take with
o Clindamycin
o Lincomycin
o Aspirin
o NSAID
o Warfarin
108-Lindane is no longer the treatment of choice for scabies because of
o Cardiotoxic effects
o Nephrotoxic effects
o Neurotoxic effects
o Hepatotoxic effects
o All of the above
109-Phenol peeling causes:
o Cardiotoxicity(cardiac arrythmia)
o Renal toxicity
o Neural toxicity
110-The most safe form of procaine is:-
 Novocain
o Lindocain
o Lidocain
111-Patient who is sensitive to procaine can be given
o Lidocain
o Benzocaine
o Novicain
112-Best topical anesthesia with:
o 1%lidocaine+adrenaline
 2%lidocaine
113-The best for nerve block of mohs micro surgery
o Procaine alone

50
o Lidocain alone
 Lidocain + epinephrin
o Bupivacine + epinephrin
114-Alternating days of prednisone eliminates the increased risk of corticosteroid induced :
o Osteoporosis
o Posterior subcapsular cataracts
o Avascular necrosis of the hip d.
o Growth retardation
115-The following tests can assess the efficacy and strength of topical corticosteroids except
o The vasoconstriction test
o The repeated open application test
o The atrophy test
o The erythema test
o Pyrexal erythema test
116-The most frequent adverse effect of topical corticosteroids in children is
o Scarring
o Local irritation
o Contact sensitization
o Skin discoloration
 Skin atrophy
117-There is little evidence of a need to taper off a course of corticosteroid therapy (single morning dose)
of duration less than:-
o One week
 Two weeks
o One month
o Six weeks
o Two months
118-Cause of hypertension
o Dexamethasone
o Prednisolone
o Hydrocortisone
o Triamcilone
119-Drug which is most likely to cause hypertension in equivalent dose
o Prednisolone
o Dexamethasone
o Hydrocortisone
o Betamethasone
o Triamcinolone
120-1 gm of dexamethasone equal to how many .gm of triamcinolone:
o 1
o 2
o 5
o 10
121-The following drugs should not be used concomitantly with dapsone all EXCEPT
o Methotraxate
o Primaquine
o Zidovudine
o Chloroquine
 Oral contraceptives
122-Dapsone is most useful in patients with:-

51
o Guttate Psoriasis
o Psoriatic Erythroderma
o Psoriasis Follicularis
o Psoriasis rupiodes
o Palmoplantar Pustular psoriasis
123-Patient took dapsone after 2 days developed bluish lip &hand what to give
o Acetylcystein
o Ethanol
o Methelyne blue
o Chrcool
124-The most prominent side effects of Dapsone is:-
o Hemolysis
o Liver damage
o Renal damage
o Bone marrow depression
125-Methotrexate before use
o Liver enzyme/function
o Liver biopsy
126-Current recommendation for monitoring methotrexate hepatoxicity in patients without the risk factors
for liver disease is
o Liver biopsy before starting methotrexate
o Liver biopsy after the first 200 to 400mg of methotrexate
o Liver biopsy when patients have been treated with 1 to 1.5g of methotrexate
o Liver biopsy when patients have been treated with 3g of methotrexate
 Liver biopsies are not necessary as long as patient have liver function tests drawn monthly.
127-Methotrexate what right
o No need to biopsy in every case
o Liver enzyme is necessary
o Pneumonia is more common with rheumatoid arthritis than psoriasis
 All of the above
128-Drug causes hepatotoxicity by cumulative effect:-
o Ketoconazole
o Methotrexate
o Tetracyclines
o Ampicillin

129-Side effects of methotrexate include all except


o Hepatotoxicity
o Phototoxicity
o Chills and fever
o Cytopenia
o Keratitis and conjunctivitis
130-In methotrexate hepatotoxicity :
o Liver biopsy is not necessary in patient lack clinical and laboratory evident of liver disease.
o The hepatic c fibrosis in patient with rh arthritis is more benign than in patient with psoriasis.
o The pnemonitis side effect is far more benign in patient with psoriasis than in patient with rheumatoid
arthritis.
o Liver enzymes are essential before treatment despite controversy about their role.
o All above
131-Methotrexate what the latest
o Teratogens

52
 Myeloplstic
o Bone narrow suppression
o Stomach upset
o Hepatotoxicity

132-The principal route of excretion of methotrexate is


o Biliary
o Renal
o Cutaneous
o Respiratory
o None of the above
133-Cyclosporine is
o Nephrotoxic
o Hepatotoxic
o Hepatonephrotoxic
 All true

134-Cyclosporine is commonly:-
 Nephrotoxic
o Hepatotoxic
o Hepatonephrotoxic
o All of the above
135-Cyclosporine levels are reduced by:
o Ketoconazole
o Cimetidine
o Rifamficin
o Allopurinol
136-Cyclosporine level is increased by all EXCEPT:-
o Erythromycin
o Rifampicin
o Ketoconazole
o Allopurinol
137-Drugs affect cyclosporine metabolism:-
o Rifampicin
o Erythromycin
o Ketoconazole
o All of the above
138-Drug that increase level of cyclosporine is
o Ketoconazole and fluconazole
o Phenoparbitorat
o Rifampicin (rifampin)
139-One of the following, the most important side effects of cyclosporine therapy are:-
o Leukopenia and bone marrow suppression
o Granulocytopenia and infection
o Hepatotoxicity and nephrotoxicity
o Neurotoxicity and temporary dementia
o Colitis and hemorrhagic cystitis

140-Drug more safe in pregnancy is


o Gentamicin
o Ampicillin

53
o Ketoconazole
141-Which statement of the following is untrue regarding Itraconazole?
o It is a triazole
o The absorption is increased by food
o Pulse dosing may limit potential toxicity
o It is fungicidal
o Gastric acid secretion suppressors produce erratic or decreased absorption
142-Ketoconazole not used with:
o Aspirin
o Theophylline
o Erythromycin
 Astemazol
o All of above
143-Fluconazole, what is wrong:-
o Cannot take in liver disease
o Half live 30 h
o Taken every one week
o The renal excrtectis not important

144-As the half life of fluconazole is long , it can be use every:


o Day
o Week
o 2 weeks
o 1 month
145-About fluconazole what is wrong:-
o Can be given every week
o Triazole group
o Metabolism not affect the kidney function
o Its half-life 24:-30 hour
o Mainly renal excretion
146-Itraconazole levels may be elevated if the patient is concomitantly taking which of the following
drugs-
o Cimetidine
o Phenobarbitone
o Phenytoin
o Rifampin
o Carbamazepine
147-Inhibition of squalene epoxidase is the mechanism of action of
o Nystalin
o Fluconazole
o Itraconazole
o Ketoconazole
o Terbinafine

148-The maximum daily dose of amphotericin B does not usually exceed


o 1 mg/kg body weight
o 10 mg /kg body weight
o 100 mg/kg body weigh
o 1 gram
o 10 grams
149-Drug that have the greatest potential phototoxic reaction:-

54
o Benoxaprofen
o Doxycycline
o Sulphonamides
150-The most photosentizing drug is:
o Chloroquine
 Diphenylhydramine
151-All of the following are photoaggravated EXCEPT
o LE
o Herpes simplex
o Dermatomyositis
o Pimphigus
o Xeroderma pigmentosa
152-Systemic photosensitivity caused by which group of drugs
o Sulphones
 Sulphonylureas
153-The most common cause of drug allergy:
o Morbilliform exanthema.
o FDE.
o Hyperpigmentations.
154-Which of the following drugs photosensitivity
 Sulfonylurea
o Sulfones
o Hydantion
o Chloroquine
155-All of the following are photoexaggerated EXCEPT:
o Herpes simplex
o Dermatomyositis
o LE
o Xeroderma pigmentosa
o Pemphigus vulgaris
156-Photo patch test can read
o 24h
o 36h
o 48-72h
o 96h
157-Most common sensitivity test (skin test)
o Prickle test
o Patch test
o Rast test
158-Photosensitizing drugs include all EXCEPT
o Chloroquine
o Nsaids
o Thiazides
o Sulphonamides
 PABA
159-What is best cytotoxic drug can be used in patient with severe ocular damage
o Corticosteroid
o Azathrioprin
o Cyclophosphamid
o Infleximab

55
160-Cytotoxic drug safe on eye
o Azathioprine
o Methotrexate.
o Cyclosporine
o Mycophenolate mofetil
 Cyclophosphamide
161-Fluorouracil inhibits DNA synthesis by
o Transfer RNA blockage
o Dihydrofolate reductase blockage
o Thymidylate synthetase blockage
o Adenylcyclase blockage
o None of the above
162-Immunosuppressive therapy
o Metoterxat
o Cyclosporine
o Azathioprine
o All
163-Imiquimod when used for treatment of genital warts , what is the most popular side effect
o Ulceration
o Erosion
o Pain
o Dyes.

164-Which of the following TNF inhibitors are monoclonal antibodies?


o Adalimumab and infliximab
o Adalimumab and Etanercept
o Etanercept and infliximab
o Etanercept and Efalizumab
o None of the above
165-Thalidomide use in treatment of
o Psudoxanthoma elasticum
o Parapsoriasis
o Pyoderma gangrenosum

807-In relation to fixed drugs eruption which is TRUE:-


o Is cell mediated immunity
o Is type in hyper sensitivity
o Toxic effect of drug
808-One of the following drugs may cause eruptive xanthoma:-
 Retinoids
o Ampicillin
o Dapsone
809-Most common clinical manifestation of drug eruption is:-
 Morbilliform eruption
o Lichenoid drug eruption
o Photo allergy
o Fixed drug eruption
o T.E.N
810-The most common form of drug eruption is:-
o Bullous

56
o Fixed
 Exanthematous
811-Onset of exanthematous drug eruption after taking therapy is
o 2 weeks
o More than 3 weeks
o 1 month
o 3 months
812-The only drug that cause anetoderma is:
o Delatin
o Pencillamine
o Procainamide
810-To diagnose drug reaction:
o Skin test.
o Brick test.
o Rast test.
o Increase blood oesenophil.
o None of the above.

860-Patient allergic PAPA ( para amonic benzoic asid ) wich treatment you don’t give
o Pencillin
o Gunclon
o Aminglicoside
o Sulphanomide

871-Antibiotic ci of patch test +ve what is contraindicated


 Hydroxyzine hydrochloride

877-Elimination of terbenafine is increased by


o Excreted mainly in sebum not in sweet
o Metabolized in liver
o Excreted into urine 80% &20% into faeces
o Cimitidine increased terbenafine
 Phenobarbital oral decreases levels of terbinafine oral by speeding up drug metabolism.

912-Most common complication of azathioprine


o Hair fall
 Leukopenia
o Pancytopenia

916-A 42 years truck driver presented with papulovesicular lesion on the penis since 2 weeks and after
one week suffer from firm, hard, painfull lymph node in right side then notice sinuses and discharge. He
received treatment from dermatologist and after 2 weeks the case become worsen with generalized
lymphadenitis (LGV). What is the best treatment :

57
o Penicillin benzathin
o Oflaxacin
 Tetracycline
o Acyclovir

918-Lichenoid drug eruption-


 Tetracycline
o Penicillin
o Erythromycin
o Ciprofloxacillin
952-What true about fluconazole : diflucan
o Decrease carbamazipine
o Asother antifungal fromazoles
 Triazole difluorobenzyl 1,2,4 triazole
o Not related tor enal
o Half life 30 to 37 hours

958-Most common from long term treatment with retinoids:


o Renal
o Cirrhosis
 Skeletal hyperosteosis

969-Chloroquine may produce


 Premement neurotoxic lesion of retina

972-Erthromycin can be incraesed by


 Hypoglycemic drugs
o Cis 13 retenoic acid
o Insulin

973-Erythromycin increases the level of the following drugs EXCEPT


o Opc (Oral Contracepitve pells)
o Theophylline
o Chlorpromazine
o Glycburide

975-The administration of erythromycin can even the blood level of:-


 Theophylline
o Oral contraceptives
o Hydrochlorothiazide
o Insulin

983-All of the following cause photosensitivity EXCEPT:


o Lithium

58
 Propranolol
o Tetracycline
o Chloropromazine
o Chloropropamide

978-Azathioprine side effect is


o Alopecia
o Malignancy
o Tetralognecity
o Hepatoxicity

988-Patient is using sulphonamide and had allergic contact dermatitis before and afraid of cross reaction
with ttt
o Neomycin
o Ehylendiamine
o Hyroxizine
 PPD (para-Phenylenediamine)

994-Female with acne and is taking metformin for treatment of polycystic ovary what is your choice
o Finisteride
 Spironlactone
o Acitretin

1036-A 56 year-old man presents with blue-gray discoloration on his face, ears, and dorsal hands. What is
the most likely offending agent?
o Minocycline
 Amiodarone
o Chloroquine
o Clofazimine

1039-Steroid tapering
 No need less than 2week
o No need less 20g/day
1040-Steroid topical more potent
 Clobetazole propionate
o Betamethazone dipropronate
o Clobetasole diproponate

1043-Acyclovir low dose is recommended if recurrent HSV rate of


 1-2m

59
o 2-4m
o 4-6m
o Year
1044-Antimalarial TTT check for ophthalmslogist at least
o Every 1-2m
o Every 4-6m
 Every 6-12m
o A very rare side effect is damage to

1046- 6 year boy, 3-day he has multiple yellow, crusted erosions on the cheeks, chin, and upper
extremities .the rest is normal. Treatment for this condition ?
o Oral acyclovir
 Oral cephalexin
o Topical ketoconazole
o Topical 2% hydrocortisone

1952-Which of the following medications would not induce ochronosis ?


o Cream containing hydroquinone
o Antimalarials
 Antihypertensives (Beta blockers)

1057-Which supplements decrease hemolysis associated in patients taking dapsone?


o Vitamin A
o Vitamin B6
o Vitamin D
 Vitamine E

1076-Bone marrow transplant immune disease what druges don't use


o Immunosupresent
 Retinoids

1077-Pt. Has asymptomatic papule & plaque on trunk back sole a generalized lymphoadenompathy taken
ttt and after 24 hours he become worse what he take
 Pencillin
o Erthromycin
o Clindamycin
o Dapson
1078-Pt.has popul itching under arms worst in summer and there ishyperkeratosis in pathology what is
the best TTT (fox-fordycedisease)
 Ritinoid cream
o Tacrolimus cream
o Pova
o Ksbutriaul cream

1094-Any following medications cause skin orange


 Rifadin (rifampin)

60
1100-An immunocompromised patient presents with a painful vesicular rash that has a dermatomal
distribution. The most appropriate management for this patientis:
o Oral acyclovir
o Analgesics and drying compresses
o Systemic corticosteroids
 I V acyclovir and hospital admission
1126-Most potent antihistamine
 Doxepin,citrizine
o Hydroxyzine hydrochloride
o Hydroxyzine pamoate
1149-Etretinate not given in a pt with high risk of
o DM
o HTN
 hyperlipidemia

1167-Cause of fillagellate hyperpigmentation


 Bleomycin
o Rifampicin
o Chloroquine

1168-The most important side effect of isotretinoin:


 Teratogenicity
o Eruptive xanthomas
o Facial erythema
o Chelitis
1169-Stimulation of keratinocyte by:
o TNF alfa
o Interferon alfa
o Interferon Gamma
 TGF beta

1171-Fluconazole mechanism of action


 Lanosterol 14 demethylase
o Squalene epoxidase
1192-A 23 yr old pregnant diabetic female comes with lake of pus lesions on skin (pustular psoriasis).
Treatment is
o Methotrexate
o Azathioprine
o Retinoids
 Cyclosporine

1205-Drug that lead to orange color in contact lenses of patients :


 Rifampicin
o Erythromycin

61
o Metronidazole
o Tetracycline

1221-Drugs for delusional parasitosis


 Pimozide

1233-Warfarin
o Telogen effluvium

1235-TLR toll like recepter ,what is TRUE:


o Probiotic use TLR
 TLR used in ttt of acne vulgaris

1258-Patient taking dapsone 100 mg /day he will suffer from:


 Hemolysis
o Agranulocytosis

1267-Grisofulvin not used in these cases EXCEPT:


o Hypersensitivity
o Hepatic
o Renal
 Pulmonary dysfunction
o Porpheria

1313- The best mineralocorticoid


 Methylprednisolone

1371-Papules on both knees what drug cause it (sarcoidosis and EN)


o Tetracycline
o MTX
o Isotretonin
 Sulphonamides

1417-The difference between side effects of etretinate & isotretinoin is that isotretinoin cause facial
dryness & redness of lips
o True
o False
1424-A major advantage of intralesional steroid injection is that it can be done without incising or
draining the lesion, avoiding the possibility of scar formation:-

62
o True
o False
1452-B. Blockers may exacerbate psoriasis
o True
o False
1453-Fixed drug reaction may occur in mucous membranes
o True
o False
1454-Fixed drug eruption is type hypersensitivity
o True
o False
1455-Antimicrobial agents are the commonest drugs cause drug reaction
o True
o False
1456-Exanthematous reaction usually appears a week after taking the drug
o True
o False
1453-In case of drug taken for 1st time, reaction will occur in the 1st 7 days
o True
o False
1457-Unpredictable drug reaction is dose independent
o True
o False
1458-Intolerance is the inability of the individual to tolerate therapeutic doses
o True
o False
1496-Drowsiness caused by antihistamines is considered a type of drug reactions
o True
o False

1529-Drugs cause acne form eruptions are:- Lithium, Hydantion, Steroid, Androgen, Contraceptive
pills.
1530-Retin A in acne treatment give approving after:- 6 to 8 weeks.
1531-Retinoid not given to patient with:- hyperlipidemia.
1551-The following drugs decrease the serum level of itraconzol:- phenytoin, rifampicin.
1552-Normal sebum content:- Waxester + Squalene + Cholesterol Esters (Wsce) ‫ويسكي‬
1553-Staph is sensitive to all except:- vancomycin.
1554-The least effect of this antibiotic on the soft tissue infection:- doxycyclin.
1555-Testosterone is converted to dihydrotestosterone in:- target organs.
1567-What is the most potent steroid:- clobetasone propionate.
1568-The principal route of excretion of methotrexate is:- renal.
1569-Orally administered methotrexate is best absorbed:-on an empty stomach,
1571-Phenol toxicity on:- cardiac, renal.
1591-Acyclovir is given as prophylactic in recurrent Herpes simplex:- 200mg ∕ 5 time or 400mg ∕ 3 time
1597-The photosensitizing action spectrum of the furocoumarines is:- 300-360.

1606-Topical therapy with salicylic acid preparations:- is very unlikely to result in dangerously
elevated serum salicyliate levels in adults
1614-The commonest side effect of hydroxyurea therapy as used for psoriasis:- anemia.
1615-A side effect of chloroquine therapy, peculiar to blonde and red hair patients is:- bleaching hair.

63
1619-The characteristic follicular eruption seen in patients on lithium carbonate is:- hyperkeratotic,
extensor.
1621-Thiazide cause:- Lichenoid drug eruption.

1641-The drugs with the appropriate characteristics:-


Cyclophosphamide alkylating agent, potent immunisuppresant effect,
Azathioprine purine analoge, potent anti inflammatory effect,
Methotrexate potent immunisuppresant effect,
Cytosine arabinoside potent immunisuppresant effect,
Azaribine none

From the questions in page 81 :

174-Azthiprine: T-cell
175-Cyclophosphamide: B-cell
176-Allylamines; Disrupt ergosterol synthesis earlier (squalene epoxidase)
177-Fluconozale: Disrupt ergosterol synthesis: later
(primarily: lanosterol 14-a demethylase;a cytochrome 45dependent)
(secondarily:sterol A22-desaturase;a cytochrome 450-dependent)
178-Itraconazole: same as fluconazole(both are triazoles)
179-Itraconazole: Demethylase, a cytochrome P-450-dependent enzyme
180-Fluoconazole: Demethylase, a cyctochrome P-450-dependent enzyme
181-Itraconazole. Terbinafine: reach the nail plate through nail bed (faster)
182-Griseofulvin, ketoconazole: reach the nail through nail matrix 18m
183-Aspirin: Urticarial
184-Azathioprine: malignancy
185-Bleomycin: Black nails:
186-Cyclophosphamide . Myelosuppression. Haemorrhagic cystitis, azoospermia
187-Cyclosprines: Nephrotoxicity
188-Cytotoxics; anagen effluvium
189-Dapson hemolytic anemia, agranulocytosis
190-Erythromycin: Hetotoxicity,Steven-johnson
191-Etretinate: hair thinning
192-Griseofulvin: photosensetivities SLE exacerbation
193-Ichthyotic: Triparanol
194-Isotretinoin: dryness(worst:teratogenicity)(contraindication: ↑↑lipids
195-Itraconazole: Hepatitis, cholestatic jaundice
196-Ketoconazole: nephrotoxicity
197-Lichenoid: Gold,color developers
198-Lindane: neurotoxicity
199-Lithium: Acne steroid, Darrier Methotrexate: Myelosuppression,hepato(dose-dependent)NOT nephro
200-Minoxidil: ECG changes, ↑↑ hair thickness and weight
201-PABA:NOT with sulfoamides
202-Penicellamine: Anetoderma, Atrophoderma, Morphea, Pemphigus.
203-Penicillin: Morbilliform (commonest rash:NOT FADE)
204-Photosensitivity:Diphenyhydantoin
205-Photoxicity:Tetracyclines,frusemide,sulphonamides,thiazides,phenothiazine,benoxaprofen,
naproxen,griseofulvin.
206-Pigmentation:chlorpromazine

64
207-Psoralens: Occular damage
208-Quinones: Retinal toxicity
209-Red yellow tattoo: Cadmium sulphide
210-Steroids: Acneiform
211-Teracycline: Onycholysis:
212-Thalidomide: Teratogenesis
213-Topical anesthetics: Allergic contact dermatitis

From the questions in page 84 :

106-Commonest agent for leucoderma- paratertiary butyl phenol (PTBP)

107- Commonest agent for hair dye allergic contact dermatitis- paraphenylene diamine (PPD)
108- Commonest agent for footwear allergic contact dermatitis- Mercaptobenzothiazole (MBT)

65
Eczema,Dermatitis & pruritis
41-Concerning lichenification all are true EXCEPT:-
o Thickening of epidermis & accentuation of skin markings
o Due to rubbing or scratching
 Occurs in acute dermatitis

167-What is the following disease has genetic susceptibility (Hereditary disease)


o EM
o EN
o E Margentium
o Eczema

168-Disease which can be caused by immunological reaction is:-


o Urticaria
o Seborrehic dermatitis
 Contact dermatitis
o Atopic dermatitis
169-The primary histologic feature of eczema is
o Spongiosis
o Acanthosis
o Hyperkeratosis
o Vasodilation
o Lymphocytic infiltration
170-All are similar in histopathology EXCEPT:-
o Contact dermatitis
o Nummular dermatitis
o Atopic dermatitis
o Psoriasis
o "id" reaction
171-All of the following are true atopic dermatitis EXCEPT:-
o Pruritis
o Chronicity
o 2% of patients have Ig E
o +ve family history
172-All are suggesting of atopic dermatitis EXCEPT:-
o Can be localized on cheeks
o Very itchy
 Depigmentation
o >2 months age
173-The uncommon manifestation of atopic dermatitis is
o Eczema herpeticum
o Infection
 Chicken pox
174-Child atopic dermatitis what %for having cataract capacity
o 1-5%
o 10-15% (5-10%)
o 20-30%

66
o 70-80%
175-One of the following is a major criteria of atopy:-
o Pruritis
o White dermaographism
o Icthyosis
o Pityriasis alba
176-Infant with atopic dermatitis , what do you suspect to see in patient :
o Flexor lichinification
o Erythematous lesion with crust
o Cataract
o Decrease IgG
177-In atopic dermatitis, all are true EXCEPT:-
o White dermographism is present
o 30-40% of the patient have icthyosis vulgaris
o There is an increase susceptibility to staphylococcal infections
o The ige is elevated in 20% of the patients
o There is predisposition to generalized herpes simplex infections
178-Regarding Seborrehic dermatitis all true EXCEPT:-
o Blepharitis is common
 Incidence is equal in both male & female
o Pityrisporum ovale is the causative organism
o Seborrehic dermatitis of scalp is the most common forms (Pityriasis Sicca)
179-A 3 months infant presented with erythematous rash with well-defined border covered with greasy
scales in groins & the folds were not involved. Diagnosis is:-
o Psoriasis
o Atopic dermatitis
 Seborrehic dermatitis
o Candidiasis
180-The histologic feature most helpful in distinguishing Seborrehic dermatitis from psoriasis is
o Hyperkeratosis
o Parakeratosis
o Acanthosis
o Inflammatory infiltrate
o Spongiosis
181-Coin sized subacute eczematous plaques with thin scale and minimal exudation usually few in
number, occurring primarily on the extremities in older patient with no history of atopy:-
 Cutaneous T cell lymphoma
o Nummular eczema
o Atopic eczematous dermatitis
o Tinea corporis
o Stasis dermatitis
182-House wife dermatitis can exaggerated by all EXCEPT:-
o Water
o Soap
o Gloves
 Ingestion of rice
183-Hair-dye contact dermatitis is caused by
o Para-phenylenediamine

67
184-Mason (‫ )عامل بناء‬with contact dermatitis both hands with remission when he gives in holiday , what
most probably cause :
o Neikle
o Cement (potassium dichromate)
o Cobalt
185-Cement dermatitis is most commonly caused by:-
o Potassium dichromate
o Nickel
o Cobalt
o Arsenic
186-Contact dermatitis in photographer most commonly due to:-
o Potassium dichromate
o Formaldehydes
o Arsenic
o Gold
o Copper
187-Typical sensitizing agents in contact dermatitis include all EXCEPT
o Lanoline
o Rubber
o Ethanol
o Colophony
188-Perioral dermatitis due to chewing gum caused by:-
o Citric acid
o Lennon oil
o Argenum
o Cinnamon oil
189-The differential diagnosis of perioral dermatitis:
o Rosacea
o Contact dermatitis
o Acne vulgaris
o Seborrehic dermatitis
o All of the above
190-Perioral dermatitis:-
o Primarily affects ♂ between the ages of 16 and 45
o Primarily affects ♀ between the ages of 16 and 45
o Is definitely caused by ultraviolet light
o Is definitely caused by fusiform bacteria
o None of the above
191-Facial foundations suitable for extremely long wearing and for camouflaging should be
o Water – based
o Oil- based
o Oil- free
o Water- free
o Any of the above
192-Foundation makeup without sunscreen provides SPF around
o <2
 3 to 4
o 8 to 10
o 10 to 15
o > 15
193-The most important sensitizers in lipsticks are

68
o Perfumes
o Antioxidants
o Insoluble colors
o Mordants
o Fluorescein stains
194-Detergent dermatitis of the hands
o Is mostly irritant dermatitis
o Is mostly allergic dermatitis
o Can be easily diagnosed with patch tests
o Is usually due to enzyme additives
o Is usually due to perfume additives
195-Both allergic contact dermatitis and contact urticaria may be caused by:-
o Parsley
o Indian bean
o Onion
o Tomato
 Garlic
196-Gray pigmentation of the skin of the face and neck in middle-aged women is most likely due to
o Silver
o Gold
o Mercury
o Bismuth
o Lead
197-The pigment in talon noir (black heel) is
o Melanin
o Lipofucin
o Silver
o Shoe leather dye
o Hemoglobin
198-Elastic parts of underwear are rendered allergic through the reaction with:-
o Detergents
o Oily lubricants
 Laundry bleaching
199-The cause of blister beetle dermatitis is
o Beetle sting
o Beetle bite
o Beetle feces
o Retained beetle parts
o Beetle crushes against the skin
200-Papular dermatitis treatment
o Permethrin
o Ivermectin
o NSAID
o Antihistamine
201-Most common topical drug that causes allergic dermatitis is
o Neomycin
o Chloromaephencol
o Penicillin

202-The usual site of patch test is:-


o Face

69
o Back
o Arm
o Thigh
203-Photosensitive Tatto is due to
o Cadmium sulfide
o Ferric oxide
o Mercury
o Chromium oxide
204-The differential diagnosis of flexural dermatitis in infancy includes all EXCEPT
o Seborrehic dermatitis
o Lettere-Siwe disease
o Asteotic eczema
o Wiskott-Aldrich syndrome
o Miliaria
205-What is true about Wischott-Aldrich syndrome:-
o Chronic dermatitis resembling atopic dermatitis
o Thrombocytopenic Purpura
o Recurrent infections
o All of the above
206-Chronic dermatitis with thrombocytopenia plus recurrent infection occur in
o Wiskott Aldrich syndrome
o Bazex syndrome
o Xeroderma pigmentosa
207-To differentiate between hyper ige syndrome and atopic eczema:
o By clinical picture.
o Increase IgE above 5000 iu ,in hyper IgE.
 Presence of staph lung abscess & skin abscesses in hyper ige.

866TTT of nodular prurigo


o PUVA
o ….......photo
o …..........photo
Antihistamine

807-In relation to fixed drugs eruption which is TRUE:-


o Is cell mediated immunity
o Is type in hyper sensitivity
o Toxic effect of drug
900-Langerhans cells
o Play a role in contact dermatitis
o Reduces of the number in psoriasis
o Disrupted function by ultraviolet
 All of the above
909-Hepatic pruritis best treatment:
o Topical steroids
o Antihistaminics
 Opiode antagonist

950-Hepatic pruritis best treatment:


o Topical steroids

70
o Antihistaminics
 Opiode antagonist

962-Reaction type of A.C.D


o 1
o 2
o 3
 4

791-Food can cause both allergic contact dermatitis and contact urticaria may becaused by:
o Preseles
o Onion
o L imon
 Tomato

974-Skin erruptive of the penis the shaft only


o Fixed drug eruption
 Contact dermatitis
o Allergic dermatitis

1005-Prevalence of ACD
o Balsam of peru
 Nickel
o Neomycin
o Bacitracin

1075-Which one of the following component causes contact dermatitis in children ?


o Citricacid
o Cinnamon
 Poison ivy
o Name of very strange tree which is very unrecognizable

1079-Tacrolimos the best for


 Atopic
o Eczema
o Acne
o Rosacea
1109-Patient went for a holiday on the beach and stayed six months and contact dermatitis infection what
is the reason
o Nickel
o Acid alheirovlorett
 Compounds and sun screens sensitizer paba para amonicbenzoic acid

1114-After using balsam Peru cause allergic contact dermatitis what you advice pt to avoid :

71
o Chemical dyes
o Painted metals
o Rubber
 Fragrances??
1115-Pt with proven allergy to balsam of peru should avoid:
 Perfumes
o Metal plates
o Rubbery shoes
o Water proof glue

1163-About the atopy all true EXCEPT"


 Associated lens opacity 20-30%

1164-All Characteristics by atopy except


o Netherton syndrome
o Job syndrome
o Ataxia telegentacsia
 Chediak hegashi syndrome

1179-After systemic intake of sulphonamide pt developed acute eczematoid reaction in site of previously
allergic due to contact with which substance:
o Neomycin
 Paraphenyldiamine .

1236-Young patient with papulovesical lesion on nape of neck and dorsum of hands appears after sun
exposure during summer and resolve after it:
o Actinic prorigo
 Polymorphic light eruption.
o Phytophotodermatitis
o Actinic reticuloid

1261-45 years old patient with itchy erythematous popular lesions on face, V area of chest for 3 years,
severe in summer, improve in winter test to diagnos: (photodermatitis)
 Photo patch test

1269-Most common cause of systemic photosensitivity


 Sulphonyl urea
o Sulphone
o Dyphenylhydramin

1295-The best ttt for child with atopic dermatitis is


Topical tacrolmius
1432-Wheel is the characteristic lesion in urticaria
o True
 False

72
1433-Wheel occurs only in urticaria
 True
o False
1434-Lichenification is a feature of chronic contact dermatitis
o True
o False
1436- Infantile atopic dermatitis is mostly in the nape
o True
o False
1437-In acute dermatitis there is a Spongiosis
o True
o False
1438-Adult atopic dermatitis is mostly in the extensor surfaces
o True
o False
1439-Chronicity & family history are minor criteria in atopic dermatitis
o True
o False
1440-Chronic eczema is characterized by oozing vesiculation
o True
o False
1441-In atopic dermatitis, there is in cellular immunity
o True
o False
1479-RAST is the best test for allergic dermatitis
o True
o False
1485-Pruritis is almost always due to allergy
o True
o False
1486-In eczema systemic steroids are always can be used
o True
o False

1539-Which test done to diagnosis of drug eruption:-skin test. While Allergy Patch test

1629-The usual site to detecting minimal erythematous lesion:- Abdomen.

From the questions in page 78 :

46-Atopic dermatitis: pruritus (the cardinal manifestation)


47-Allergic contact dematits: Nickel (most common agent in +)
48-Allergic contact dermatitis: Rubber (common affect feet)
49-Allergic contact dermatitis: Neomycin (most common antibiotic)

73
50-Allergic contact dermatitis: Plants (most common agent in children)
51-Allergic contact dermatitis: Cinnamon oil(most common agent in chewing gum)
52-Allergic contact dermatitis: Phenylene diamine (most common agent in color developer, hair dye)
53-Allergic contact dermatitis: Painters, cementmprick,shoe makers (most common affected)
54-Allergic contact dermatitis: Prick test-contact urticaria within 15-20 minutes (if possible)
55-Allergic contact dermatitis: ttt sensitizers e.g. Neomycin (most common agent in leg ulcer)

From the questions in page 80 :

133-EM (erythema multiform):H.S, orf, suiphonamides (major causes)


134-EM: deposits around superficial blood vessels
164-Xerosis: the most common geriatric dermatosis
165-Xerosis: the most common cause of pruritus

From the questions in page 82 :

36) Most common underlying disease in kaposis varicelliform eruption - atopic dermatitis
37) Most common site of adult atopic dermatitis- ante cubital fossa
38) Most common site of pediatric atopic dermatitis- cheek
39) Most common cause of cumulative Irritant contact dermatitis- detergents, Wet work
40) Most commonest cause of Allergic contact dermatitis-nickel
41) Most common cause of air borne contact dermatitis- parthenium
42) DOC for air borne contact dermatitis - azathioprine

74
Erythema
842- 6 y child erythema and scaling with p.p hyperkeratosis and interphalangeal hyperkeratotic
papules(PRP) the best treatment
o Narrow band UVB
 Retinoids
o MTX
842- Treatment of recurrent Erythema multiform
o Thalidomid
o Dapson
o Anti-HSV
842- Erythema multiform mostly commonly associated with:-
o HSV
o HCV
o EBV
o HPV
842- Recognized causes of erythema multiform include all EXCEPT
o Herpes simplex infection
o Mycoplasma pneumonia
o Sulphonamides therapy
o SLE
o Chronic liver disease
842- Transepidermal elimination which investigate not needed
o Blood glucose
o Creatinin
o Cortisone
o HIV

854-The mainstay therapy for necrolytic migratory erythema is:


o Chemotherapy +
o Pulse steroids
o Excision of the pancreatic tumor
o Nutritional supplements
o Electron beam
855-Multiple annular multicnetric lesion on the back, the lesion was migrating by 12cm/day best
treatment (NME)
o Removal of underlying pancreatic cancer
o MTX
o PUVA

891-Erythrema similar to wood groins


 Erythema gyratum repens

895-What is not located in the erythema nodosum


 Soreness
o Joint pain
o Conjunctivitis

75
953-Which of the following is not a cutaneous marker of internal malignancy?
o Bullous pemphigoid
o Acanthosis nigricans
o Dermatomyositis
 Erythema chronicum migrans

955-Multiple annular multicnetric lesion on the back, the lesion was migrating by 12cm/day best
treatment (NME)
 Removal of underlying pancreatic cancer
o MTX
o PUVA
o Topical corticosteroid
956-Erythematous lesion a mild scaling growing more than 7mm/d (Erythema gyratum repens)
 Advice to search for malignancy
o Borrelia bord graferi
o Top corticosteroids only
o Antihistone
956-Which of the following associated with internal malignancy ?
 Erythema gyratum repens
o Toxice pidermal necrolysis
o Erythema elevatum dinetum
957-Erytherma polycyclic large maculae, a central clearing, the edge increase many mm daily what is the
treatment (Erythema gyratum repens)
o Steriod
o Antibiotic
 Search for associated malignancy

980-Treatment of erythema nodosum leprosum


o Prdnisone 40-60 mg
 Thalidomamide100-400 mg
o Clofazimine 200 mg-d

1010-Flushing is caused by all of the following EXCEPT :


o Carcinoid syndrome
 Glucagonoma syndrome
o Mastocytosis
o Alcohol
o Menopause

1027-Annular erytherma of RHF


 Occurs in corps

1098-The lesions of erythema nodosum are most commonly located on the:


o Arms
o Face
 Lower extremities
FRUITS IN MEDICINE
76
Strawberry tongue- kawasaki disease,scarlet fever,tss
Strawberry gums- wegener's granulomatosis
Strawberry hemangioma- capillary hemangioma
Strawberry vagina in trichomoniasis.
Strawberry skin n nasal mucosa- sarcoidosis
Strawberry nasal mass- rhinosporidiosis
Strawberry shaped skull - edward syn
Strawberry gallbladder -cholesterolosis
STRAWBERRY HAMENGIOMAS IN SKIN
STRAWBERY VAGINA-TRICHOMONIASIS
STRAWBERRY RECTOSIGMOID -INFECTION OF SPIROCHETE
STRAWBERRY SKIN- sarcoidosis
Strawberry sigmoid-b.vincenti
Strawberry nevus - capillary haemanigoma.
BLUEBERRY: Rubella, CMV
(congenital disease)
MULBERRY: Tuberous sclerosis
(mulberry tumor), syphilis
(mulberry molar,neonatal finding)
RASPBERRY TUMOR: Umbilical
adenoma
CRANBERRY JUICE - prevents UTI
Berry aneurysms - saccular aneurysm of cerebral artery.
Berry ligament- lateral thyrohyoid ligament.
Blueberrymuffin baby - congential CMV.
INFECTION,toxoplasmosis,rubella
Bunch of grapes sign- hydatiform mole.
Mulberry calculus- urinary calculus.
Mulberry spot- abdmonial eruption in typhus fever.
Mulberry molar - congenital syphilis.
Mulberry hypertrophy - Rhinitis medicamentosa.
Apple peel appearance : Intestinal
atresia
Apple jelly nodules: Lupus vulgaris
Apple core deformity- in
carcinoma colon (double contrast
barium enema finding)
Apple peal appearance- ileal atresia
Apple sauce appearance- meconeum illeus
Apple core sign - synovial chondramatosis of femur
Peau'd orange skin:Breast cancer
�Lemon on stick appearance :
cushing's syndrome

77
Banana nd � lemon sign.. Both r usg finding of arnold chiari type 2 malformations (sp
a/s with sipna bifida) lemon sign due to indentation in frontal bone. Banana fr contour of
cerebellum
Banana fracture- paget disease.
�Pear shaped bladder-pelvic haematoma, bilateral
lymphocoele.
Cherry red spot - cherry red spot myoclonus syndrome.
Cherry angioma - senile haemanigoma.
CHERRY RED SPOTS-central retinal artery occlusion,berlins oedema,tay
sachs,niemanns picks, gauchers

78
GenoDermatoses & Ichthyosis
AD AR XLR XLD
1. Neurofibromatosis 7. Xeroderma pigmentosum 12. X-linked 18. Incontinenti
2. Ichthyosis vulgaris 8. Pseudoxanthoma ichthyosis a pigmenti
3. PPK 9. Phenylketonuria 13. Dyskeratosis 19. Focal
4. Darier's disease 10. NBIE congenital dermal
5. C1 esterase angioedema 11. Acrodermatitis 14. Ehlers-Danlos hypoplasia
6. Peutz Jaghers enteropathica syndrome 20. CHILD
15. Weskot-Aldrich's syndrome
syndrome
16. Chronic
granulomatous disease
17. Fabry disease
It is currently used for clinical diagnosis of Marfan syndrome.
It considers :
a) Family history
b) Cardinal clinical signs in the absence of family history
c) Presence or absence of Fibrillin mutation

64-Which of the following criteria will usually help differentiate autosomal dominant and recessive
inheritance all except
o Frequency of phenotypic involvement of parents
o Consanguinity
o Frequency of involvement of male vs females
o Presence or absence of the phenotype in succeeding generations
o The severity of the disorder
65-Each of the following is transmitted by autosomal dominant inheritance EXCEPT:
o Gardner’s syndrome
o Cronkhite-Canada syndrome
o Peutz-Jeghers syndrome
o Cowden’s disease
o Muir-Torre syndrome
66-Each of the following is transmitted by autosomal dominant inheritance EXCEPT :
o Gardner’s syndrome
o Cronkhite-Canada syndrome
o Peutz-Jeghers syndrome
o Cowden’s disease
o Muir-Torre syndrome
67-All of the following are A.D EXCEPT:-
o Peutz-Jergers syndrome
o Xeroderma pigmentosa
o Darier's disease
o Neurofibromatosis
68-The following are autosomal dominant EXCEPT:
o Neurofibromatosis
o Xeroderma pigmentosa

79
o Ataxia
o Pentz – jeugers.

167-What is the following disease has genetic susceptibility (Hereditary disease)


o EM
o EN
o E Margentium
o Eczema

730-Neurofibromatosis , type 1 what is WRONG


o Start at puberty
o Defect in neurofibromin
o Highly vascular
731-Malignant degeneration of Neurofibroma in von Recklinghausen neurofibromatosis is
o Unreported
o 2%-5%
o About 25%
o About 50%
o More than 80%
732-A child presented by café au lait patches, all should be done except:-
o Complete physical examination
o Skull x:-ray
o Fundus examination
 Examination of the fundus of the mother & the father
733-The most common manifestation of tuberous sclerosis
o Cardiac arrhythmias 50%
o Mental retardation 60-70%
 Seizures 80-90%
o Enamel pitting
734-Basal cell nevus syndrome includes all of the following EXCEPT
o Multiple BCC in at least 50% of patients
o Multiple jaw cysts
o Palmoplantar pits
o CNS anomalies
o Follicular atrophoderma on extremities
735-Marfan’s syndrome is characterized by striking lack of which of the following components of the
connective tissue ?
o Collagen
o Elastin
o Fibrillin
o Collagenase
736-All of the following is present Sjogren Larsson syndrome EXCEPT:-
o Congenital icthyosis
o Mental retardation
o Spastic diplegia
o Anterior Congenital cataract
737-Patient have multiple pigmented macules in lip, palate, tongue with no other anomalies the diagnosis
o Peutz-jeghers syndrome
o Melasma

80
o Hemochromatosis
o Addison disease
738-Difference between number of hair follicles in newborn& adultlife
o Number increase in adultlife
o Number decrease in adultlife
 No difference in number in newborn , adultlife
739-Features of Preus syndrome include all of the following EXCEPT
o Generalized hypopigmentation
o Hypochromic anemia
o Growth retardation
 Lichenoid skin eruption
o High-arched palate
740-The characteristic histologic feature of epidermolytic hyperkeratosis is :
o Hyperkeratosis
o Acantholysis
o A subepidermal bulla
o A spongiform pustule
o A reticular pattern of vacuolization of the epidermis
741-Muirr –tori syndrome NOT develop with:
o Keratoacanthoma
 Adenoma sebaceum
o Adenomatous carcinoma
o Sebaceous carcinoma
o Sebaceous epithelioma
742-Histopathology of porokeratosis shows
o Coronoid lamille
743-Drug used in treatment of pediclosis
 septra
 rifampicin, doxycycline,azithromycin, erythromycin
o amoxycillin
744-Netherton syndrome defect
 Serine protease
o Conexins
o Spink 5 gone
745-Investigation of Addison syndrome
 Cortisol level
o ACTH level
746-Ichthyosis linears circumflexa, trichorrhexis invaginata and atopy are characteristic of
o Refsum syndrome
o Dorfma-chanarin syndrome
o Netherton syndrome
o Rowel syndrome
747-Kawasaki syndrome is manifested by:-
o Cervical lymphadenopathy
o Skin desquamation
o Strawberry tongue
o All of the above

81
748-Child suffered from fever then peripheral peeling strawberry tongue best ttt (kawasaki)
o Erthromycin
o Emollient
o IV Ig
o Assurance
749-About Bloom's syndrome what is WRONG:-
o Telangectatic Erythema on butterfly area
o Dwarfism
o Photosensitivity
 Autosomal dominant
o None of the above
750-Which of the following disorders of pigmentation is thought to result from mosaicism?
o Piebaldism
o Vitiligo
o Hypomelanosis of Ito
o Waardenburg syndrome
o Woof syndrome
751-Cutaneous manifestations of Darier's disease may be exacerbated by:-
o Dapsone
o Beta blockers
o Lithium
o Thiazides
o Antimalarial
752-Axillary slight erythematous papules since 7 years H/P hyperkeratotic and dilated hair follicle best
treatment(Fox–Fordyce disease)
o Retinoid
o Antibiotic
o Tacrolimus
o Emollient
753-Dirty warty papules on the neck of male offensive edema +subrabasal cleft vesicles best treatment
o Isotretinoin
o Emollient
o Cytotoxic
o Antifungal
754-Skin colored or slightly pigmented follicular papule in both axillae. H/P keratin plug in follicular
infudibulum best treatment
o Dapson
o Antihistamin
o Topical retinoid
o Topical Vit D3
755-A 30-years-old patient presents with greasy, keratotic, yellowish-brown papules and plaques over the
retroauricular area, scalp, forehead, front of chest, and midline on back. Large vegetating foul-smelling
crusted plaques are present on axilla . Small punctuate keratotic pits are present on the palms and soles .
Biopsy from plaque reveals suprabasal cleft , acantholytic cells, and focal dyskeratosis . What is the
treatment of choice ? (Darier)
o Systemic corticosteroids
o Cyclophosphamide
o Acitretin
o Thalidomide
756-Acanthosis nigricans is commonly associated with:-
o Internal malignancy

82
o Diabetes
o Obesity
o Insulin resistant diabetes
 All of above
757-Commonly acanthosis nigrigans is associated with:
o Diabetes insipidus
o Insulin resistance
o Adenocarcinoma
758-Female patient have Pseudoxanthoma Elasticum and she is pregnant what do u suspect she will not
suffer from:
o Abortion
o Cardio vascular system problems
o Early striae
 Sudden blindness
759-Pregnant female suffers from psudoxanthoma elasticum what is true
 Highly liability to stria occurrence
o Abortion
760-Pseudoxanthoma elasticum-like illness is reported to be associated with all of the following EXCEPT
o Penicillamine intake
 Vitamin A administration
o Use of nitrate fertilizers
o Multiparity
761-Female patient have pseudoxanthoma elasticum. And she is pregnant what do u suspect she will not
suffer from:
o Abortion
o Cardio vascular system problems
o Early striae
 Sudden blindness
762-Which of the following diseases may cause elastic fibers pathology in dermis
o Darier disease
o Pseudoxanthoma elasticum
763-Pregnant female no straie in
o Psudo xanthoma elasticum
o Cutis laxa
o Marfan Syndrome
 Ehlers danlos
764-Inheritant of Ectodermal dysplasia
o AD
o AR
o XL-D
o XL-R
765-Inheritant of Tuberous Sclerosis
o AR
 AD
o Sporadic
o XL
766-The differential diagnosis of acrokeratosis verruciforms include all EXCEPT
o Dariers disease
o Plane warts
o Seborrehic keratosis

83
o Epidermodysplasia verruciforms
 Subcorneal pustular dermatosis
767-The most common presentation of lipoid protenosis is:-
o High blood lipids
o Convulsions
o Hoarseness of voice
o Albuminurea
768-Treatment of nevus sebaceous is:-
o Electrocautery
o Total excision before puberty
o Total excision after puberty
770-Steroid sulfatase enzyme deficiency is present in:-
o Icthyosis vulgaris
o Epidymolytic hyperkeratosis
o Lamellar icthyosis
o X-linked recessive icthyosis
771-New born girl vesiculobullous linear tense bullae on arm incontinentia pigmentia
o XLR (X-linked recessive )
o XLD (X-linked dominant )
o AD (autosomal dominant)
o AR(autosomal recessive)
772-Hypomelanosis of Ito
o Is a variant of incontinentia pigmenti
o Is inherited as XLD trait
o Occurs much more frequently in males than females
o Can be associated in 3/4 of patients with skeletal abnormalities, mental retardation and ocular
abnormalities
o Is best treated by oral retinoids
773-The following statements about incontinentia pigment (IP)is /are correct all EXCEPT
o It affects males more than females
o Cutaneous lesions occur in 100% of patients and appear at birth or shortly after it
o Leucocytosis and peripheral eosinophilia are not uncommon
o Ocular anomalies are among the most severe systemic anomalies and can lead to blindness
o Anodontia or late dentition are not uncommon

883-Which of the following diseases may cause elastic fibers pathology in eps(elastosis perforans
serpiginosa)
o Darier disease
 Pseudoxanthoma elasticum

938-Pregnant patient will not have striae at all in:


o Elastosis perforans serpiginosa
 Ehlers danlos
o Marfan syndrome
o Cutislaxa

84
951-Child 4 months with bruises over face, arms, limbs what is the diagnosis:
 Wiskot Aldrich syndrome
o Down syndrome
o Phenylketonuria
o Peutz jagger

970-The following disaeses assocaited with endocrinal abnormalitis EXPECT


o Acanthosis negricans
o Viltigo
o Neurofibroma
 Sturge weber syndrome

986-Which of these syndromes don't present with atopic dermatitis like lesions-
o Hyper ige
o Wiskot Aldrich
 Chediak Heggashi
o Ataxia telangiectasia

1007-Hansen syndrome is best investigated by:


 Skin test
o Kveim test
o Histamine test
o Metcholin test
o Lepramin test

1038-Sjogren larsson syndrome


o AR
o ischthysoform syndrome
o no cardiopathy

1053-Papillon-lefevre and haim-munk syndromes have which of the following symptoms?


o Right-ventricular cardiomyopathy
o Pseudoainhum
o Eccrine syringo fibradenoma
 Periodontitis with tooth loss

1058-HLA in Behcet disease


 HLA-B 51

1060-Birt-hogg-dube syndrome is most strongly associated with which of the following malignancies?
o Basal cell carcinoma
 Renal cell carcinoma
o Trichoepithelial carcinoma

85
o Eccrine syringo fibroadenoma

Digeorge syndrome
 Thymic aplasia(thymus)

11230-Male patient with history of depression has rounded and oval pupuric and ecchymotic patches over
his thighs, what investigations needed help to diagnose the case :( Gardener diamond syndrom)
o Platlet time
o Bleeding and clotting time
 Autologous intradermal injection of RBC's

1135-Pseudoxanthoma elasticum all true except


 Gene defect chromosome 16 p 3
o Peau d, orange most common retinal finding
o Doppler blood pressure to investigate intermittent claudication
o Similar clinical picture by D penicillamine treatment

1136-Pseudoxanthoma elasticum-yellowish papules on neck h/p defect in elastic fibers is reported to be


associated with all of the following EXCEPT :
o Penicillamine intake
o Vitamin A administration
o Peadu orang retinal affection
o Multiparity
 Defect in the 16q3 gen

1162-Linear verrucous lesion on LL :


o Incontinentia pigmenti
o Ichthyosis hystrix
 ILVEN (inflammatory linear verrucous epidermal nevus)

1165-3y old with Magnolian spot best ttt


o Co2 laser
o Qswitched ruby
o Qswitched nd yag
o IPL
 Counselling

1193-Erythema nodusm – sarcoidosis - hilar lymphadenopathy what is the diadgnosis


 Loffegren syndrome

1195-Circular depression over rim of helix, linear earlobe creases, facial vascular malformation,
macroglossia
 Beckwith–Wiedemann syndrome

86
1198-Vogt-Koyanagi-Harada Disease:
 Retinal detachment is the first association

1228-Anhidrosis compensated with hyperhidrosis, Tonic pupil, absent tendon reflexes :


 Ross syndrome

1248-Male with Wilson disease taking penicillamine for 2 years developed hyperkeratotic erythematous
follicular pulgging lesion on back. What is true:
 Transepidermal elimination of collagen
o Transepidermal elimination of elastin
o Transepidermal elimination of keratin

1257-AR disorder with photosensitivity, retinitis, pigmentation, mental retardation, DNA instability:
Cockayne syndrome

1262-Birt–Hogg–Dubé syndrome associated with:


 Renal cell carcinoma
Fibrofolliculomas

1264-A case of decreased lacrimation with increased sweating and generalized pruritus .Diagnosis:
 Familial dysautonomia
o Sensory neural defect

1227-Newborn baby presented with eczematous rash, recurrent abscess on the face, arm and groin.
Pustule on areas of the body with a cold Abscess and bronchitis, IgE serum > 3000 units, low platelet
level, high eosinophilic. 1000%
 Wiskott-Aldrich syndrome
o Job syndrome
o Netherton syndrome
o All of the above
o None of the above

1294-Child 2 months, papules and plaques heal by milia & scarring, nail dystrophy,erosion on palat.
What is the pattern of staining:( Dominant generalised DEB)
 Collagen VII at base
o Bp 230, collagenVII at roof
o Bp 230 at roof ,anti k14, collagen VII at floor
o Bp 230,anti k14, at roof collagen VII at floor

1299-AR disease, with mental deficiency , mutation in gene lead to DNA killer
o Rothmund-Thomson syndrome
 Bloom syndrome

87
o Ataxia telangiectasia

1314-10 year old child, horn with fine scales, shiny skin, fatty liverand skletal muscle affection,
diagnosis:
o Nerthon syndrome
o Sjogren larsson syndrome
 Dorfman-chanarin syndrome
o Refsum syndrome

1377-Picture of hyperpigmentation in all the body , palm crease , hypopigmentation and weakness, the
diagnosis:
 Addison

1351-adult patient with connective tissue navus (shagreen patch) with history of counvulsion since
childhood, what is the diagnosis:
 Tuberous sclerosis
 AD

1352-Fibrofolliculomas, trichodiscomas & acrocondrons are characteristic of


o Vorner syndrome
o Kindler syndrome
o Kasabach-Meritt syndrome
o Louis-Bar syndrome
o Birt-Hogg Dube syndrome

88
1389-Incotenia pigmenti
 XLD

1543-Ichthyosis vulgaris (AD) defect in:- filaggrin

1493-Ramsey-Hunt syndrome affects facial nerve, auditory nerve, but not trigeminal
o True
False

1563-Porokeratosis Histopathology:- coronoid lamellae

1600-Characteristic features of jop syndrome do not include:- pseudomonas abscess.


1632-Wiscott-Aldrich syndrome + chronic granulomatous disease is XLR while hyper ige syndrome AD
1633-Hereditary angioedema all false except:- AD

From the questions in page 79 :

59-Neurofibromatosis:6 CAM 5mm, prepubertal, major criterion

From the questions in page 80:

163-Ichthyosis; X-linked: ↓↓steroid sulphatase


164-Xerosis: the most common geriatric dermatosis
165-Xerosis: the most common cause of pruritus
166-Heriditary-Haemorrhagic telangiectasis: autosomal dominant AD
172-Howel-Evans syndrome: Palmoplantar keratoderma+ cancer esophagus
173-Erythroderma: lymphoma-leukemia (25%of cases)

89
Glands Diseases
45-Sebaceous gland activity and vaginal discharge in infancy usually is due to
o Maternal sex hormones
o Adrenal hyperplasia
o Maternal diabetes mellitus
o Maternal prenatal vitamin therapy
o None of the above
55-The physiologic function of human sebum is
o A barrier layer
o A natural antiseptic
o A buffer
o A water retainer
o Undetermined
56-Eccrine gland developed intrauterine during
o 2th month intrauterine
o 4th month intrauterine4-6
o 6th month intrauterine
57-Development of eccrine glands in sole (palm & planter)
o 2-3 months
o 4-5 months
o 6-7 months
o 7-8 months
58-Normal human eccrine function is
o Sympathetic adrenergic
o Sympathetic cholinergic
o Parasympathetic adrenergic
o Parasympathetic cholinergic
o Androgen-dependent
59-Apocrine glands all EXCEPT
o Develop from follicular epithelial cells
o Are located in the axillae, perianal region and areole
o Secret an oily malodorous material
o Have no well-determined function
o Are influenced by sex hormones
60-Stain of epithelial of eccrine hidrocystoma
o S100 eccrine hidrocystoma (langerhans cells, lands, schwann cells, chondrocytes , adipose tissue and
melanoma) (clear cell acanthoma)
o PASS (for granules)
o Zehl nelson
o Tolidin blue
61-Apocrine hidrocystoma best investigation diagnostic
o PAS
o S100 ptn
o Toluidine blue

90
652-All of the following are implicated in the pathogenic of Rosacea except :
 Increased sebum
o Dermal tissue changes
o Helicopacter pylori
o Vasculature hyperactivity

683-The primary pathogenic event in milliaria is


o Bacterial infection
o Ductal leakage
o Ductal hypercornification
o Portal occlusion
o Increased sebum production
684-A man with skin coloured papules and papulovesicles on central face he noticed seasonal variation in
which lesions increase in size during summer months on biopsy a clear fluid was seen diagnoosis is
o Miliaria rubra
o Trichoepithelioma
 Hydrocystoma(cystadenoma)
685-Atrophy of the sweat glands is well known in:-
o Systemic sclerosis
o Systemic lupus Erythematosis
o Siogren's syndrome
o All of the above
o None of the above
686-The areas of hyperhidrosis in the following condition may be explained by sensory or sympathetic
dermatomal
o Hyperhidrosis associated with autonomic dysre…… flexia
o Hyperhidrosis sensory to orthostatic hypotenia
o Hyperhidrosis due to post-traumatic syringemyelia
 All of above
o None of above

901-Kyrle infects
o Grape fruit
 Perifollicle
o Terminal tube
o All of the above

1444-Unilateral hyperhidrosis on face & neck you must see intra-thoracic.


o True
o False

1461-Hidardentis suppurativa is the disease of apocrine gland


o True
o False

1480-Hidardentis suppurativa is treated by 7 days antibiotics


o True

91
o False

1578-Lichenification cause:- hypohidorisis.

1637-Fox for dyce disease is disorder in:- Eccrine gland

1638-Fox for dyce spots is disorder in:- Sebasceous gland

1442-Unilateral hyperhidrosis on face & neck you must see intra-thoracic.


o True
o False

1613-By electron microscopy, human apocrine secretion is:- apocrine and merocrine.

92
Metabolic, Granulomas , Infeltrations, Vitamines &FDE
( histiocytosis-Porphyria- Granuloma- Pheochromocytoma- Diabetes insipidus- Perforating Dermatoses-
sweet syndrome- Neutrophilic dermatosis- …etc.)

544-First line treatment of granuloma faciale is


o Oral isotretinoin
o Topical adapalene
o Oral doxycycline
o Dapsone
o Oral corticosteroids for a 5-7 days

551-Neutrophilic dermatosis include all EXCEPT


o Familial Mediterranean fever
o Behcet's disease
o Sweet syndrome
o Pyoderma gangrenosum
o Carcinoid syndrome

552-Pathergy reaction occurs in the following:-


o Behcet's syndrome
o Pyoderma gangrenosum
o Sweet syndrome
o All of the above
553-Sweet syndrome not associated with
 PCT
o LE
o Myelocytic leukemia
o Rheumatoid arthritis
o Inflammatory bowel disease
554-There is no relation between sweat syndrome
o Pyoderma gangenrasum
o IBD
o LE
o Myloproliferative disorder
 None of the above
555-Sweet syndrome associated with
o Myeloproliferative leukemia )acute myelogenous leukemia(
o Inflammatory bowel disease
o Myelocytic leukemia
o Lymphoma
 All of the above
556-Sweet syndrome is commonly associated with:-
o Hematopoietic malignancy
o Malaria
o Dwarfism
557-Adult male he is febrile having vesiculopapular lesion on left malar area with cervical LN what is the
best treatment: sweet syndrome

93
o Steroid
o Anti histaminics drugs

562-Pyoderma gangrenosum is commonly associated with:-


o Cardiovascular diseases
o Inflammatory bowel diseases
o Sarcoidosis
o Internal malignancy

563-All the following are types of pyoderma gangrenosum except one:


o Bullous form
o Pustular form
o Ulcerative form
o Psoriasis form
o Vegetative
564-Inflammatory bowel disease can be associated with the following EXCEPT
o Vitiligo
o Psoriasis
o Pemphigus foliaceous
o Arthritis
o EBA

573-Comments organ affected in Langerhans's cell histiocytosis is


o Skin
o Lung
o Skelton (Bone)
o L.N.
574-Langerhans Cells histiocytosis most complication
o Skeletal
o Skin
o Blood
o Lung
575-Diabetes insipidus is a characteristic feature of
o Eosinophilic granuloma
o Hand-Schuller-Christian disease(multifocal histocytosis)
o Urticaria pigmentosa
o Wegener granuloma
o Lymphatoid granulomatosis

774-Histopathological differential diagnosis of granuloma annulare is:-


o Sarcoidosis
o Necrobiosis lipodica
o Behcet's disease
o Pyoderma gangrenosum
775- 21 year old nurse has plaques on both of her hands. The histopathology of the lesions showed
palisaded granuloma with histiocytes and clear mucin in the centre that is stained with alcian
blue.diagnosis is
o Granuloma annulare
776-Necrobiotic palisading granuloma with mucin
o Necrobiosis lipodica

94
o Granuloma annulare
o Rheumatoid nodules
o Sarcoidosis
777-How many percentage of diabetics with necrobiosis lipoidica diabeticorum
o 0-3%
o 1%
o 5-10%
o 50-70
778-Male to female ratio in Necrobiosis lipodica:-
o 10: 1
o 1: 10
o 1: 1
o 1: 3
o 3: 1
779-Diabetic dermopathy characteristically occurs on the:-
o Scalp
o Neck
o Axilla
o Shin
o Sole
780-Sarcoidosis in black patients' mostely
o SC nodules
o Erythema nodosum
o Scar
 Maculopapular
781-Serum angiotensin 1 converting enzyme ACE is typically elevated is:-
o Atopic dermatitis
o Chronic urticaria
o Pemphigus vulgaris
o Sarcoidosis
o Melanoma
782-Asymptomatic skin colored multiple papules on the back and proximal arm of an 8 years child and
history of cough and mild fever. What diagnosis
o Lichen nitidus
o Keratosis pilaris
o Papular Sarcoidosis
o Vit A
783-Patient Comes to the ER &his chest x ray reveals bilateral hilar ln and he gives u previous history of
red indolent lesion on the face, ear, nose and lip what ur diagnosis:
o SLE
o Sarcoidosis
784-Triad of hilar LN,arthritis,skin lesion:
o Lefogrin syndrome
o Netherton
o Herodite
785-Erythema nodosum ,bilateral hailar lymph node adenopathy
o Sjogern
o Lefogrin syndrome
o Heartoed syn
786-The most common cutaneous manifestations in amyloidosis:-
o Bullous amyloidosis

95
o Nail dystrophy
o Alopecia
o Purpuric lesions & ecchymosis
787-Amyloidosis be secondary to chronic diseases such as all of the following EXCEPT :
o Rheumatoid arthritis
o Sjogren’s syndrome
o Inflammatory bowel disease
o Hodgkin’s disease
 Generalized psoriasis
788-The most common skin sign of systemic amyloidosis is/are:
o Bullous lesions
o Alopecia
o Petechiae, purpura, and ecchymoses+
o Nail dystrophy
o Cutis laxa
789-Amyloidosis be secondary to chronic diseases such as all of the following EXCEPT :
o Rheumatoid arthritis
o Sjogren’s syndrome
o Inflammatory bowel disease
o Hodgkin’s disease
 Generalized psoriasis
790-Photosensitivity of the skin occurs in all types of porphyria EXCEPT
o PCT
o HCP
o VP
o AIP
o CEP
791-The blisters of the porphyria is formed
o Subcorneal
o Intraepidermal
o Subepidermal
o In different levels depending on the type of porphyria
o Because of cutaneous hemorrhage
792-Factors known to be provocative of variegate porphyria include EXCEPT
o Anticonvulsants
o Alcohol
o Pregnancy
 Vitamin C deficiency
o Febrile illness
793-A 10-year-old patient presents with multiple tuberous and tendinous xanthomas over the albows ,
Achilles tendon, knees, and corneal arcus. An older sibling has similar lesions . What is the MOST
probable defect ?
o Mutation in lipoprotein lipase enzyme
o Deficiency of apolipoprotein C2
o Defect in apolipoprotein A
 Defect in LDL receptor
794-Patient with nodular lesions on the elbows and knees with small eruptive yellow papules on the
thighs and buttocks(hypertriglyceridaemia). Which is the disease?
 Eruptive xanthema
o Nodular xanthema

96
o Tuberous xanthoma
o Tendinous xanthema
795-Child presenting with tendinous or tuberous xanthomas and has a brother with the same
manifestation, the defect is probably in
o Lipoprotein lipase
 LDL receptors
o Apoprotein B 100
o Apoprotein C
796- 61 year old man with yellowish ulcerating lesion on eyes. Histopathology of lesion shows
epitheloid cells with multinucleated giant cells and touton cells (necrobiotic xanthogranuloma)
o Mostly associated with Ig M monoclonal gammopathy
o High levels of cholesterol and vldl
o High level of TG
o Diabetis insipidus common complication
 C1 esterase inhibitor decreased
o Mostly associated with igg monoclonal gammopathy
797-Female with yellowish plaques and papules on bilateral medial canthus, biopsy revealed touton cell
(xanthelasma) what is TRUE
o VLDL & TGS
o LDL
 Hyperlipoprotenemia
798-Differential diagnosis of gouty tophi include:-
o Tuberous xanthoma
o Sarcoidosis
o Erythema nodosum
 All of the above
799-The most biologically active form of vitamin E is
o Alpha tocopherol
o Alpha tocopherol acetate
o Alpha tocopherol sulfate
o Beta tocopherol
o Beta tocopherol acetate
800-Male has follow scales in front of thigh with no evidence of vitamin ↓↓ best treatment
 Retinoid
o Vit A
o Vit B
o Vit C
801-Acrodermatitis entropathica
 Zinc
o Ca
o Vit C
o Fe
802-Drug cause pellagra like symptoms all EXCEPT
o 5 mercaptopurine
o INH
 Captopril
803-All of the following are drugs which may induce a pellagra like eruption EXCEPT
o Sulfonamides
o 6-mercaptopurine
o ACE inhibitors (Antihypertensive)

97
o Isoniazid
o Anticonvulsants
804-Pellagra caused by deficiency in
o Nicotinic acid Niacin (vitamin b3)
o Folic acid
o Riboflavine B12
o Thaiamine B1
805-Pellagra is due to
o Zinc deficiency
o Vit B deficiency
o Niacin deficiency
806-The mucin of myxedema is predominantly
o Hyaluronic acid
o Chrondroitin sulfate A
o Chrondroitin sulfate B
o Heparin
o None of the above

808-One of the following drugs may cause eruptive xanthoma:-


 Retinoids
o Ampicillin
o Dapsone
809-Most common clinical manifestation of drug eruption is:-
 Morbilliform eruption
o Lichenoid drug eruption
o Photo allergy
o Fixed drug eruption
o T.E.N
810-The most common form of drug eruption is:-
o Bullous
o Fixed
 Exanthematous
811-Onset of exanthematous drug eruption after taking therapy is
o 2 weeks
o More than 3 weeks
o 1 month
o 3 months
812-The only drug that cause anetoderma is:
o Delatin
o Pencillamine
o Procainamide
813-To diagnose drug reaction:
o Skin test.
o Brick test.
o Rast test.
o Increase blood oesenophil.
o None of the above.
814-Ochronosis caused by all the following drugs except
o Topical formulation containing hydroquinone
o Phenol
o Beta blockers (antihypertensive drugs)

98
o Amiodarone
815-Which of the following medications would not induce ochronosis ?
o Cream containing hydroquinone
o Antimalarials
o Antihypertensives
o Products containing resorcinol and phenol
816-Contact dermatitis on one eyelid commonly due to
o Mascara
o Eyeliner
o Eyeshadow(if both eyelids)
o Solution of contact lenses
 Nail polish
817-Dose of nifedipine in treatment of Raynaud phenomena
o 10 mg twice daily
o 20-30 daily
 30-40 daily
o 10-80 daily
818-Dose of nifedipine in scleroderma
o 10-20 mg
o 10-80 mg
o 20-40 mg
819-Warfarin toxicity (painful lesion on LL darkness bluish) treatment
o Vit k1 IV in emergency
o Heparin
o Monoclonal anti body - purified protein C concentrate
o Debridement grafting
o All true
820-In patient treated with warfarin and develop echomosis in lower limbs, the most probable cause is :
o Prolonged PTT& PT.

831-Patient with rubbery swelling in the lower lip and side of the tongue which sometimes decrease or
increase and histopathology shows some foci of granuloma what is best diagnosis:
o Melkerson rosenthal syndrome

832-Male with enlarged lips and fissured tongue all are treatment except
o Corticosteroid
o Dapsone
o Retinoid(acetritin)
o Thalidomide

847-Lesion of Kyrle's disease:-


o Follicular
o Interfollicular
o Acrosyringium

848-The most common misdiagnosis of follicular mucinosis is:-


o Sarcoidosis
o DLE
o Leprosy
o Discoid eczema

99
o MF
849-Alopecia mucinosis
o Presents a variable clinical appearance
o Usually progresses to lymphoma
o Exhibits metachromasia with Alcian blue stain
o Is more likely to be serious in children than in adults
o Is confined to the scalp
850-Which of the following statements about bromoderma is untrue
o It is common manifestation of chronic bromide poisoning
o It is unrelated to blood bromide levels
 It may appear only after discontinuation of the offending drug
o It may present multiple types of lesions
o All of the above
851-Treatment of cholastasis
o Antihistamin
o Cryotherapy
o Topical steroid
o Opioid antagonists
852-Jordans’ anomaly (lipid droplets in leuckocyte ) are found in
o Eccrine glands
o White blood cells
o Red blood cells
853-Jordan anomaliy
o Lipid droplets in eccrine colis
o Lipid droplets in granulocytes
o Elevation of muscle enzymes
o Elevation of hepatic enzymes

856-Child with asymptomatic 2mm vesicular lesions contain milky color liquid on posterolateral tongue.
The lesions are 2 x 5 mm. Best treatment is
o Counseling
o Removal of part of the tongue
o Intralesional steroids

857-The early feature of lipoid proteinosis:


o Hoarseness of voice.

861-Calciphylaxis the setting is


o Lung team dialism
o Hyperparathyroidism
o Intoxication
o Endothelial vascular lesion

875-Common site of affection histiocytosis in children


o Skin
o Skelton

889-Defention of perforating disease (Trans epithelial elimination best described as):

100
 Extrusion of dermal materials through the epidermis with minimal damage
 Several different perforating diseases have been associated with chornic renal failure including
kyrle,sdiseases
 These are a group of disease in which altered components of skin are eliminated via epidermis

914-What is the envelope protein of the cornified cells


 Loricrin
915-Langerhan's cell histioplasmosis is usually in:
o Skin
o Heartmuscle
 Bone
935-Old pt itchy myxedematous papules on trunck and extremities and alopecia and oedematous tongue
will have:
 Low level of T3 and T4
990-A 12 yr old female, non pruritic annular eruption in the left foot for 8 months, looks pale and not
scaling. Had no response to 6 wks of miconazole .
o Discoid lupus erythramatosis
o Erythema nodosum
o Tinea corporis
 Granuloma annulare.
1011-Febril patient with erythematous, painful plaque in the malar area is:
 Sweet syndrome
o Painful necrotic vasculitis
o Migratory nodular panniculitis
o Glucagonoma

1013-Patient with round brown lesion in face+ cough+ splenomegaly+ hepatomegaly+X-ray revealed
enlarged hilar lymph nodes what is the possible diagnosis?
o Myeloproliferative lymphoma
 Sarcidosis

1034-Phrynoderma due to
 Low essential fatty acids + vit A
1073-Pt. Comes to the ER & his chest x-ray reveals bilateral hilar LN and he gives u previous history of
red indolent lesion on the face, ear, nose and lip and on tatooo what ur diagnosis:
o SLE
 Sarcoidosis

1035-Pheochromocytoma
Metabolic disassociated with n.fibromatosis

1081-More types porphyria sensitivity and reaction to solar radiation


 Porphyria cutanea tarda

1117-All true about sweet syndrome EXCEPT

101
 Show enhancement if give dapsone

1132-A renal patient with purple colored papules on the chin of the tibia that started to ulcerate centrally
which is correct (Calciphylaxis)
 There is associated hyperparathyroidism
o This is mild with low mortality
o Ca deposition is seen in large vessels
o Can treated with drugs to render calcium and phosphate more soluble

1136-A female patient developed a red smooth lesion on top of a scar in the abdomen.It was excised and
histopathological examination revealed spindle cells in astoriform pattern. CD 34 was positive. The most
appropriate about this lesion is:
o CD44 helps to confirm diagnosis
o Simple excision is the treatment of choice
o Radiotherapy does not add in reducing the recurrence
o Metastasis commonly occur to regional lymph nodes

1137-Calciphylaxis the setting is:


o Lung team dialism
 Hyperparathyroidism
o Intoxication
o Endothelial vascular lesion
1138-Most common cause of calciphylaxis:
o Chronic renal failure
o Malignancy
o Hypercalcemia
o Hyperparathyriodism
1083-Person has single papule on the back of the hand without symptoms and there blistered biopsy,
showing the presence of collagen fibers with myosin and PAS test negative, what is the treatment (skin
disease focal moccin)
 Eradicate the lesion surgically
o Waiting and monitoring
o Lesion injection Balstroi

1143-Male patient with blue greenish discoloration of axilla and under clothes and dark urinee asking
which investigation not needed (Alkaptonuria):
o X-ray lumbar spine
o Urine porphyrins
o Light microscopy (also skin biopsy)
 Bendict test
1144-Pigment on ear, sclera, nose, darken diaper area. Which investigation not needed (Alkaptonuria(:
o Need for skin biopsy
o Urin analysis porphyrin
o Lumbar space X-ray
 Benedict's test
1145-Sweet syndrome associated with all the following except:
o Pregnancy

102
o Rheumatoid arthritis
o Myeloprolferative disorders
 Recurrent herpes simplex

1186-A case of porphyria characterized by hyperpigmentation and skin fragility. The causativedefective
enzyme is
 Uroporphyrinogen decarboxylase

1191-Female patient with annular lesion in the leg which is asymptomatic , histologically showing
pallisading granuloma with mucin and negative rheumatoid factor, (Granuloma annulare ) the treatment is
 Reassurance and follow up
o Steroid

1204-Describe a case of mastocytosis ( patient with macules and develop urticaria when rubbing ) which
drugs is contraindicated
Aspirin

1215-What is true about protein c:


o It inhibits protein s
 It activates va, vii- a vitamin K-dependent glycoprotein-

1246-Patient with nodular single lesion with prominent follicular pores without any systemic affection,
histopathologically showing perivascular lymphohistiocytic inflammatory infiltrate with Grenz zone
formation
o BCC
 Granuloma faciale
Sweet syndrome

1263-Which type of xanthoma not due to lipid abnormalities:


o Plane
o Tendineous
 Xanthoma disseminatum
 Diffuse plane xanthomatosis
 verruciform xanthoma
o Popular
o Verrociform

1300-Increse lipids in
 verruciform xanthoma

1311A case of amyldosis


 Electrophoresis of serum and urine shoul be done

103
1335-Patient clinical pic. And pathology (perforating dermatosis) what investigations not needed:
o Fasting blood glucos
o HIV
o Kidney function test
 Serum cholesterol

1348- Pt. Serpiginous papules cocentric 5x4 on dorsal of hand (feet) not itching. In histopathology
granulomatous tissue & mucin.
Which ttt
o Steroid
o Dapson
o Reasurance
o Topical steroid

1375-Picture of ear auricle with two papules in it (gouty tophi),investigation needed


 Uric acid in urine
o Xray
o ECG

1541-Tuberous xanthoma and tenderous xanthoma defect in:-LDL receptor

1605-Phototherapy of neonatal hyperbilirubinaemia is accompanied with light having a wave length of:-
400-500 nm.

Match
1637-The diseases with the enzymes in which there are defects:-
 Phenylketonuria phenylalanine hydroxylase,
 Albinism tyrosnase,
 Ochronosis homogentistic acid oxidase,
 Homocystinuria cystathione synthetase.

From the questions in page 78 :

104
32-Sarcoidosis in black, HLA-B8 (highest incidence)
33-Sarcoidosis: Thyroid, cruptococccosis, lymphoma, lung cancer, granuloma annulare (associations)
34-Sarcoidosis; kveim test (6 weeks): brucella, lymphoma, Crohn\s –false positive
From the questions in page 79 :

95-Histtiocytosis x: Bone (the commonest organ affected)


From the questions in page 80 :

120-PCT: Liver disease


121-Pseudo PCT: kidney disease
From the questions in page 81 :

102-PCT: Liver disease

105
Hair
636-Hair development in utero start at
o 7th w
o 9thw
o 11thw
o 13th week
637-The daily growth rate of scalp hair is approximately:-
o 0.3 mm
o 0.1 mm
o 1 cm
o 2 cm
638-Ophiasis is
o A disease of the hair
o A parasitic disease
o Growing together of the eyebrows
o A disease of the eye
o None of the above
639-The "hairs" of black hairy tongue are
o Hyperplastic filiform papillae
o Hyperplastic fungiform papillae
o Fungal hyphae
o Colonies of bacteria
o None of the above
640-Hair loss may be caused by all EXCEPT:-
o Cyclophosphamide
 Tetracycline
o Heparin
o Contraceptive pills
641-Telogon effluvium occurs after all EXCEPT:-
o Physical & emotional stress
o Fever
 Methotrexate therapy
o Severe nutritional deficiency
642-Eyebrows may appear not to regrow after shaving because
o Their growth rate is extremely low
o Their telogen phase is relatively long
o Shaving damages the matrix
o Shaving induces the telogen phase
o None of the above
643-Alopecia due to antimicrobial drugs is called:-
 Telogon effluvium
o Anagen effluvium
o Catagen effluvium
o None of the above
644-Anagen alopecia caused by:
o Cytotoxic drugs.
o Fever.

106
645-A 6 years old boy under chemotherapy presented by scalp hair fall diagnosis is:-
o Alopecia totalis
o Telogon effluvium
o Anagen effluvium
o Alopecia areata
646-Regarding Cicatricial alopecia all are true EXCEPT:-
o DLE
o HZ
 Oral contraceptives
o LP
647-Alopecia areata:
o Cause scaring alopecia
o Presented with patchy hair loss with question mark
o There is a potential for regrowth of hair
648-Patient with cicatriatial alopecia & surround with pustules, papules, on biopsy prolonged remission
neutrophiles &eosinophiles infection from one year (Folliculitis Decalvan)
o Girisofulvin
o Rifampicin
o Steroid
649-About alopecia areata what is TRUE:-
o Nail involvement is 10%
o Increase incidence with atopic dermatitis
o +ve family history in 25% of cases
 All of the above
650-Bad prognosis in alopecia areata:-
o Ophiasis
o Prepubertal with duration longer than 5 years
o Onychodystrophy
o Atopic patient
 All of the above
651-Alopecia areata any of the following is TRUE:-
o Cause localized scarring patch of the scalp
 There is potential ability of the hair to regrow in all causes
o Characterized by question mark hairs
653-Alopecia Aerate in children the safe treatment
o Intralesional steroid
o UVB
o Systemic steroid
654-Children with traction alopecia best treatment
o Intralesional steroid
o Minoxidil
o Change hair style
655-Conversion of testosterone to dihydrolestosterone occurs in –
o Target organs
656-Most common alopecia in men:-
 Androgenic alopecia
o Telogon effluvium
o Anagen effluvium
o Alopecia areata

107
657-If andro-genetic alopecia is manifested in young women, an endocrinological evaluation is especially
indicated if there is also present significant:-
o Acne
o Hirsutism
o Virilization
o All of the above
o None of the above
658-Etiologic causes of hirsutism include all EXCEPT
o Polycystic ovarian syndrome
o Anabolic steroids
o Congenital adrenal hyperplasia
o Acanthosis nigricans
o DLE
659-Studies required in a suspected case of hirsutism include:-
o Serum unbounded testosterone level
o Serum dehydroepiandrosterone sulfate (DHEAS)
o LH/FSH ratio
o Prolactin
 All of the above
660-With virilizing ovarian tumors one observes
o Hirsutism
o Masculinization
o Elevated urinary 17-ketosteroids
o All of the above
o None of the above
661-Perifolliculitis capitis abscedens et suffodiens all except
o Most commonly affects white males
o Is associated with certain forms of arthritis
o Can be painful
o Can be mistakenly diagnosed in children with inflammatory tinea capitis
o Is best treated by oral isotretinoin

1017-Alopecia scaring
 LE/LP

1069-Hirsutism associated with which of the following


o Anorexia
 Juvenile hypothyroidism
o Digoxin toxicity
o C/ocitrate ? ?

1082-Fetus is covered by:


o Vellus hair
 Laugo hair
o Terminal hair
o None of the above

108
1084-What is the keratin in the follicle of hair
o 4
 14
o 18
 81
 84
1085-Patient has a tough hair on the chin and cheek, what the appropriate classification of the degree of
the situation on the scalp
o Fitzpatrick scale (skin colour)
o Ludwig (androgenc alopecia)
 Ferriman gallwey (hirsutism)

1185-A 15 years patient with multiple depigmented and hypopigmented lesions with areas of
hyperpigmentation presented to you , The father has an area of depigmentation with white hair in the
scalp. The most appropriate line of treatment.you choose is (piebaldism):
o PUVA
o NB-UVB
 Culture melanocytes

1211-Pt. Cicatrical alopecia follicular surrounded by papules&pustules&biopsy showed neutrophiles&


eosinophiles infection from one year TTT(folliculitis decalvans)
o Hydroxycloroqin
o Greazovolvin
 Refampicin
o Minoxidile

1244-Female patient complain of hair loss since 1 year with christmas tree appearance,what the diagnosis:
 Female pattern hair loss
o Alopecia areata
o Telegon eff.

1321-Picture of patient with two patches of hair less area in the scalp. Best treatment is
 Intralesional steroids
o Minoxidil

1321-Photo of Alopecia Aerate in children the safe treatment


o Intralesional steroid
o UVB

109
o Systemic steroid
 Topical steroid

1322-Child with alopecia with accompanying graphic:


o Alopacia avenata
o Androgentic alopecia
o Traction alopecia
 Trichetillomania

1323-Figure of child with hair very tighten and oval area with no hair TTT:
o Oral antifungal
 Advice change the habit of tightening hair
o Antipsychotic

1323-picture of alopecia areata, expected nail lesion wih this disease is:
 pitting
o onycholysis
o onychomycosis
1368-Photo of hair with lice TTT:
 Permithrin
o Antifungal

110
From the questions in page 80 :

162-Alopecia areata: Hair re-growth potential in all cases

From the questions in page 83 :

103-DOC for localised alopecia areata- intralesional steroids

From the questions in page 84 :

104-Most effective drug in alopecia areata- contact sensitizers

111
Hypersensetivity ,Urticaria & Angioedema
37-Type IV hypersensitivity is mediated by:-
 T lymphocyte
o Lymphokines
o Langerhans cell
o IgA

687-Concerning wheal all EXCEPT


o Smooth raised flat skin lesion
 Asymptomatic
o Disappears within 24 hours
o Most commonly with urticaria
688-The most frequent cause of urticaria and angioedema is:-
o Food additives
o Foods
o Drugs
 Infections
o Emotional stress
689-About urticaria:-
o Can be non-immunological reaction
o Drugs can caused histamine release without immune reactions
o Infections agents can cause it
 All of the above are true
690-The most common involved mediator in urticaria is:-
o Serotonin
 Histamine
o Gasterin
o Acetylcholine
691-Choose the wrong answer:-
o SLE is more common in renal involvement than dermatomyositis
o Nail involvement is common in LP
o Pemphigus vulgaris is commonly involve mucous membranes
 Urticaria is type III in immunological reaction
692-About urticaria what is TRUE:-
o The most common type in physical urticaria
o Dermogarphism is the most common type of physical urticaria
o Aspirin causes exacerbation of urticaria
o Acquired cold urticaria is the most type of physical urticaria
 All of the above
693-Patient have urticaria and not response to treatment what you should give him
o Methotraxate
o Cyclosporin.
694-Patient have urticated wheal in back, which druge avoid it
o Asprin
o Corticosteroid
695-Which of the following is also known as stress-induced urticaria:
o Pressure urticaria

112
o Aquagenic urticaria
o Cold urticaria
o Cholinergic urticaria
o Papular urticaria
696-Urticaria due to stress is :
o Cholinergic urticaria
o Adrenergic urticaria
o Heat urticaria
697-Stress uricaria
o Cholinergic uricaria
o Cold uricaria
o Aquagenic uricaria
o Pressure uricaria
698-Dermaographism not occur with
o Hypothyroidism
o Hyperthyroidism
o Penicillin allergy
o None of the above
699-White dermogaphism is seen in
o Atopic dermatitis
o Psoriasis
700-In hereditary angioedema there is defect in:-
o Tyrosinase enzyme
o Steroid sulfatase enzyme
o CI esterase inhibitor enzyme
o Cytochrome p 450 enzyme
701-Patient presented to er with facial edema ,difficulty in breathing change of voice and abdominal pain
his brother died from an episode of breathing difficulty. For above case
o Antihistamines will treat the patient
o C1 esterase concentrate is the drug of choice
702-Patient has criticaria and respiratory distress, takes corticosteroid, withdraws in other special clinic,
and now has respiratory distress, urticaria, myalgia, abdominal pain and joint pain. He had two brothers
died from this case this patient must take HAE
o Steroids again
o Bradykinine
o Thalidomide
o C1 inhibitor concentrate
703-Regarding hereditary angioedema all of the following statements are true except:-
o C1 esterase inhibitor levels may be normal or elevated
o Non-pitting edema is known clinical manifestation
o Transmitted as autosomal dominant trait
o The skin lesions are itchy and discolored
o Can be associated with abdominal pain, vomiting and diarrhea
704-All are treatment of angioedema EXCEPT
o Daazol
o Stanazol
o C1 esterase inhibitors
o Corticosteroid + scadrenalin
705-What is not used as prophylactic for hereditary angioedema
o Stanazol
o Danazol (best treatment )

113
o Fresh plasma(used in emergency)
o Traneaxamic acid
706-Lower face edema appear after the patient change with anti HPT drug what may be that new drug
o Captopril
o Atenolol
o Deltazem
o Propranolol
707-Urticaria pigmentosa is characterized by accumulation of mast cells
o At the DEJ
o Around venules
o Around arterioles
o Around epidermal appendages
o Diffusely in the dermis

911-Father admitted to the ICU with shocking three of his sibling have the similar condition (hereditary
angioaedema): what is true about this case
 C1 esterase inhibitor supplement treat the condition

922-Alternative to corticosteroid when tapering it in urticarial patient :


 Cycloporin
o Colchicine
o Cyclophosphamide
o Methotrexate

927Angioedema in siblings:
o Give low dose sedative antihistaminic
o All will have high oesinophilia
 Give anti c1 estrase in attack
o All have low bradykinin

941-The most incriminated drug in EM:


 Sulphonamide
o Penicillin
o Rifampicin

1023-Arthropode stings contains


o Biologically active substances/ histamine/ach/kinins/lecithine

1024-Acid phosphatase
o In platelets

114
1123-Patient has uriticaria and respiratory distress, takes corticosteroid, withdraws in other special clinic,
and now has respiratory distress, urticaria, myalgia, abdominal pain and joint pain. He had two brothers
died from this case this,patient must take
o Steroids again
o Bradykinine
o Thalidomide
 C1 esterase inhibitor concentrate

1150-Treatment of hypocomplementemic urticarial vasculitis


 Hydroxyzine
o Epinephrine
o Hydroxychloroquine
o Indonethacin

1181-Patient have urticated wheal in back, (Darier sign) which druge avoid it
 Asprin
o Corticosteroid

1286-Photo irritated test can be read after:


o 24
o 36h
o 48h
o 72h

115
Infection
DNA virus RNA virus
 HPV  Measles (Paramyxovirus)
 Hhvs  Rubella (Togavirus)
 HHV1 (Herpes simplex)  Coxsaxkie (Hand-Foot-Mouth disease)
 HHV2 (Herpes simplex)  Hepatitis A
 HHV3 (Herpes zoster)  Hepatitis C
 HHV4 (EBV) (infectious  Hepatitis D
mononucleosis)  Hepatitis E
 HHV5 Cytomegalo virus  AIDS
 HHV6 Roseola Infantum
 HHV7
 HHV8 Kaposi sarcoma
 Pox virus
 Vaccinia (monkey pox, Cow pox, Small
pox)
 Orf (milker nodule
 Molluscum contagiosum
 Hepatitis B

276-Drug that best covers impetigo is


o Erythromycin
o Azithromycin
o Clarithromycin
o Clindamycin
o Dicloxacillin
277-Bullous impetigo is caused by:-
o Streptococci
 Staphylococci (coagulase +ve)
o Staphylococci (coagulase -ve)
o None of the above
278-Staph infection can split the DEJ by toxins directed against : (ssss)
o Desmoglein III
o Desmoglein I
o Desmocollin I
o Desmocollin III
279-Which antibiotic has the LEAST activity against Staphylococcus aureus ?
o Erythromycin
o Clindamycin
o Vancomycin
o Dicloxilln
o First generation cephalosporins
280-The main local source of S aureus contaminating the skin is
o Nasopharynx
o Scalp
o Axillae
o Perineum

116
o Mouth
281-Most common among factors predisposing to development of furunculous is
o Nasal carriage of the organism
o Underlying immune disorder
o Anemia
o Diabetes
o Poor skin hygiene
282-The folliculitis caused by contact with cutting oils is due to
o Solidification of the offending oil in the follicle
o Promotion of bacterial growth by the offending oil
o Follicular hyperkeratosis
o None of the above
o All of the above
283-Gram-negative folliculitis is best treated by
o Tetracycline
o Erythromycin
o Doxycycline
o Minocycline
o Isotretinoin
284- A 35 years old man has marked pruritic, flesh colored papules limited to the face, neck, upper chest
and back. A skin biopsy reveals perifollicular inflammation with numerous eosinophils vascular
proliferation and dermal fibrosis. Bacterial and fungal are negative. Serum ige elevated. The most likely
diagnosis is:-
o Chronic granulomatous disease
o Eosinophilic folliculitis
o Eosinophilic cellulitis
o Urticaria
o Hyperrimmunoglobulinemia E syndrome
285-Fatal outcome SSSS is least likely in which of the following age groups
o 0-1 year
o 1-5 years
o 5-15 years
o 15-40 years
o 40-60 years
286-Best treatment of erysipelas in children is:
o Penicillin
o Floxacillin
o Erythromycin
o Azithromycin
287-Necrotizing Fasciitis what possible treatment
o Debridement
o Antibiotic
o Corticosteroid
288-What is the MOST important step in the management of necrotizing fasciitis ?
o Bedrest and pain control
o Histologic confirmation of the diagnosis
o Culture and antibiotic sensitivity testing
o Surgical debridement and decompression
o Assessment of the patient’s immune status
289-Child symptoms had sore throat then L.N. later on palmoplanter peeling and strawberry tongues best
treatment (Scarlet fever)

117
o Erythromycin
o Acyclovir
o Retinoids
o Antipyretic
290-Patient taken phenytoin and after that developed blister and target lesion, erosion 75% of his skin
without oral lesion in probable diagnosis
o EM
o SJS
o TEN
o Drug exanthen
291-Patient presented to ER congestive eye after she receive phenytoin she has targetoid lesion on her
hand & feet she developed skin lesion dusky cooler which peel sheets she suffer from painful oral lesion.
What is your diagnosis
o HSV
o EM
o SJS
o TEN
292-Rate of mortality in TEN with Steven Johnson syndrome:-
o 5%
o 5%-15%
o 15-30%
o 50%
293-Most common causes of death in TEN
o Electrolyte disturbance
o Hemodynamic shock
o Sepsis
o Renal failure
o Hepatic failure
294-In patients with extensive erythrasma one should rule out
o Cancer
o Diabetes mellitus
o Hypothyroidism
o Chronic renal disease
o Addison disease
295-Regarding erythrasma, all true EXCEPT:-
o Caused by coryenbacterium minutissimum
o Response to erythromycin
 Healing center & vesicles in periphery
o Coral red in woods light
296-Axillary lesion that gives coral red fluorescence with woods light is:-
o Erythrasma
o Seborrehic dermatitis
o Psoriasis
297-Tricomycosis axillaries all EXCEPT
o Is a superficial fungal infection
o Is caused by corynebacterium species
o Is often related to poor personal hygiene
o Can be associated with hyperhidrosis
o Can be treated by nafitine hydrochloride 1%
298-Patient has a red maculo-papular rash which begins on wrist, ankles and then progress to form
petechiae, the most diagnosis

118
o Rocky mountain fever
o Measles
o Rubella
299-Rocky mountain spotted fever (RMSF) is caused by:-
o R. Ricketsii
o R. Prowazaki
o R. Typhi
300-Which of the following organism don’t have systemic involvement like vasculitis and necrotic
change
o Pseudomonas aregonose
 Group B streptococcus(streptomyces)
o Leprosy
o Aspergellous
301-The causative organism in cat disease is
o Bartonella quintata
o Brucella melitenesis
o Bartonella bacilliformis
o Bartonella henselae
o Coxiella bumetti
302-The highest concentration of organisms in an infant with early-onset neonatal listeriosis is in the
o Liver
o Lungs
o Brain
o Skin
 Blood
303-The treatment of choice for listeriosis is intravenous
o Ciprofloxacin
o Dixocycline
o Minocycline
o Ampicillin
o Cephalexin
304-The best treatment for steatocystoma suppurativa is
o Oral isotretinoin
o Topical metronidazole
o Oral tetracycline
o Topical vitamin D3 analogues
o Intralesional IFN-α
305-On culture reveals klebsiella best treatment is
o Steroid
o Doxycycline, tetracycline
o Retinoid, isotretinoin
o Dapson
o Clofozamin
306-All of the following are RNA viruses EXCEPT:-
o Measles
o V.Z.V
o Rota virus
o Rhinovirus
307-The following are DNA viruses EXCEPT:
 Measles.
o Varcella zoster virus

119
o Epstein bar virus
308-Which of the following viruses are RNA:-
o Herpes simplex
o Hepatitis C
o HPV
o Hepatitis B
o Molluscum contagiosum
309-Which of the following virus are DNA virus:-
o Measles
o Rubella
o Hand-foot-and-mouth disease
o Infectious mononucleosis
o Hepatitis A
310-Regarding herpes simplex all true EXCEPT:-
 Type I is more recurrent than type 2
o Fever & lymphedema may occur
o Tzank smear is diagnostic
311-Primary herpetic gingovostomatitis differs from herpagnia mainly by
o The age group involved
o Absence of systemic illness in the later
o Presence of vesicles in the former
o More posterior location of lesions in the latter
o None of the above
312-Recurrent vesicles on penis
o HSV
o Chancre
o Chancroid
313-Nerve most commonly affected in herpes zoster:-
o Thoracic
o Trigeminal
o Lumbar
314-A man 69 years old with herpes zoster, which of the following is an indication for ophthalmic
consultation:-
o Involvement of maxillary nerve
o Involvement of auditory nerve
 Involvement of the tip of the nose
o Facial paralysis
315-Varicilla complication in adult most common
o Hepatitis
o Encephalitis
 Varicilla pneumonia
o Reye’s syndrome
316-Disease which may present as grouped vesicles and bullae include:-
o Herpes simplex
o Herpes zoster
o Dermatitis herpetiform
o Adult linear iga bullous dermatosis
o All of the above

317-About verruca vulgaris all are true EXCEPT:-

120
 It is caused by HTLV in virus
o Some of the lesion may resolve spontaneously
o Some serotypes of the causative virus is potentially oncogenic
o Genital warts can be treated by podophylline
318-Adult molluscum contagiosum is common to:-
o Mucous membranes
 Genitalia & nearby areas
o Palms
319-Giant molluscum contagiosum in an immunocompromised HIV-infected individual can be treated by
o Curettage
o Phenol
o Lazarotene 0.1%
o Topical tretinoin 0.25%
o Topical imiquimod cream
320-Erythema infectiosum is caused by:-
o Varicella zoster virus
o Parvovirus B19
o E.B.V
o H.T.L.V
o Coxsackie virus
321-Viruses cause Morbilliform eruption include:-
o Erythema infectiosum
o Measles
o Pityriasis rosea
o Rubella
 All of the above
322-Measles is infectious
o After 2 days of contact with household relatives
o From the appearance of the enanthen
o From the appearance of exanthema till skin desquamation
o Three days before the prodrome till the enanthem
o From the prodrome till 5 days of the exanthema
323-The most common dermatophyte worldwide is
o T. Mentagrophytes
o T. Rubrum
o M. Canis
o E. Floccosum
324-All type of fungal microorganism diagnosis with wood’s light give florescence except :
o T. Sheonlline
o Violisicum
o M. Canis
325-A fungal culture grew light brown, cottony aerial mycelia that became powdery in the center. There
were numerous spindle shaped multiseptate. Microconidia organism is
o T. Tonsurans
o T. Mentagrophytes
o T. Rubrum
o M. Canis
o M. Audouinii
326-What is the slowest growing zoophilic fungus in tissue culture ?
o T. Equinum
o T. Verrucosum

121
o T. Mentagrophytes
o M. Canis
327-Which of the following organisms is MOST likely to cause tinea capitis in adult ?
o Microsporum audouinii
o Trichophyton mentagrophytes
o Epidermophyton floccosum
o Trichophyton tonsurans
o Pityrosporum orbicularis
328-One of the following fungi is characterized by endothrix type hair invasion:-
o Trichophyton tonsurans
329-Which is false about tinea capitis:-
o Common in children
o Black dot tinea capitis is caused by T tensurans
 Kerion is preceded by patch of alopecia
330-Most serious complication of untreated kerion is:-
o Secondary bacterial infection
o Non cicatricial alopecia
o Cicatricial alopecia
o Spread of infection
331-Most cases of tinea barbae are caused by
o Zoophilic fungi
o Anthropophilic fungi
o Geophilic fungi
o Non dermatophyte fungi
o Saprophytic fungi
332-Fungal infection on subungual Pedis , what is the causative organism :
 T. Rubum
o T.tensorance
333-Endonyx onychomycosis is caused mainly by
o T. Soudanes
o T. Rubrum
o T. Mentagrophytes
o Asperigillus nigra
o Scopulariopsis brevicaulis
334-Onychomycosis of fingernails is best treated by
o Girisofulvin, 12.5 mg/kg/day for 3 months
o Fluconazole 150 mg/week for 4 weeks
o Ketoconazole 200 mg/day for 6 weeks
o Terbinafine 250 mg/day for 6 weeks
o Itraconazole 200 mg daily for 7 months
335-All are useful in diagnosis of Pityriasis versicolor EXCEPT:-
o Wood's light
o Clinical examination
 Skin biopsy
o Scrub & KOH
336-Brown or black patches on palms & soles diagnosed tinea nigra is caused by:-
o Phareoannellomyces werneckii
o Microsporum canis
o Trichophyton rubrum
o Epidermophyton flocusum
337-About fungal infection all are false EXCEPT:-

122
o Tinea versicolor if treated never recur
o A circinate lesion of healed center & active edge is characteristic of T capitis
o Coral red fluorescence is seen in T capitis
 Thrush, paronychia & napkin rash are manifestation of moniliasis
338-The relationship of C. Albicans to the skin is best described as
o Resident flora
o Transient flora
o An absolute pathogen
o An opportunistic pathogen
o An accidental contaminant
339-Intertrigo may be secondarily infected with:-
o Streptococci
o Staphylococci
o Candida
o A superficial dermatophyte
 All of above

340-The safest drug for treating sporotrichosis during pregnancy is


o Ciprofloxacin
o Itraconazole
o Fluconazole
o Ketoconazole
o Amphotericin B
341-The causative organism of sporotrichosis is
o Histoplasma capsulacum
o Sporothrix schenckii
o Phialophora verrucosa
o Exophiala jeanselmei
342-Regarding Sporothrix schenckii, the following statements are true except
o It is dimorphic
343-Bloodstream dissemination is common
o Lymphocutaneous sporotrichosis is the most common presentation
o The lipid compound of the cell wall inhibits phagocytosis
o It can grow on brain-heart infusion
343-Cryptococcus neoformance is commonly disseminated to:
o Lung
o GIT
o CNS
o Kidneys
o Bone
344-Cryptococcosis ( Cryptococcus neoformans) is most commonly affect:-
o C.N.S
o Skin
o Lungs
o Bone
345-The antibiotic least likely to be successful in treating an actinomycetoma is
 Tetracycline
o Streptomycin sulfate
o Dapsone
o Rifampicin
o None of the above

123
346-Treatment of actinomycosis
o Streptomysin + miconazol
 Septrin + streptomycin
o Penicillin
347-Mycetoma madura foot (case discreption) treatment
o Streptomycin or amikacin
o Voriconazole
 Imidazole dervatives
o Itraconazole and terbinafine more than ketconazole
348-Which mycetoma is more inflammatory, more destructive, and invasion of bone at an earlier period?
o Eumycetoma
o Actinomycetoma
o The mycetoma that remains encapsulated for the longest period
o They mycetoma that produces most fibrosis
o None of the above
349-The treatment of choice for lobomycosis is
o Itraconazole
o Surgical excision
o Ketoconazole
o Terbinafine
o None of the above
350-The period between exposure to scabies & development of symptoms is:-
o One day
o One week
o Two weeks
 Four weeks
351-About scabies all true EXCEPT:-
 More in back
o Iry lesion in burrow
o Sarcoptes scabiei has 8 legs
o Black fecal concretion of sarcoptes scabiei known as scybala
352-Characteristic lesion of scabies
o Burrow
353-Scabies typically spare the flowing sites:-
 Mid upper back
o Fingers
o Buttocks
o Genitalia
354-Scabies differs in infants form adults in all EXCEPT:-
o 2ry infection is more common
 Burrows are rare
o Wide spread involvement in atypical sites as palms, soles & neck
o Presence of vesicles & bullae
355-When treatment scabetic patient what point u will not consider:
o Efficacy and toxicity of drug
o Number of family members affected
o Cost of the drug
o Age of patient
356-The following are true about ivermectin for pediculosis treatment EXCEPT
o It is not ovicidal

124
o Given as 2 doses one week apart
o Best drug in epidemics
357-A 5 year old child infected with pediculosis best treatment is
o Apply topical for 10 minutes and wash and then fine combing
o Apply topically overnight then fine combing
o Combing with electronic device
358-Fleas are vectors for all of the following diseases EXCEPT
o Typhus
o Fungiasis
o Bubonic plague
o Onchoceriasis (african river blindness)
o None of the above
359-A dermatologist may select the following therapies to treat cutaneous Lieshmaniasis EXCEPT
o Cryotherapy
o Parenteral pentostam
o Oral Itraconazole
o Systemic corticosteroids
o Local application of heat
360-Therapeutic modalities of Lieshmaniasis include all EXCEPT
o Rifampicin 600-1200 mg/day for up to 3 months
o Itraconazole 200 mg/day for 4-8 weeks
o Ketoconazole 200 mg 3 times daily for up to 3 months
o Rifampicin + intralesional SSG
o Topical 5% fluorouracil
361-Lesion upon pressure by a glass slide give apple jelly appearance is:-
o Lupus vulgaris
o Erythema nodosum
o Lieshmaniasis
362-The most common cause of skin TB is:
o Scrofloderma.
o Lupus vulgaris.
o V.cutis.
363-The most common lns involved in Scrufuloderma are:-
o Inguinal lns
o Cervical lns
o Mesenteric lns
o Epitroclear lns
364-The principal reason for using a combination of drugs in the chemotherapy of TB is
o To prevent side effects
o To ensure effect against other mycobacteria
o To avoid drug resistance
o To prevent overgrowth by opportunistic pathogens
o For economic reasons
365-Thick of nerve and loss of sensation occurs early
o TL
o LL
o BL
o BT
366-Leprosy stained with :
o Ziehl Neelson stain
367-Lepra cells or Foam cells of Virchow occur in:-

125
o Tuberculoid Leprosy
o Borderline Tuberculoid
o Borderline Leprosy
o Borderline Lepromatous Leprosy
o Lepromatous Leprosy
368-Foam cells are seen in:-
o SLE
o Scleroderma
o Lepromatous leprosy
o DM
369-Neural signs of leprosy:-
o Anesthesia
o Muscle wasting
o Trophic changes
o Nerve enlargement
 All of above
370-Mitsuda test is completely done in:
o 3 days
o 7 days
o 10 days
o 15 days
o 28 days (3-4 wks)
371-Lepromatous leprosy is characterized by all EXCEPT
o Numerous symmetric erythematous or shiny ill-defined macules, papules or nodules
o Affection of internal organs
o Strongly positive lepromin test
o Vacuolated macrophages in histology
o Peripheral nerve involvement
372-Leprosy best diagnostic tool is
o CBC
o Slit skin smears
o Lepromin test
 Skin biopsy
o Histamine test
373-Retired, endontulous 60 years old male lives alone, eating only biscuits soaked in water , he
developed extensive ecchymotic patches , perifollicular hemorrhagic purpuric lesions , what is the
diagnosis :
 Scurvey
o Senile purpura
o Leakemia
o Lp
o Psoriasis
374-Leprosy bactericidal drug
o Rifampicin
o Cloforzemine
o Dapsone
o Minocycline
375-Erythema nodosum may show all of the following EXCEPT
o Ulcer
o Erythema
o Hotness

126
o History of drugs
376-Hypopigmented patch indurated plaque after therapy painfull papules nodules(erythema nodosum
leprosum) Drug choice:-
o Dapsone
o Thalidomide
o Tetracyclines
o Corticosteroids

862-The most common suberficial fungal infection:


o T. capitis
o T. corporis
 T.pedius
o T. cruris

878-Leatest complication of varicella


o Encephalitis
o Peneomonia
 Cardiac (Pericarditis)
o Liver

923-Honey crusted lesion in 15y pt :


Imptigo

937-Patient with certain lesions on investigations revealed hyphae what is the best treatment?
o Septrin and streptomyces
 Imidazole derivatives
Corticosteroids

943-Acute febrile neutrophilic dermatosis associated with


o Multiple myloma
 M.leukemia
o Hepatitis

965-As regard to scabis the burrow contain


 10-12 mites
o 20-30 mites
o 30-40 mites
o About100 mites
964-The best TTT of scabies
o Application of topical benzyl benzoate 25% daily for 7 days (NO)
 Scrabing and signle topical use of benzyl benzoate 25% ( YESS)

980-Treatment of erythema nodosum leprosum


o Prdnisone 40-60 mg
 Thalidomamide100-400 mg
o Clofazimine 200 mg-d

127
984-Scabies infestation , all true except
o Rarely involve head and neck
o 5 % lindane is effective
o Benzobenzoates is equally effective to 5% lindane
 Itching occurs 1week after infestation

997-Pyoderma gangrenosum lesion search for the following except


 Bact,viral,fungal infection
o Active acute and chronic hepatitis
o I.B.D
o Lymphoma

998-In pyoderma gangrenosa investigation is done to exclude:


o Bacterial infection
 Active liver disease
o Gammopathy
o Diabetes

1000-Patient had lesion on right axilla with discharging sinuses and bluish red nodules in the neck and
left axilla. Mantous testis non specific.with stain Gm +ve rods, your diagnosis
 Scrofuloderma
o Asperigillosis
o Blastomycosis

1014-Competes with epidermal melanocytes to form depigmentation of the skin is:


o Albinism
o Vitiligo
 Pityrasis versicolour

1029-Botryomycosis is chronic granulomatus disease coused by staph or pseudomonas


Ttt floxacillin/erythromycin/fuciceam

1030-Balanoposthitis its most common cause


 DM
1031-Gram +ve bacteria (micrococcus sedentarius)
 Pitted keratosis

1037-Pyoderma gangarenos TTT all EXEPT


o &interferon
 Surgical procedure
o Antibiotic
1383-Picture of kerion, what is the treatment:
 Systemic greisofulvin
o Topical antifungal
o Antibiotic

128
o Incision &driange

1056-Pediculus humanus var.corporis (Body lice) is the vector in:


o Endemic typhus
 Epidemic typhus
o Q fever
o Rickettsial pox

1072-Mach the following


o Tinea pedis→t.rubrum
o Kerion →m.canis
o Verrucous t.pedis→e.floccosum
o Bullous t.pedis→t.mentagrophytes
o Tinea impricata→t.concentricum
o Tinea nigra phaeo -→annellomyceswerneckii

1100-An immunocompromised patient presents with a painful vesicular rash that has a dermatomal
distribution. The most appropriate management for this patientis:
o Oral acyclovir
o Analgesics and drying compresses
o Systemic corticosteroids
 I V acyclovir and hospital admission

1101-All of the following conditions classically produce lesions involving the soles and palms except:
o Erythemamultiforme
 Chickenpox
o Secondary syphilis
o Hand-foot-and-mouth disease

1102-A pink to red maculopapular rash that starts on the face and rapidly spreads to the trunk and
extremities and is associated with lymphadenopathy (especially postauricular, suboccipital and posterior
cervical) is most characteristic of:
 Rubella
o Roseola

129
o Rubeola
o Varicella
1103-All of the following statements regarding erysipelas are true EXCEPT:
o It occurs most commonlyon the face and the lower extremities.
 It is caused by staph. Aureus.
o It is atype of cellitis.
Exam typically reveals a tender, red plaque with raised and sharply demarcated borders

1106-A tzanck smear can be useful in making the diagnosis of all of the following EXCEPT:
o Herpes simplex (types1and2)
o Varicella (chickenpox)
 Erythema multiforme
o Herpes zoster (shingles)

1108-The presence of "target lesions"(erythematous plaques with dusky centers and bright red borders)
on the palms and soles is classically associated with:
 Erythema multiforme
o Secondarysy philis
o Disseminated gonococcal disease
o Pityriasis rosea
1131-Familial mediternian fever manifested by
o Purpura
o Erythema
 Erysipelas like

1174-SSSS
 Staph aureus group 2 subtype71
o Sepsis and death
o Identify organism by doing culture from base of lesion

1207-Scabies not present in


 Scalp
o Wrist
o Axilla
o Genitalia

1209-Lepromatous leprosy site of skin biopsy is


o lesional site
 Perilesional site
o anesthetic site

1214-Ritts disease (SSSS) TTT is: (β-lactamase-resistant antibiotic)


 NSID
o Sys. Steroid
o MTX

130
o Dapson

1232-10 years old child with fever rash prominent of tongue papillae tachycardia irregular pulse
o Toxic shock syndrome
o Scarlet fever
o Measles
 Kawasaki

1238- 45 y old man, sudden eruption all over the body with palm and foot involvement. Most likely Dx
is:
o Syphilis
 Erythema multiforme “most probably”
o Erythema nodosum
o Fixed drug eruption
o Pityriasis rosea

1254-Sock and gloves caused by:


o Cytomegalovirus
o Parvo virus
o EBV

1277-Latest systemic oral medicine for scabies is


 Ivermectin
o Benzyl benzoate
o Carbaryl
o Gamma benzene hexachloride

1278-Which of the following is found least frequently in patients with TEN (Toxic Epidermal Necro-
lysis)?
o Anemia (erythrocytic)
o Lymphopenia
o Neutropenia
 Eosinophilia

1296-In measles, infective prodroma is followed by


 Exanthema on fifth day

1302-Candida
 Not commensal flora
 Vaginal flora

1304-Child with purpura (Acute heamorrhagic pupura of childhood),what is the least cusative organism:
 HSV
o CMV
1305-Exanthema subitum (roseola infantum)
 HHV6
o Parvovirus

131
1306- 34 y patient with hypopigmentation in neck with history of plaque or nodule with normal sensation
and fever from 1 year
 Post kala azar leishmaniasis
1307-Sneddon-Wilkinson disease (Subcorneal pustular dermatosis)
 Hypopyon abcess

1316-Buruli ulcer
 Mycobacterium ulcerans

1334-Picture for baby with crust redness face with history of atopy, what is true:
o History of conjunictivities not run in families in this condition
 More prone to staph infections
o Not inherdit condation
o Washing with soap will decrease condition

1368-Itchy scalp , cervical LN on hair


 Lice
1343-Figure face lesion on chin
o HSV
o Lupus vulgaris
 Tooth abscesses dental sinus

1344-Investgation for dental sinus


 Tooth X-ray

1345-Figure of thickened toe nails -ve fungal with skin colered papules in forehead neck chest and
progressive also father and sister has the same what we must invsestigate:
(DNA-biobsy-KOH for exclude onychomycosis-rarely eye )
o Chest
o ?

132
1369-Photo of hair with lice TTT:
 Permithrin
o Antifungal

1385-Diagnosis
 molluscum

1391-???
 Eczyma herpeticum

1392-Patient with anaesthetic macule behind the ear for 5 years with enlargement of the greater auricular
nerve. Best treatment regimen will be (with picture)
o Dapsone 100mg daily + rifampicin 600mg monthly for 6 months
o Dapsone 100mg and clofazimine 50mg daily + rifamicin 600mg and clofazimine 300mg monthly for
12 month

1393-Photo Eman him glasses and hand with lesions nerve involvement with weakness and loss of
sensation and the nerve become more thikness is began
o TT
o BT
o B
 BL
o LL

1416-The optimum time for culturing candida is 24 hours while for dermatophytes is 2 weeks
o False
o True

133
1436-Tinea versicolor is caused by epidermophyton
o True
o False
1437-Girisofulvin is effective treatment in candidiasis
o True
o False
1463-Herpes zoster is the effect of Iry infection with varicella zoster virus
o True
o False
1490-The ulcerative form of impetigo is called ecthyma
o True
o False
1491- Candidiasis is common with humeral immunodeficiency
o True
o False
1492Candidiasis is common with cellular immunodeficiency
o True
o False
1494-Tinea versicolor may be transmitted by long contact
o True
o False
1495-Psychosis barbae causes scarring alopecia
o True
o False
1532-Gram-ve folliculitis is classified as different from acne vulgaris mainly because of:-its flora and
the absence of comidones.
Eumycetoma (IP 20y ) ( fungi ) - keticonozale, Itraconazole
Actinomyretoma ( IP 5y ) ( Bacteria ) - streptomycin + dapson
-streptomycin + TMP-SMZ
- dapsone + TMP-SMZ

1547-Punch out ulcer on toe and also on bony prominence with no pulsation, no hair and painful:-
pyoderma gangrenosuus
1559The best treatment of pyogenic granuloma is:- surgical cautary.
1560-Causative virus of oral hairy lecoplakia:- E.B. virus.
1561-Antimalarial ophthalmic examination every- 1-3m
1563-Causes of molluscum contagiosum:- pox virus.
1566-Kawasaki treatment:- aspirin 100mg/kg +IV Ig
1575-Most common fungal infection all over the world:- tinea pedis.
1580-Leishmaniasis the etiology:- leishmania tropica.
1585-The most important source of infection for neonatal candidiasis is:- the mothers vagina.
1586-The following organisms does not commonly colonize healthy skin:- candida albicans.
1587-Highest carrier rates of staph pyogenes are found in:- infancy.
1590-Most common superficial infection in the world:- Ptyriasis versicolor.
1591-Acyclovir is given as prophylactic in recurrent Herpes simplex:- 200mg ∕ 5 time or 400mg ∕ 3 time
1592-The mortality rate of untreated steven-johnson syndrome is:- 5-15%. ‫ فى الكتاب‬10-20%
1593-E.M. associated with T.E.N. mortality rate :- 15-30%. ‫ فى الكتاب‬30%
1601-Lyme arthritis is associated with:- erythema chronicum migrains.
1601-Pyoderma gangrenosum, especially if superficial and bullous should be cause one for:- leukemia.
1601-Characteristic features of jop syndrome do not include:- pseudomonas abscess.

134
1601-Susceptibility to tinea imbricate to be related mainly to:- inheritance.

1647-
T. Capitis M. Canis, M. Audouini, M. Gypseum (small ectothrix) (Gray patch)
M. Canis, M. Audouini, M. Gypseum (small ectothrix) (Kerion)
T. Tonsurans, T. Violaceum (Endothrix) (Black dot)
T. Schoenleinii (Favic) (Favus / Adult)
T. Pedis T. Rubrum & occasionally E. Floccosum (Hyperkeratotic - moccasin)
T. Mentagrophytes & occasionally E. Floccosum
(Vesiculobullous)
T. Mentagrophytes (Ulcerative)
T. Mentagrophytes (Macerated toe web)
Onychomycosis T. Rubrum (DLSO)
T. Mentagrophytes (SWO)
T. Rubrum, T. Mentagrophytes (PSO) 90% with AIDS
T. Rubrum, T. Mentagrophytes (EO)
T. Imbricala T. Concentricum

From the questions in page 78 :

2-Herpes zoster: thoracic is much more important than all other types.
3-Herpes zoster: Nose tip is supplied by a branch of ophthalmic (nasociliary)
4-Herpes simplex: Recurrence may be a cause of erythema multiform.
5-Herpes simplex: Recurrence 6 times/year necessitates prophylactic ttt.
6-HIV: The most common bacterial infection (aureus)(superficial folliculitis)
7-HIV: the most common visceral affection: GIT
8-HIV: the most common occurs mostly in GIT, lungs, NOT in penis.
9-HPV: Serotypes 16 & 18- C accuminatum- cancer cervix
10-Epidermodysplasia verrociformis: ↓↓CMI (normal humoral)
11-Hand-foot-mouth disease: • Coxsaxkie
12-HPV6: roseola infantum
13-Parvovirus B19: Erythema infectiosum
14-T pedis: most common fungal infection.
15-T. Capitis: adults T. Tonsurans (most common)
16-T. Capitis: children: T tonsurans, M canis (most common)
17-T. Nigra Palmaris; Phaeonnellomyces (Exophiola) werneckii
18-Piedra, black: Pieraia hortae
19-Piedra,white: Trichosporon beigelii
20-Aspergillosis: neutropenia & systemic steroids (most important risk factors)
21-Aspergillosis:(culture features)
22-Mycetoma: dapsone + streptomycin ttt of choice
23-TB; lupus vulgaris, commonest cutaneous variant
24-TB; lupus vulgaris, apple jelly nodules (on diascopy,characteristic)
25-TB;scrofuloderma:lateral neck (most common site)
26-TB, BCG: a live attenuated vaccine
27-Leprosy: skin biopsy (best diagnostic test)
28-Leprosy: early nerve affection (in TT)
29-Leprosy: Fernandez test +ve 24-48h
30-Leprosy: Mitsuda test +ve 3-6 weeks
31-Scabie:Spares mid-back

135
From the questions in page82 :

1) Most common organism causing tinea- Trichophyton rubrum


2) Most common organism causing tinea capitis- Trichophyton violaceum
3) Most common cranial nerve involved in Hansens- facial
4) Most common nerve taken for nerve biopsy in Hansens- radial cutaneous (upper limb), sural
(lower limb)5
) Most common cause of mononeuritis multiplex - Hansen (India), DM (world)
6) Most common cause of ENL- LL> BL7) Most common cause of a negative Slit skin smear in
Hansen- neural leprosy
8) Cause of Type 1 reaction- BB> BT> BL
9) DOC for type 1 and type 2 reaction- steroids
10) DOC for chronic, recurrent ENL- thalidomide
11) Most common side effect of dapsone- hemolytic anemia
12) Most common side effect of clofazimine- pigmentation
13) Most common cause of inverted saucer lesion- borderline leprosy
14) Most common cause of leonine facies- LL
15) Earliest sensation lost- temperature
16) Most common Hansen- Borderline Tuberculoid
18) DOC for tinea - terbinafine
19) DOC for tinea capitis- griseofulvin
20) Most common type of onychomycosis – Distal and lateral onychomycosis (In HIV, the most
common type is proximal subungual onychomycosis and superficial whiteonychomycosis)
21) DOC for sporotrichosis- itraconazole> potassium iodide
22) Most common cause of reactive arthritis- Chlamydia> Shigella

From the questions in page84 :

113) Most common cause of erythema nodosum- Strepococcus.

119) Commonest cause of apple jelly nodules- Lupus vulgaris


120) Commonest cause of hypopigmented, minimally scaly macules and patches on chest and
back of young adults- P. Versicolor

136
Internal medicine
ORANGES IN MEDICINE
o *orange placenta---- syphilis
o *orange tonsils --- Tangiers disease
o *peau'd orange appearance--- ca breast
o *orange discolouration of urine-- dehydration
o drugs--rifampicin, furazolidone, sulfosalazine, Rifabutin,Entacapone,Clofazimine
o *orange discolouration of skin--carotenodermia
o *orange/saffron coloured amniotic fluid-- post maturity
o *Orange skin cornea is seen in Mustard Gas poisoning used in biological warfare...
o *orange cylinder - - Cyclopropane

1291-Diabetes mellitus is more common with:


o Erythro coproporphyrin
o Erythro pro porphyria
o Congenital erythropoietic porphyria
o Hemosidrosis
 Hemochromatosis

1599-The association of diabetes mellitus, hepatic cirrhosis, cardiac failure and cutaneous
hyperpigmentation is characteristic of:- hemochromatosis.

1622-Site of diabetic neuropathy:- Soles.

137
Kaposi Sarcoma
421-Etiological agent of Kaposi sarcoma is:-
o Herpes simplex virus type 1
o Herpes simplex virus type 2
o Herpes simplex virus type 6
o Herpes simplex virus type 8
422-The main proliferative cellular elements in Kaposi sarcoma are
o Endothelial and fibroblastic
o Lymphoblastic and histiocytic
o Fibroblastic and granulocytic
o Neuroblastic and endothelial
o Endothelial and neuroblastic
423-Classic Kaposi sarcoma occurs most commonly in:-
o Africans
o Jewish
o Southern & Eastern European
o Asian
424-Classical kaposis sarcoma occurs most common in
o Mediterranean and ashkenazi jewish descent
o Negroids in central Africa
425-The site most frequently involved KS is
o Face
o Legs and feet
o Lymph nodes
o Liver
o Bone
426-Kaposi can manifest in young African as
o Benign nodular
o Aggressive
o Florid
o Lymph adenopathic
427- 19 yrs. Old African male with +ve HIV & suffering from Kaposi sarcoma, what is the clinical
presentation of the sarcoma:-
o Nodular lesion
o Macular patches
o Hyperpigmented brown maculo-papular lesion
o Ulcerative lesion
o All of the above
428-KS with AIDS
o Lesion usually pruritic
o Tumor never metastasis
 EBV nearly always found within the tumor cell
o Number of lesion decrease during period of opportunistic infection
429-Kaposi sarcoma in HIV patient most commonly associated with:-
 EBV
o HCV
o Molluscum contagiosum virus
430-Which of the following are more affected in HIV associated Kaposi sarcoma:-
o Lungs & GIT

138
o Lungs & Urinary bladder
o GIT & Urinary bladder
o Urinary bladder
431-Kaposi's sarcoma AIDS-related:
o Always painful lesions
o Mimic bacillary angiomatosis and dermatofibroma1
 E.barr virus always within the tumor2
o Lesions development decrease when opportunistic lesions increase

882-Clssic kaposi, sarcoma all are true EXCEPT:


o A prepondence of people of jewish ashkenaziandor mediterranean descent
 Also documented in northern european countries
o A specific hla marker is not known
o Male to female ratio of 15-1
o Median age of onset 64-66years

1121-The lesion most commonly found in patient with Kasaba-Merite syndrome is:
o Retiform hemangioendethiloma
o Epitheloid hemangioendethiloma
 Kaposi hemangioma

1206-Endemic kaosi sarcoma which is true


 LN are more affected than classic
o Contagious
o Ulcerative
874-Kasabach Merritt syndrome associated with:
o Angiosarcoma
o Kaposiform hemangioendothelioma+tufted angioma‫اكتر االورام الوعائية المرافقة‬+platet
trapping+consumption coagulopathy

139
Melanoma

635-Skin disorders that have been associated with MM include all of the following except
o Scleroderma
o Scelromyxedema
o Polyarteritis nodosa
o Subcorneal pustular dermatosis
o Sweet syndrome

899-Signs of malignant transformation of melanoma


 Asymmetrical shape
 Irregular borders
 More than one color
 Uneven distribution of color
 Greater than 6 millimeters in diameter
 Evolution

579-The most common sites of malignant melanoma in Black & Asians:-


o Mucous membranes
o Palms
o Soles
o Nail
o All of the above
580-Latest affected site by malignant melanoma
o Liver
o Brain
 Bowel
o Lung
o Bone
581-Least affected site by MM
o Liver
o Brain
o Bowel
o Lung
 Bone marrow
582-Malignant melanoma with ↑ S 100 protein high level means:
o Impeding metastasis
o Metastasis is excisting
583-Increased s100 in melanoma indicates:
 Metastatic tumor
o No complete surgical removal of 1ry tumor
584-The most common type of malignant melanoma is:
o Nodular melanoma
o LMM
 Superficial spreading melanoma
o Acrolentigomaligna melanoma
585-Malignant melanoma in female appears on:

140
o Upper limb
o Face
o Head & neck
o Chest
o Lower limb
586-The most common type of malignant melanoma that strongly related to sun exposure is:
o Nodular melanoma
o LMM
o SMM
o Acrolentigomaligna melanoma

587-Most nevus cell nevi of the oral mucosa are


o Junctional
o Compound
o Intramucosal
o Blue
o Malignant

588-Typical features of melanocytic nevi include


o Usually appear in early childhood
o Develop after the age of 40 years
o Juvenile nevi are smooth, papillomatous, hairy nodules
o Intradermal nevi are circular, brown macules <10mm in diameter
o 20% lifetime risk of malignant transformation
589-Lentigo simplex should be different clinically from:
o Lentigomaligna
o Freckles
o Junctional melanocytic nevus
590-Lentigo simplex should be clinically differentiated from:-
o Lentigo maligna
o Junctional melanocytic nevi
o Freckle
591-Small and giant congenital nevi share the following histologic features
o Nevomelanocytes are present within the lower two third of the dermis and within the subcutaneous
tissue
o No nevomelanocytes are found within hair follicles, sebaceous glands, eccrine apparatus, vessel walls,
and nerves
o The presence of perivascular and perifollicular inflammation resembling a figurate erythema
o All of the above
o None of the above
592-The following statements regarding Atypical moles are correct except
o They are in blacks and middle eastern populations
o They can occur in sun-protected areas
o They often are larger in size than common moles
o Lesions have a well-defined edge with uniform pigmentation
o Lesions can progress to melanoma
593-All are changes suggestive malignancy in pigmented navi except;-
o Presence of new pigment around a nevus
o Development of uneven surface pigmentation
o Sudden enlargement of previous quiets mode
o Increase in hair in hairy nevus

141
1020-Acral malignant melanoma
 More in black people

1230-Stage 3 MM good prognosis when spread to :


o Liver
 Skin (SC)
o LN
o Brain

1287-What is the most common types of melanoma?


 Superficial spreading melanoma
o Acral lentiginous melanoma
o Mucosal melanoma
o Nodular melanoma
o Polypoid melanoma

From the questions in page 79:

73-Malignant melanoma:50 years old (peak incidence)

74-Malignant melanoma in ♀ mostly on:- lower limb.

75-Malignant melanoma in ♂ mostly on:- trunk.

76-Malignant melanoma occur in:- giant nevus.

77-Malignant melanomas of the oral mucosa are located on the:- maxillary mucosa.

78-Malignant melanoma related to sun exposed:-lentigo malignant.

79-Malignant melanoma all are right except:- hair in hairy nevous.

80-Malignant melanoma (Acral) is:- Central European and Asian.

81-Malignant melanoma: brain site affected

82-Malignant melanoma: giant congenital nevus(4-10% time risk)

83-Malignant melanoma: pigment in a nevus (a sign of malignancy)

84-Malignancy melanoma: hair growth in a nevus (NOT a sign of malignancy)

From the questions in page 83:

96) Most common melanoma- superficial spreading melanoma


97) Poorest prognosis in melanoma- nodular

142
Mycosis fungoids
566-Large plaque parapsoriasis is associated with:-
o Mycosis fungoids
o Hodgkin lymphoma
o Kaposi sarcoma
567-Besides the skin, the most frequent organ involved in Mycosis Fungoids is the:-
o Spleen
o Lymph node
o Heart
o Bone
o Liver
568-Resolution of mycosis fungoids lesions under electron beam therapy can be expected with:-
o Eczematous lesions
o Plaques,
o Erythroderma,
o Tumors
 All of these
o None of these
569-The second site include in MF after the skin is
o Bone
o Interastinal
o Lymph nodes
570-A pathological feature diagnostic of mycosis fungoids
o Pautrier microabsceses
o Atypical histocyte in the dermis
o Atypical histocyte in the epidermis
o Multinucleated histocyte
o None of the above
1125-Massive periappendigeal lymphocytic infiltrate of lymphocyte. Diagnosis
o DLE
o Lymphocytoma cutis
o Lymphoma
 MF
1201-Old patient with hypo pigmented patches over the back with infiltration since 6 months on
treatment…..recently, he developed multiple papules and vesicules over face and extremites, What should
you give him? ( MF recently developed drug eruption)
o NSAID
 Steroids
o Pencillin
o Tetracycline
o Retinoids

1086-Mycosis fungoides big plaque (satisfyings recurrent erythema, papules no frequent relapsing
dasky erythem cd 30+ve)
 PUVA
o Metcrksat
o Cyclosporin
o Tetracycline

143
1255-Young male with mycosis fungoides, prgnosis:
 Single patch more common than older
o Single patch not respond to electron beam
o Develop sezary syndrome
o Bad prognosis than elderly

1271-Bexarotene, an oral retinoid is indicated in


o Severe acne vulgaris
o Pustular psoriasis
 Mycosis fungoides
o Lichen planus

1382-Pic of buttocks with scally erythematous mild itchy plaques in old female,what is the diagnosis:
 mycosis fungoid
o pytriasis rosea
o taniea cruris
o pustular psoraisis

From the questions in page 82:

29- DOC for early mycosis fungoides- Electron beam therapy > Phototherapy

From the questions in page 83:

144
Miscelaneus
39-Heat receptor can tolerate temperature at what level after that pain
o 42 o C degrees
 45 o C degrees
o 52 o C degrees

868-Patient with knuckle pads on his finger


o Intralesional injections of corticosteroids
o Salicylic acid or urea
o Trauma main cause
o Familial cases observed
o All above

869-About smoking all following true exept


o Smokers are sun protected
o Smoking cause premature aging wrinkles
o Mutch of wrinkeles from infra red smoke effect
o Smoking decrease collagen deposition

910-Axillary papules,itchy since one year (foxfordyce) the question is: what is the best treatment:
 Retinoides

942-`What is accompanying viral disease porphyria


 Hepatitis C
o Hepatitis B

1101-All of the following conditions classically produce lesions involving the soles and palms except:
o Erythemamultiforme
 Chickenpox
o Secondary syphilis
o Hand-foot-and-mouth disease

1105-An elderly man has papular lesions = Onyx and cruston the front faster treatment

1106-A positive nikolsky's sign is typical of skin lesions associated with:


o Toxic shock syndrome
o Herpes zoster
 Toxic epidermal necrolysis
o Contact dermatitis

145
1112-You have received the computed tomography (CT) scan report on a34-year-old mother of three who
had a malignant melanoma removed 3 years ago.Originally, it was a Clerk’s level I and the prognosis was
excellent. The patient came to your office 1 week agocomplaining of chest pain and abdominalpain. A CT
scan of thechest and abdomen revealed metastatic lesions throughout the lungs and the abdomen. She is in
your office, and you have to deliver the bad news of the significant spread of the cancer. The FIRST step
in breaking news is to:
 Dliver the news all in one blow and get it over with as quickly as is humanly possible.
o Fire a „warning shot” that some bad news is coming.
o Find out how must the patientknows
o Find out how much the patientwants to know it.
o Tell the patient not to worry.

1118-8 months child presented by red ,hot ,solitary lesion on the Lt shoulder ,it has smooth surface it's
present since age 2 months ,most appropriate management:
o Oral steroids
 wait and watch
o interferon
o Surgical removal

1124-The question gives a history with the following P. Foliaceus like lesions plus malar erythema
exacerbated by sun. The most beneficial diagnostic procedure:
o Skin biopsy
 Immunoflorescent studies(DIF)
o Electron microscopy

146
Nail
662-The lunula is
o A hypovascular area
o A parakeratotic area
o An area of increased nail plate thickness
o The visible part of the matrix
o Caused by air spaces in the nail plate
663-Which of the following structures contribute substantially to the nail as it appears at its distal free
margin
o Matrix
o Nail bed
o Proximal nail fold
 Hyponychium
o All of the above
664-Black toe nail is produced by:
o Trichophyton rubrum
o Tetracycline
o Trauma
o All above
665-Drug causes nail pigmentation:
o Bleamycin
o Doxorubicin
o Zidovudine
o All of the above
666-These drugs cause nail pigmentation:-
o Chemotherapy (bleomycin)
o Antimalarial
o Antibiotic (minocycline)
o Arsenic poisoning
o Anti-viral (Zidovudine)
o Fungal (T. Rubrum)
 All of the above
667-Black longitunal lines in nail is due to:
o Trauma
o Bacterial
o Viral
o Antimalarial drugs
668-Patient suffer from black nail plate and skin around nail is black, best treatment
o Surgical removal
o Curette
o Biopsy
669-Most cases of fingernail hypertrophy originate with
o Fungal infection
o Psoriasis
o Trauma
o Alopecia areata
o Lichen planus

147
671-In disorders of the nail all except
o Koilonychias suggests B12 folate deficiency
o Onycholysis is associated with psoriasis
o Leuconychia is a feature of severe liver disease
o Splinter hemorrhage is a feature of severe liver disease
o Beau lines disappear faster from fingernails than toenails
672-White nails associated with leuconychia
o Anemia
o T.b
o Niphritis
o Liprosy
o All true
673-Patient with liver cirrhosis all nail have white proximal part while distal part are normal in color
o Onychodystrophy
o Twenty nail syndrome
o Leukonychia
o Terry nail
674-Onychorrhexis is mostly present in:-
o Lichen planus
o Dariers disease
o Alopecia areata
o Psoriasis
675-Ptrygium nails are commonly associated with:-
o Darier's disease
o Psoriasis
o Lichen planus
o Onychomycosis
676-The following are characterizing the nails in Dariers disease:-
o The edge of the nail is showing V:-shaped scalloping
o Subungual hyperkeratosis
o Longitudinal red & white lines
o Keratotic papules
o All of the above
677-Onycholysis occurs with –
o Tetracycline
678-Tetracycline affect nails in the form of:-
o Onycholysis
o Pitting
o Subangular hyperkeratosis
o None of the above
680-Painful nails can be due to all except
o Ingrowing toenail
o Glomous tumor
o Herpetic whitlow
o Periungual fibroma
o Subungual hematoma
681-Most common nail skin malignancy
o Squamous cell carcinoma
o Basal cell carcinoma
o Malignant melanoma

148
1327-Picture of patient with two fingers with destroyed nail. Best treatment is
 Intralesional steroids
1325-Diagnosis
 Kiolonycia
o Onychorrhexis
1326-Cause of Kiolonycia
 iron defficencyanemia

1328-Arrow pointing to the side. what is it:


o Onychomadesis (shedding at beginning)
 Onycholysis (separation at the tip )4 ‫فى عدم وجود‬
o Koilonychia (spoon nails)
 Onychodystrophy(nail destruction in LP)

1329-65 year male with black longitudinal with blurred edges line in index finger, periungual show black
colour, what you advise:
 Excision biopsy
o Colonoscopy
o Reassurance
o Antifunal

149
1330- (picture) nail with redish swelling under the nail bed
 Glomus tumour
o Subungual fibroma
o S.C.C
o Melanoma

1345-Figure of thickened toe nails -ve fungal with skin colered papules in forehead neck chest and
progressive also father and sister has the same:
 Pachyonychia congenita
o Nail psoriasis
o Dermatophyte onychomycosis
o Saprophyte dermatomycosis

1585-The pathogenesis of nail pits in psoriasis is:- punctuate parakeratosis.

1388-Hepatic patient showing all nails with proximal white and distal parts. the patient is positive
hepatitis-B virus but inactive. Which of the following disease can lead to the same picture (terry nail):
 Congestive heart failure
o Retinoid therapy
o Hypothyroidism
o Lichen planus

150
Other Skin Cancers
Most common cancer
A) MALE
1) Incidence---Lung cancer
2)Mortality---Lung cancer
B)FEMALE
1)Incidence----Breast ca
2)Mortality----Breast cancer
C) OVERALL
● Most common incidence and mortality both---Lung cancer
• “Revised Ghent criteria

Painfull tumors LEND AN EGG

-L Leiomyoma -A Angiolipoma -E Endometrioma


-E Eccrine spiradenoma -N Neurolemmoma -G Glomus
-N Neuroma -G Granula cell
-D dermalefibroma

571-The atypical cells in lymphomatoid papulosis stain with which of the following markers?
o S-100
o HMB-45
o CD30
o CD20
o CEA
572-Primary CD30 (Ki-1) anaplastic LCL is characterized by all the following except
o It affects mostly adults
 Draining lymph nodes can be involved in 25% of patients
o It is highly associated with (2,5) translocation
o Spontaneous remission can occur in 25% of patients
o It carries a better prognosis than the systemic form

594-Becker nevus is
o Hyperpigmented hairy epidermal nevus
o Premalignant lesion
o Variant of nevus of Ito
o Collage nevus
o Persistent extensive junctional nevus
595-Nevus arenus in pregnancy
o Is of only cosmetic significance
o Indicates a trophoblastic tumor
o Is quite rare
o Seldom resolves after pregnancy
o Indicates a hepatic disease
596-The most common location for the appearance of the common blue nevus is:-
o Head and neck

151
o Torso
o Hands and feet
o Genitalia
o Buttocks
597-All is true about desmoids tumors EXCEPT
o They are benign fibrous neoplasms
o They arise most commonly from the rectus abdominus in postpartum women
o They are most common in persons aged 10-40
o They are locally aggressive
o They are resistant to radiation therapy
598-A man presented with a nodule on the side of the neck with a fistula histopathology showed hyper
keratosis and papillomatosis and a cratwe with upper portion lined with squamous epithelium and lower
portion lined with apocrine epithelium diagnosis is
o Parotid gland tumor
o Paget's disease
 Trichoblastoma
 Syringocystadenoma papilliferum
o TB nodule
599-Cutaneous horn may arise from
o Filiform wart
o Seborrehic keratosis
o Keratoacanthoma
o All
600-All is true about cutaneous horn EXCEPT
o Usually it occurs in the elderly
o Both sex are equally affected
o SCC can be presented at its base
o It occurs exclusively on sun-exposed areas
o Skin biopsy with adequate specimen depth is obligatory in almost all patients
601-Which malignant sign is itchy
o Lasser-Trelait sign
o Erythema gyratum repens
o Trousseau sign
602-Pruritis is the most common symptom in
o Acanthosis nigricans
o Lasser-Trelait sign
o Trousseau sign
o Hypertrichosis
603-Sudden appearance of numerous itchy Seborrehic keratosis in adult is called:-
o Sign of Lesser-Trele
o Auspitz sign
o Darier's sign
o Nicolesky's sign
604-Malignant acanthosis nigricans is associated with malignancy in:-
o GIT
o CNS
o Kidney
o Lungs
605-What is painful:-
o Eccrine poroma
o Syringoma

152
o Steatocystadenoma papilliferom
o Eccrine spiradenoma
606-The most common cutaneous neoplasm with ossification is:-
o Nevi without inflammation
o Nevi with inflammation
o Basal cell carcinoma
o Chondroid syringoma
o Pilomatrichoma
607-Possible cutaneous manifestations of systemic malignancy include all EXCEPT
o Generalized pruritus
o Acanthosis nigricans
o Late-onset dermatomyositis
o Generalized hyperpigmentation
o Seborrehic dermatitis
608-A 50- year old woman and her mother have multiple cutaneous and uterine leiomyoma. She has an
increased risk for
o Breast carcinoma
o Thyroid carcinoma
o Ovarian carcinoma
o Renal cell carcinoma
o Colon carcinoma
609-Glomous tumor arises from:-
o Nerve cell
o Nerve fiber
o Smooth muscle cell(Glomus body)
o Fibrocyte
610-Granular cell tumor affects mainly:-
o Tongue
o Palms
o Lobule of the ear
611-Treatment of keratoacanthoma
o Excisional biopsy
o Radiotherapy
o Punch biobsy
o Jucisiomol biobsy
612-Face and scalp crusted papules on removal leaves erythematous surface best least invasive medicine
to remove it
o Retinoid
o Cryotherapy
o Surgical
o Currete
o 5FU
613-A constant feature of both actinic keratosis and leukoplakia is
o Epidermal hyperplasia
o Acantholysis
o Parakeratosis
o Atypical epidermal cells
o Spongiosis

153
614-Histopathology of stucco keratosis is most closely resembles
o Verruca vulgaris
o Actinic keratosis
o Lichen planus
o Epidermodysplasia verruciforms
o Dariers disease

620-Which of the following treatments used for heamangioma may cause dysplastic paraplegia
o Interferon alpha
o Prednisolone
o Embolization
o Radiation
621-Involution in haemangioma of infancy is most typically heralded by
o Ulceration
o Color change from bright to dull red
o Increased prominence of telangiectases
o Rapid decrease in size
o Pain
622-Which category best describes a plaque like hemangioma of infancy distributed along an
embryologic mesenchymal prominence?
o Segmental
o Developmental
o Problematic
o Syndromic
o Multifocal
623-An infant with several cutaneous haemangioma of infancy (HOI), tachypnea, and increasing
abdominal girth is likely to demonstrate
o Absent femoral pulses
o Digital clubbing
o Heure-positive stools
o Hepatomegaly
o Splenomegaly
624-Regarding HOI, ulceration is most likely to occur in what phase of growth?
o Involuted
o Involuting
o Proliferating
o Plateau
o Nascent
625- A 7-year-old child presents whith port wine stain of the left cheek and verrucous epidermal nevus on
the leg . What is the diagnosis?
o Phakamatosis pigmentovascularis I
o Phakamatosis pigmentovascularis II
o Phakamatosis pigmentovascularis III
o Phakamatosis pigmentovascularis IV
626-Kazbach merrit
o Haemangioma
627-Best treatment of Angiokeratoma of srotum
o Surgical excision
o Cryosurgery
o Topical treatment

154
628-The most common type of leukemia in adults is:-
o Granulocyctic leukemia
o Chronic lymphocytic leukemia
o Acute myelomonocytic leukemia
o Myelocytic leukemia
629-The most frequent non specific feature of leukemia is:
o Bulla.
o Erythroderma.
 Pruritus.
o Vasculitis.
o Paller.
630-Patient has coin size lesion severe itch subacute erythroderma on neck extrematisis no history of
atopy what diagnosis
o Numeuler eczema
o Cuteneos T cell lemphoma
631-Gingival hyperplasia is characteristic of
o Acute lymphocytic leukemia
o Chronic lymphocytic leukemia
o Acute granulocytic leukemia
o Chronic granulocytic leukemia
 Acute Myeloblastic Leukemia
632-Gingival hyperplasia is most commonly occurs with:-
o Hodgkin's lymphoma
o MF
o Acute myelomonocytic leukemia
o GVHD
633-The most common cutaneous lesion of Hodgkin disease is
o Papules
o Tumors
o Exfoliative dermatitis
o Ichthyosis
o Ulcers
634-A case of pruritus, icthysosis & hyperpigmentation is more likely correlated with:-
o Hodgkin lymphoma
o Mycosis fungoids
o Psoriasis vulgaris
635-Skin disorders that have been associated with MM include all of the following except
o Scleroderma
o Scelromyxedema
o Polyarteritis nodosa
o Subcorneal pustular dermatosis
o Sweet syndrome
858-An old man has rolled on ulcer near the eye, the histopathology showed basiloid cells.best treatment
is
o Wide excision
o Electrocurettage
o Cryotherapy
o Moh's surgery

155
862-Glucagoma disease of
o Alpha cell of pancreas
o Betacell of pancreas
863-Erytherma polycyclic large maculae, a central clearing, the edge increase many mm daily what is
the treatment
o Steriod
o Antibiotic
o Search for associated malignancy
881-Which of the following presents with pruritus as a common feature
o Trousseau's sign
 Leser trelat sign
o Hypertrichosis

864-54 year old patient present with lesion on skin investigation showed TNF high , which diagnosis
o Malignancy
o Flummenent pust.psoriasis
o Genral erthromderma
873-Erythematous lesion mild scaling more than 7mm /d
o Advice to search for malignancy
o Borrelia bordgraferi
o Top corticosteroids only
o Anti histone

906-Dysplastic nevus syndrome (AD)all of following are true except:


 Autosomal recessive

959- 28 male patient with bluish color in thumb finger lesion, with dimpling sign (fibroma), what
treatment:
 Excision
o Leave and follow up

991-Most common tumour with skin metastasis occurs with


o Renal cancer
 Lung cancer
o Prostate cancer
o Laryngeal cancer
o Pharyngeal cancer
992-Carcinoma cutis occur due to
o Cancer prostat
 Cancer stomach
o Canncer liver
993-Basal layer most dominant compotent
 Collagen IV
o Collagen VII
o Laminin 332
o Fibronectin
858-An old man has rolled on ulcer near the eye, the histopathology showed basiloid cells.best treatment
is
o Wide excision

156
o Electrocurettage
o Cryotherapy
o Moh's surgery
908-Ossification occurs mostly with
o Chondoroidsyringoma
o Syringosystadenoma
 Epithlioma of malhebre(pliomatrixoma)
o Trichoepithelioma

1004-The presence of TNF alpha indicates:


 Malignancy

1012-A 70 -year- old patient presented with a skin lesion in the left thigh for many years.this lesion is
black, size 1x1cm. It started to be more pigmented with bleeding. You will advice :
o Cryotherapy
o Incisional biopsy
 Wide excision

1015-Patient develops wrinkled skin with putting hands in water what is the disease?
o Larva migrans
o Asthma-eczema syndrome
o Sarcidosis
 Cystic fibrosis

1019-Painful tumor? Lend an egg


 Leomyoma
 Eccrinespirademafibroma
 Neuroma
 Dermatofibroma
 Angiolipoma
 Neurolemmoma
 Enomatroma
 Glomustumor
 Granularcellcelltumer

1055-When metastases to the skin occur from a thyroid malignancy, they are usually due to:
o Medullary carcinoma
 Papillary adenocarcinoma
o Follicular carcinoma
o Anaplastic carcinoma

1089-What's more place to keratoacantoma


o Forearms
 Scalp
o Legs
o Palms

157
1090-What treatment fastest and least disruptive of the photosensitizer keratosis
o 5 floriorassel
 Azot((99%cure rate))
o Electriccoagulation
o Alritinoitdattopical

1096-Internal malignancies EXCEPT


o Hypertrichosis
o Inflammation of the skin and muscle
o Bazaks syndrome
 Halobo

1111-Malignancy which most metastasize to skin


 Lung tumor

1146-Mass in abdomen, solitary, violaceous, and by biopsy showing spindle cells.(leiomyoma) What is
WRONG:
o Nifedipine improve the symptoms
o Actin and desmin
o Surgery is the treatment
 Reassurance
1146- 30y old women suffer from painfull gradually increase in size erythematous plaque.h/p atypical
spindle shape cells & occasionally CD34 +ve wt is true:(DSFP)
o Simple excision is the treatment of choice
o Metastasis commonly occur to regional lymph nodes
o Radiotherapy does not add in reducing the recurrenc
o CD44 help to confirm the diagnosis
1147-Patient with painful erythematous subcautanous nodule on trunk H/P (dermatofibroma) what is the
following statement incorrect:
o Treatment is surgical excision
o Nifdipine is symptomatic relief
o +ve actin & desmin
 Focal nuclear atypia is suggestive for malignant transformation
1170-Dermatofibroma:
o Haphazard spindle cells in fascites
o Haphazard collagen bundles
o Fibroblast in myxoid vascular stroma
 Mucin between collagen bundle

1184-Lesion on medial canthus (description of BCC) The question is: what is the best treatment
Mohs Syrgery

158
1194-A case of melanoma in face5x7cm after surgical removal develop dog ear scar, which is not the
TTT of this scar:
o Tensile removal scar
o Remove excess skin
o M plasty
 T plasty
1213-What is best treated with PDT
 Superficial basal cell carcinoma
o Keratoacanthoma
o SCC
o BCC

1218- Male patient has circular bluish discoloration on the nail of left thumb which is very painful in cold
weather and when touch with the tip of the pin cause pain, the treatment (Glomus Tumor):
 Excisiton
o Cryo
o Nifidpine

1239-The most painful tumor is


 Eccrine spiradema

1245-23 years old with brown red nodules in the face 1 year ago no nerve affection and no any other
lesions in the hole body, Grenz zone & lymphocyte in histopathology. What is the diagnosis:
o Cutaneous leishmaniasis
o Verruca vulgaris
 Lymphocytoma cutis

1250-Nail cyanotic circular area on exposure to cold, very painful on touch with tip of pen (Glomus
tumor) how to deal:
 Exision
o Nifidipine
o Cryosurgery
o Sclerotherapy

1284-A paient suffer from flushing which becomes wrose with alchol,dyspnea and attacks of diarrhea, CT
scan shows affection of the liver, which investgation should be done for diagnosis(carcinoid syndrome) :
o Serum dopamine
o ANCA
o Serum sertonin
 Urinary 5-hydroxy indole acetic acid

1309-Frontal bossing
 Gorlin syndrome(nevoid basal cell carcinoma syndrom)

159
1337-Histology of 5 y nodule near the nose 1cm with rolled edge :
 BCC
o SCC
o Melanoma

1338-BCC pathology

1339-A picture of an old man has nodule near the eye, since 1.5 year ago now start to ulcerate what is
probable diagnosis:
 BCC
o SCC
o Eroded Seborrheic keratosis
o Malignant melanoma

1340-Scourge of the nose 2.5cm 3 weeks ago with a picture of the anatomy patients diagnosis
 Keratoacantoma

1347Figure of pin point skin colored papules under both eyelids diagnosis:
 Syringoma
o Follicular palioma

160
1359-Photo of scrotum brownish papules (angiokeratoma) what is the best treatment:
 Liquid nitrogen

1386-A patient suffers from papluvesicular lesions on face with sesonal varaitions (increase in summer),
with biobsy there is clear fluid & this hisopathological picture (eccrine Hydrocystoma), diagnosis:
 Uniocular cyst lined by flat cells
o Tadpole
o Eccrine duct blockage &inflamation
o Squamous epithelium &contain clear secretion

1570-The most common site for keratoacanthoma is:- sun exposed area (Elderly)

1577-Carcinoma cutis occur due to:- cancer stomach.

1640-Match the primary tumors with the relative frequencies of cutaneous metastasis in women:-
Carcinoma of breast, A
Carcinoma of colon, B
Melanoma C
Carcinoma of the ovary. D

1643-Painfull tumors LEND AN EGG

-L Leiomyoma -A Angiolipoma -E Endometrioma


-E Eccrine spiradenoma -N Neurolemmoma -G Glomus
-N Neuroma -G Granula cell
-D dermalefibroma

161
From the questions in page 75:

85-Lentigo simplex/café au lait patches :clinical differential is NOT possible


86-Keratoacanthoma: sun exposed site (most commonly)
93-Pilomatrixoma: most common tumor to ossificate
94-Hodjkin’s pruritus, ichthyosis, hyperpigmentation (common skin manifestations)
95-Histtiocytosis x: Bone (the commonest organ affected)
96-Liposarcoma:(the most common malignant tumor)
97-Lung cancer: cutaneous metasteses (most common cause)
98-Painful tumors: BLEND AN EGG.

From the questions in page 81:

172-Howel-Evans syndrome: Palmoplantar keratoderma+ cancer esophagus


173-Erythroderma: lymphoma-leukemia (25%of cases)

From the questions in page 83 :

98-Most common type of BCC- noduloulcerative

162
Pigments Diseases
97-To complete depigmentation, the drug of choice is:-
o 0.1% methoxsalen
o 0.6 mg/kg trioxalen
 20% monobenzone
o 0.1% monobenzone

98-Hydroquinone compounds cause depigmentation by their action on


o Melanin
o Dopa
o Tyrosine
o Tyrosinase
o Melanocytes

708-Which of the following statements is wrong about vitiligo:-


 It can be caused by contact with bleaching agents
o Increase incidence of organ specific autoantibobies in serum
o Association of pigmentary disorders of ocular area
o Pigmentation is first seen perifollicular after starting treatment
709-Which is false about vitiligo:-
o Rarely progress to involve whole body
 Depigmentation of hair present in vitiligo patch indicates good response to treatment
o Infant is not born with it but develops it later while growing up
710-Vitiligo may be associated with all except:-
o Addisons disease
o Thyrotoxicosis
o D.M
 Tinea versicolor
711-Most common presentation of vitiligo
o Acrofacial
o Segmental
 Focal
o Universal
712-The following disorders are associated clinically with vitiligo all except
o Addison disease
o Malignant melanoma
o Pernicious anemia
o Acromegaly
o Diabetes mellitus
713-8y child with upper eye lid hypopigmented macule best treatment
o Corticosteroid
o Tacrolimus
o Retinoid
o Topical PUVA
714-Resistant Vitiligo treatment
o Spickle skin graft
o Cultured melanocyte
o ACTH

163
o Surgical
715-In vitiligo, which of the following sites is usually the first to respond to PUVA treatment
o Face
o Umbilicus
o Nipples
o Perineum
o Hands and feet
716-The chief hazard of topical therapy in vitiligo is:-
 Phototoxic reaction
o Chills & fever
o Gastric disturbances
o All of the above
717-Halo nevus associated with:
o Vitiligo
o Melanoma
o Melanocytic navi
718-Percentage of association of halo nevus with vitilligo is:
o 50%
o 2%
o 10%
 20%
719-Halo nevus present with
 Nevocellular nevus
o Blue nevus
o Neurofibroma
720-Halo nevus is most commonly seen around
o Blue nevus
 Spiz nevus
721-Halo nevus
o Usually appears soon after birth
o Is inherited as an AD trait
o Usually occurs as a single lesion
o In histology, it is a compound nevus with lymphocytic infiltrate admixed with intradermal nevus cells
o Need surgical excision in young patients
722-Piebaldism results from mutations in which of the following genes?
o Hup2
o Kit gene
o PTEN
o Nemo/ikky
o RET
723-Tyrosinase enzyme deficiency results in:-
o Vitilligo
o Tinea versicolor
o Albinism
o Hyperpigmentation
724-Child has white forelock, hair porest, developed wide area of pigmentation on chest abdomen, and
intestinal hyperpigmented macule best treatment
o PUVA
o UVB
o Excimer laser

164
o Topical steroid
725-Patient with hyper pigmented patches due to competitive reaction with tyrosinas enzyme caused by
o Vitiligo
o Piebaldism
o Pityriasis versicolor
726-Hypomelanosis or depigmentation that result from complete inhibition of tyrosinase is:
 Pityriasis versicolor
o Vitiligo
o Nevus of Ito
o Piebaldism
727-Non-hodgkin lymphoma can be associated with:
o EBV-infection
o HCV-infection
o HIV-infection
o H.pylori infection
 All of the above
o Non of the above
728-Which of the following therapies can lead to further hyperpigmentation in melasma some groups of
patients
o Broad spectrum sunscreens
o Kligman formula
o Topical azelaic acid
 Chemical peels
o Steroid
729-Phenolic germicides cause cutaneous depigmentation by
o Inhibition of tyrosine formation
 Inhibition of tyrosine oxidation
o Inhibition of melanin oxidation
o Inhibition of cytosine action
o Destruction of all melanocytes

947-Old man had pigmented scaly lesion on the face what is the best topical treatment
Imiquimod

960-Best area to cure from vitiligo by PUVA:


o Hand and foot
 Face
o Perineium
o Nipple

1006-Child born with area of hypo and depigmention and his parent has poliosis best treatment for
hypopigmentation areas
o Eximer laser
 Culture autologous melanocytes
o Puva

165
1092-Pigmentations like rain drops on the trunk with hyperkeratosis on palm and plantar
 Arsenic
o Adison
o PUVA

1104-Hyperpigmented pityriasis alba is


o It may be related to lichenoid pigmentosus
o Pityriasis alba with pigmented contact dermatitis
 Variant of classic pityriasis alba showing a strong association with dermatophyte infection
o Post inflammatory hyperpigmented pityriasis alba
1187-32 old female was treated last year from vitilligo by puva she now has residual lesion in her face
..best tretment is:
o Puva
o Excimer
 Autologus transplantation of melanocytes

1265-Patient type 4 skin did a phenol peeling and didn't use sunscreen , will suffer from:
o Persistent erythema
 Post inflammatory hyperpigmentation
o Post inflammatory hypopigmentation
o Acneform drug eruptions

1274-halo navus in hp
infiltrate dicloses navus
lymphocte in blood
if halo nevus disappeared clinically in histologic nests of nevus cells will remain

1317-vogt-koyanagi-harada syndrome
 Retnal detachment +(alopecia,poliosis,vitiligo)
1318-Pregnant female 8th month with pigmented lesion on face (melasma) the TRUE is
o Recurrence decrease with subsequent pregnancy
o May affect maxillary, forehead and mandible
o Affect 10% of female
o Not occur with contraceptive pills
o More common in men than in women

1341-Viltigo (picture) what type of laser used in treatment of this condition


 Eximre
o Alexandrite
o PDL
o Nd - yag

166
1342-A 8-years child presented with leukotrichia , forehead and upper eyelid hypopigmented macule.
Best topical treatment is(vitiligo ):
o Corticosteroid
 Topical tacrolimus
o Retinoid
o Topical PUVA

1366-A case of hyperpigmentation in the left cheek and present in the left sclera (nevus of Ota )is treated
by
 QS Nd:Yag
o PDL
o IPL
o Excimer

1367-Picture of chin (in my opinion it was normal) Patient complain of this pigmentation with lat side of
sclera have dark pigmentation What to do to help him
 Q switched ND YAG
o Long pulsed ruby
o Intense pulsed light

1375-Diagnosis;
 Becker nevus

167
1376-becker's navus discription on back

1378-Picture of hyperpigmentation in all the body , palm crease , hypopigmentation and weakness, the
diagnosis:
 Addison

1423-The darker the skin complexion in blacks, the highest the prevalence of acral pigmented lesions:-
o True
o False

1546-In most cases, halo nevi are:- compound nevi.


Match
1638-The diseases with the enzymes in which there are defects:-
 Phenylketonuria phenylalanine hydroxylase,
 Albinism tyrosnase,
 Ochronosis homogentistic acid oxidase,
 Homocystinuria cystathione synthetase.

From the questions in page 79 :

56-Melanocyte number ↓↓with age

57-Peutz-jegher syndrome-jejumum ileum (commonly affected sites)


58-Mongolian spots :sacrum (most common site)
59-Neurofibromatosis:6 CAM 5mm, prepubertal, major criterion
60-Vitiligo: face (best ttt response)

168
61-Hypopigmentation :Pityriasis versicolor (tyrosine, competitive inhibition)
62-Hypopigmentation: cryotherapy (commonest complication)

From the questions in page 83:

100-Most common cause of hypopigmented, nonscaly, atrophic patches on cheek of endemic


area children- indeterminate hansens

From the questions in page 84 :

105- commonest autoimmune association in vitiligo- thyroid


106- Commonest agent for leucoderma- paratertiary butyl phenol (PTBP)
112- Commonest vitiligo- Vitiligo vulgaris

169
Pregnancy Dermatoses

829-What is true about PUPPP:-


o Occurs in the 1st trimester
o Neutrophils are found around blood vessels
o Occurs in the 3rd trimester
830-PUPP (pruritic urticarial papules and plaques of pregnancy)the best treatment
o Topical corticosteroids &anti histamine
o Mtrx
o Colshecin
o Oralcs

870-What is of fallowing least probable to cause prematurity of birth in the affected pregnant ladies:
o Anetadema
o Cutislaxa
o Psoriasis 1st week of life
o Mother scc 20 years ago

1021-Common warts what is the suitable ttt and safe to pregnancy


o Podphyllin
 Electrocautery
o Surgical excision
Salicylic acid applicati

1128-Fox fordyce disease


 Improve with pregnancy

1129-Pregnant female developed papulovesicular rash in face then all body before
delivery the infant develop the same rash after 3 days:
 Chicken pox
o Infectious mononucleosis
o Measles

1150-HG occurs when


o Parturition
 2nd trimester
o 3rd trimester
o Post partum
1190-Pregnant female in the 2nd trimester with vesiculobullous lesion and recurrent (HG) treatment
with
o Systemic steroid
 Topical steroid
o Cyclosporine

170
1192-A 23 yr old pregnant diabetic female comes with lake of pus lesions on skin (pustular psoriasis).
Treatment is
o Methotrexate
o Azathioprine
o Retinoids
 Cyclosporine

1198-What is pregnancy category X .


 Isotreinoin

1202Acitritin not used in :


o Immunosupression
 Pregnancy

1446-Straie gravidarum is present on top of papules of L.P


o True
o False

1500-H.G may occur with the 1st menstrual cycle


o True
o False
1501-H.G occurs in the 3rd trimester
o True
o False

1608-Female pregnant has tens bulae and vesicles started from umbilicus to all abdomen which recure
every pregnance best treatment is (HG):- Corticosteroid

1609-Female pregnant has tense bullas and vesicles started on the stria sparing the umbilicus best
treatment:- antihistaminics and tropical steroid

171
Psoriasis & Papulosquamous Diseases

42-Pin point bleeding due to thinning of the epidermis is called:-


o Nicolesky's sign
o Pathergy reaction
o Auspitz sign
o Koebner phenomenon
43-Auspitz sign is caused by
o Test tube rete ridges
o Thinning of the epidermis overlying the rete ridges
44-Auspitz sign seen in:-
o Pityriasis rosea
o LP
 Psoriasis
o Allergic contact dermatitis
45-Sign that occurs by applying force to cause shearing of the epidermis from the dermis is
o Darier's sign
 Nickolsky's sign
o Pathergy reaction
o Auspitz sign

46-Appearance of the lesion at the same site of the 1ry lesion in some dermatosis on trauma, scratching or
on wound scar or sun burn is called:-
o Pathergy reaction
o Auspitz sign
o Nicolesky's sign
o Koebner phenomenon
47-Koebner phenomenon occurs in all EXCEPT:-
 Pityriasis rosea
o Pityriasis rubra pilaris
o Wart
o Psoraisiform dermatitis
48-.Kobner’sphenomenon seen in EXCEPT
oPsoriasis ,
oLp,
oLichen nitidus
 •Bullous Pemphigoid and keratosis follicularis

53-Epidermal infiltration by polymorph nuclear lymphocyte


o Psoriasis
o Lichen planus
o PRP

242-Psoriasis have abscent of


o Granular cell layer
o Malpgian layer
o Basal layer

172
243-What is not present in psoriasis. :
o Ospitz sign
o Degenerated DEJ
o Parakeratosis
244-One of the following is not a typical histopathological feature & psoriasis:-
o Acanthosis & elongation of rete ridges
o Parakeratotic horny layer
o Thinning of the suprapapillary regions of stratum corneum
o Degeneration of the dermoepidermal junction
245-The hispatology of psoriasis is the same as that of eczema except
o Microabcess in psoriasis
246-HLA in psoriasis
o HLA B12, B17
o HLA B37
 HLA CW6
o HLA B7
247-Cell cycle in psoriasis duration is
o 2 hrs
o 20 hrs
o 37.5 hrs
o 200 hrs
248-Precipitating factors for psoriasis all EXCEPT:-
o Drugs as B. Blockers & lithium
o Stress
o Trauma
 Egg
249-The most characteristic physical sign of a psoriatic nail is
o Pitting
o Onycholysis
o Onychomycosis
o Subungual hyperkeratosis
o Greenish-yellow discoloration
250-The most common manifestation of psoriasis in nails
o Pitting
251-Oil spot in psoriasis occur on
o Nail bed
o Nail matrix
o Nail plate
o Nail fold
252-Pustular psoriasis given isotretinon what most side effect
o Xerosis
o Photosencetivty
o Cushing
o Chest toxicity
253-100 patient psoriasis put under investigation trials 2years and 2years acteritin treatment doctors taken
a liver biopsy from these patients 2year before and after 2 year what you expect
 No correlates acctretin effect patient liver
o Results correlates toinotive of psoriatic disease
o Deterioration of liver pervious disease
o No histological improvement in biopsies of liver associated disease psoriasis

173
254-Case of psoriasis in children treatment with narrow band UV effectiveness increased in combination
with
o Glucocorticoids
o Cyclsporin
o Tacrolimos
o Nothing
255-The range of concentrations of anthralin used topically for psoriasis in the "Ingram regimen" is
o 0.01% to 0.25%
o 0.1% to 2%
o 0.2% to 0.5%
o 3% to %
o 5% to 15%
256-Patient developed erytheroderma and has an elevated liver enzyme best treatment
o Isotretinoin
o MTX
 Corticosteroid
o Cyclosporine
257-Methotrexate therapy of psoriasis patient is contraindicated in:-
o D.M
o Liver cirrhosis
o Pulmonary disease
o Pustular psoriasis
258-Patient with severe psoriasis and have severe hypertension which type of drug he use it
o Methtraxate
o Cyclosporine A
o Cyclophosphamide
259-Psoriatic patient developed hypertrichosis which treatment do that
o Cyclophosamide
o Cyclosporine A
o Methtraxate
o Retinoid
260-The most suitable among the new biologic therapies for treatment of psoriatic arthritis with stable
psoriasis is
o Infliximab
o Etanercept
o Adalimumab
o Alefacept
o Efalizumab
261-Histopathology of lichen planus:-
o Orthokeratosis
o Saw tooth pattern of hyperplasia
o Band like infiltration of superficial dermis
o Civatte bodies
o All of above
262-Hypertrophic lichen planus most commonly on
o Lower lip
o Anterior lower leg
o Buccal mucosa
o Periungual finger and toes
o Scalp
263-Hypertrophic lichen planus associated with

174
 Sporadic lichen planus
o Atrophic lichen planus
o Familial lichen planus
264-Violaceous hyperpigmented plaque with wary surface on legs associated with flat topped papules on
wrists & forearms the diagnosis is:-
o Viral warts
o Kaposi sarcoma
o Hypertrophic lichen planus
o Nodular amyloidosis
265-Most choice of LP treatment is:
o Aspirin
o Methotrexate
 None of above
266-The following statements about lichen nitidus are true all except
o An association with lichen planus is not uncommon
o Most cases occur in children or young age
o Histopathology shows hyperkeratosis, focal hypergranulosis, irregular acanthosis with liqufication
degeneration of basal cell layer
o A self-limiting dermatosis
o Systemic steroids might be helpful in widespread lesions
267-Lichen straiatus:-
o Self-limited disease
o More above age of 20
o Variant of nevus lateralis
o Always limited to head and neck
268-Lichenoid drug eruption caused by:-
o Gold
o Hydrochlorothiazide
o Penicillamine
o Captopril
o Quinidine
o Antimalarial
 All of above
269-About Pityriasis rosea all true EXCEPT:-
o Self-limiting
 Never caused by drugs
o Generalized
o Symptomatic treatment
270-About Pityriasis rosea all true EXCEPT:-
 Worsen by sun light
o Begins with herald patch
o May be due to viral infection
o Self-limiting
271-A safe and effective therapy for pityriasis lichenoid chronic is:-
o PUVA
o Penicillin
o Vitamin E
o Reserpine
o Bleomycin

175
272-Best or most treatment of pityriasis lichenoid chronica
o Phototherapy
o Corticosteroid
o Cytotoxic
273-Treatment of PLEVA of child 10 year
o Erythromycin
o Tetracycline
o Puva
274-6 years old child with 90% of his body shows erythema with scales and there is severe yellowish
scales on his palm and soles with follicular hyperkeratosis on his interphalenges what is the best ttt: PRP
o Systemic retinoid
o NB-UVB
o Systemic steroids
275-Anteriolatral of thigh + anteriolatral of arm child show follicular hyperkeratosis papules resolve no
decrease of vit. (PRP) What best ttt:
o Vit A
o Codliver oil
o Vit C
o Tropical retinoid

879-The most itchy of lichen planus is:


o Atrophic.
 Hypertrophic
o Annular
885-Histopathology of parakeratosis shows
 There is nuclear in keratin cell (coronoid layer)

913-Turn over time definition


 Epidermal renewal time in normal is 14(cell cycle)+14(epidermal transit time)+14(desquamation)=39-
40 day while in ps.=5days only

928-Generalized psoriasis in alcoholic and deactive hepatitis B with lab upper normal SGPT and SGOT
and high creatinine TTT :
o Acitritin
o Methotrexate
 Prednisolone
o Cyclosporine
o Cyclophosphamide
929-Patient with plaque psoriasis since 10 years and developed diffuse erythema & exfolliation with
history of alcohol intake since 10 years & +ve HBV and laboratory tests normal values of liver enzymes
&elevated creatinine level, what is the treatment
 Prednislone
o Cyclosporine
o MTX
o Acitretin
930-The only definite indication for giving systemic corticosteroids in pustular psoriasis is:
o Psoriatic enythroderma with pregnancy
o Psoriasis in a patient with alcoholic cirrhosis
o Moderate arthritis
 Extensive lesions

176
936-Cause of bleeding point when removing scale in psoriasis:
 Thining of epidermis over dermal papillae
o Munro microabsess
o Test tube shape over rete ridge
o Absence of granularlayer

967-Psoriasis may involve


 Extr auditary meatus

1027-Ante cubital fossa


 Not affected in psoriasis
 Affected in s.derm

1070-Someone with psoriasis treated with methotrexate became cumulative dose of 1680 never being
biopsy what is the procedure for the detection of cirrhosis live check the level of liver enzymes has
o The level of liver enzymes
o Ultrasound
 Prcollagen peptide III (pn111n)
o CT scan

1091-Anti TNF alfa is used to treat psoriasis in each of the following EXCEPT
o Psoriaticery throderma
o Intractable
o Pustular
 Pregnant
o Guttate

2019-Vulvar syndrome (vulvovaginitis) vulvo vaginal syndrome caused by candida


The vulvo vaginal- gingival syndrome is occurring in erosive
 Lichen planus

1110-Grover disease characteristic feature


o Papules
 Papulovesicles
o Pustules

1133-Oral lichenoid eruption


o Early patch test for mercury in most pt
o Late patch test for gold in most pt
o Patch test for nickel oxide
 Removal of amalgam

1172-IFN inhibitors in psoriasis least indicated in


o Fulminant pustular psoriasis
o Sever erythrodermic psoriasis
o Psoriasis with pregnancy
o Guttate psoriasis

177
1173-Vulvovaginal gingival syndrome
 Lichen planus
o Behcet

1192-A 23 yr old pregnant diabetic female comes with lake of pus lesions on skin (pustular psoriasis).
Treatment is
o Methotrexate
o Azathioprine
o Retinoids
 Cyclosporine

1208-A case of psoriasis more than 35 % of his skin with no improvement with PUVA the next step is
 Acitrtin

1225-TTT of pityriasis lichenoides chronica:


o PUVA
o Topical tacrolimus
o Oral retinoids
o Topical retinoid

1234-The least disease associated with psoriasis


o Atherosclerosis
o Vitilligo
o Non skin melanoma
 Atopic dermatitis

1237-Case of psoriasis in children treatment with narrow band UV effectiveness increased in combination
with
o Glucocorticoids
o Cyclsporin
o Tacrolimos
 Nothing

1251-Patient with intense pruritis on genitalia since 4 months, lesions on vulva are glistering. HP showed
atrophy and hyalinization DIAGNOSIS :
o Lichen planus
o Morphea
 Lichen sclerosus et atrophicus

1251-Patient with intense pruritis on genitalia since 4 months, lesions on vulva are glistering. HP showed
atrophy and hyalinization DIAGNOSIS :
o Lichen planus
o Morphea
 Lichen sclerosus et atrophicus

178
1348-Picture of erythematous patch with vesicles, female present with face, axilla, nape of neck, -ve
immunofluorescence, histopath acantholysis without dyskeratosis ,what is diagnosis:
o Darier
 Hailley hailley
o Paraneoplastic pemphigus
o Pemphigus folacoeus

1361-A picture of a pregnant patient with plaque psoriasis what treatment


o Acetretin
o PUVA
o Cyclosporin
o Systemic steroid
o Tar

1362-33Y old man suffer from many years from like this lesion on the back.neck & groin erythematous
plaque bulla & erosin –ve direct immuonoflurosence
o Pemphegus valgaris
o Pemphegus folicaous
o Darier disease
 Haily-haily disease

179
1394-Lichen Planopilaris
• Presents commonly with
alopecic patches with
perifollicular erythema;
± tenderness or pruritus
• Histology: lichenoid
lymphocytes around affected
follicles with vacuolar
degeneration within affected
infundibula, perifollicular fi brosis
• Treatment: antimalarial, topical/intralesional corticosteroid

1412-oral methotrexate treatment of psoriasis liver biopsy is not indicated in stable liver:-
o True
o False
1425-Lichen planus exhibits the isomorphic (Koebner) phenomenon:-
o True
o False
1426-Lichen straiatus is self-limiting leaving linear hypopigmentation without known cause-
o True
o False

1445-Pityriasis rosea is contagious


o True
o False

1446-Straie gravidarum is present on top of papules of L.P


o True
o False

1448-L.P lesion in oral cavity is violaceous in color


o True
o False
1449-L.P lesions in oral cavity are pruritic
o True
o False
1450-Nail involvement percentage in L.P is 10-15%
o True
o False
1451-Glans penis is the most affected area of genital tract in L.P
o True
o False
1452-Flexural psoriasis is called inverse psoriasis
o True
o False
1453-The commonest nail change in psoriasis is onycholysis
o True
o False
1454-Anogenital involvement in psoriasis is common
o True
o False

180
1455-Volar psoriasis is caused by staphylococcus
o True
o False
1456-Psoriasis is extremely pruritic
o True
o False
1457-Pustular psoriasis is caused by staphylococcus aureus
o True
o False
1458-Psoriasis of scalp cause scarring alopecia
o True
o False

1484-Antibiotics are proved to be used in Seborrehic dermatitis


o True
o False
1485-In Seborrehic dermatitis, the preauricular greasy scales is the 1st symptom
o True
o False

1487-In Seborrehic dermatitis, palms & soles may be involved


o True
o False

1489-In Seborrehic dermatitis, palms & soles may be involved


o True
o False

1540-The effectiveness of Methotrexate in psoriasis is apparently due to its effect on what


cellular phase:- S.

1628-Dose of 8MOP in patent 70 kg before PUVA:- 70 X 0.4 :- 28 mg.

From the questions in page 79 :

99-Psoriasis: Epidermal cell turnover 3-4 days (Normal 26-28 days)

100-Psoriasis:HLA-B13,17

101-Psoriatic arthropathy affect:- 5% of psoriatic patient.

102-Psoriasis: pustuar (Dapsone ttt of choice)

103-Psoriasis: nail pitting (most common sign)

104-Psoriasis: pin-point hemorrhage (due to suprappaillary thinning)

105-Psoriasis: Acanthosis of rete ridges is NOT present

106-Psoriasis: Granular cell layer NOT present

181
107-Psoriasis: Hyperkeratosis NOT present

108-Psoriasis: Immune complexes are NOT present

109-Psoriatic arthropathy affect:- 5% of psoriatic patient.

110-Lichen planus: Hepatitis C is correlated

111-Lichen planus: Hpertrophic (most itch)

112-Lichen planus: Lichenification no sweating

113-Lichen planus; nail longitudinal lines

114-Lichen planus,mucosal,parakeratosis(absent granular layer)

115-Lichen planus: Subepidermal igm(DIF)

116-Lichen planus: Levamisole, Methotrexate (ttt of choice)

From the questions in page 82:

23) Most common Psoriatic arthritis- oligoarticular, asymmetric.


24) DOC for psoriatic arthritis- Methotrexate
25) DOC for arthritis mutilans- etanercept
26) DOC for guttate ps- antibiotics
27) DOC for erythrodermic psoriasis- Methotrexate
28) DOC for pustular psoarisis- Acitretin
33) Most classical joint involved in Psoriatic Arthritis- DIP

From the questions in page 83:

95- Most severe form of psoriasis- Von zumbusch

182
SCC
615-Characteristic features of bowen disease include all EXCEPT
o An uncommon condition occurring mainly in the middle and old age
o Can occur on top of porokeratosis and epidermodysplasia verruciforms caused by HPV-5
o Ulceration of lesion often is a sign of invasive growth
o Transformation into SCC occurs in 20% -30% of cases
o Early treatment with PDT using topical 5-ALA yields high cure rate with excellent cosmetic results
616-What is not predisposing SCC
o Old scar
o Sunexposed
o Actinic keratosis
617-SCC the least location
o Scar
o Trunk
618-Pre SCC condition is :
o Erythroplasia of queyrat
619-SCC is most likely to metastasis from:
o Tragus of the ear
o Upper lip
o Side of the nose
o Lower lip
896-SCC discreption:
 Pearl horns in histology early
o Grade 3 the most undiffentiated according to tumor Grading
o Adnoid type is the worst
897-Clinical and histological picture of squamous cell carcinoma in face what is true regarding this type
of tumor?
o Adenoid type is the worst type
 Horn pearl is present in well differniated tumor
o Lesion in ear shows more metastases
1260-Commonest malignancy in nail plate:
 SCC
1336-Clinical and histological picture of squamous cell carcinoma in face what is true regarding this type
of tumor?
o Adenoid type is the worst type
 Horn pearl is present in well differniated tumor
o Lesion in ear shows more metastases

183
From the questions in page 79:

87-Squamous cell carcinoma: old scar least metastic risk


89-Squamous cell carcinoma modified skin e.g. Glans (highest metastatic risk)
90-Squamous cell carcinoma: immunosuppressed 18 folds higher risk
91-Squamous cell carcinoma 7th decade peak incidence
92-Squamous cell carcinoma the condition is most likely to evanuate: ertythroplasia of Queyrat,
93-Squamous cell carcinoma a cutaneous viral infection which may evanuate: epidermodysplasia
verruciformis.

184
STDs(Sexualy transmitted Diseses)
Mycoplasma Lymphogranuloma Granuloma inguinale Chancroid
venereum
In ♂ 1ry stage Serpiginous Genital ulcer
 Urethritis  Ulcer granuloma  (painful ulcer)
 Epididymitis  Urithritis  (painless ulcer)
In ♀  Proctitis Genital
 Bacterial vaginosis Pseudobubos discharge
 Pelvic inflammatory 2ry stage  (painless LN)
disease  Lymphadenitis
In Both  Inguinal bubos, Scarring and Genital LN
 Reiter syndrome  Groove's sing mutilation  (painful LN)
In Infants
 Pneumonia 3ry stage
 Fever  Genital elephantiasis
 Respiratory distress  Anorectal strictures &
syndrome fistulas

377-Causes of Saddle nose


o Lepromatous leprosy
o Wegner’s granulomatosis
o Congenital syphilis
o All of above
378-Saddle nose deformity is found in the following EXCEPT:
o Secondary syphilis
o Multicentric reticulohistiocytosis
o Relapsing polychondritis
o Leprosy
379-Stds all except:-
o Penile molluscum contagiosum
o Genital wart
o Herpes simplex genitalis
 Scrotal herpes zoster
380-Rieter syndrome treatment :
o NSAID
 Systemic steroid
o Methotrexate
o Dapson
o Colchisin
o Thalidomid
381-Treatment of Rieter in children
o Tetracycline
o Azithromycin
o Ketoconazole
382-The most infectious stage of syphilis is:-
o 1ry syphilis
 2ry syphilis

185
o Early congenital syphilis
o Late congenital syphilis
o 3ry syphilis
383-Patient have painless ulcer in cronal sulcus since 2 weeks what is diagnosis
o Syphilis
o HSV
o Chancroid
o T.B
384-Syphilis:-
o Iry stage characterized by painful ulcers on genitalia
 2ry stage typically affect palm & soles
o 3ry stage is characterized by extensive scarring
385-Oral lesion in 2ry syphilis is in the form of:-
o Mucous patch
o Erythema
o Vesicles
386-Transmission of syphilis to the fetus mother occurs:-
o During 1st trimester
o During 2nd trimester
o During 3rd trimester
o During labor
387-Early congenital syphilis manifestation appear during:-
o 1st week of life
o 2nd week of life
o 6th week of life
388-Early sign in congenital syphilis is –
o Syphilitic rhinitis
o Syphilitic rash on palms & soles
o Condyloma lata
o Genital chancre
389-The MOST common manifestation of congenital syphilis is:-
o Fissures of the lips and anus
 Copper red macules and papules of the palms, soles and diaper area
o Mucous patches
o Condyloma lata
o Syphilitic pemphigus
390-The least common cutaneous manifestation of early congenital syphilis is:-
o Rhagada
o Condyloma lata
o Rhinitis
o Rash in palms and sole
391-Which condition is not characteristic of early congenital syphilis
o Weinberger sign
o Jaquet erythema
o Syphilitic dactylitis
o Pseudoparalysis of parrot
o Condyloma lata
392-The first serological test for syphilis to become positive in primary syphilis is usually:-
o TPHA
o FTA
o VDRL

186
o Wassermann reaction
o T. Pallidum complement fixation test
393-VDRL is usually +ve in
o 1ry S
o 2ry S
o 3ry S
o Latent
394-The earliest +ve test syphilis is:-
o FTA-ABS
o RPR
o TPHA
o All of the above
395-Patient has asymptomatic papule & plaque on trunk back sole a generalized lymphoadenompathy
after taken treatment. Which antibiotic he takes.
 Pencillin
o Erthromycin
o Clindamycin
o Dapson
396-Clues to the diagnosis of late syphilis include
o Chancre scar
o Nuchal leukoderma
o Macular atrophy
o All of the above
o None of the above
397-The drug of choice for non-venereal treponematose is
o Penicillin G procaine
o Penicillin G benzathine
o Erythromycin
o Chloramphenicol
o Tetracycline
398-Penicillin:-
o Is the only effective treatment for syphilis
o Is effective treatment for non-gonococcal urethritis
 Is ineffective against B-Lactamase producing strains of gonorrhea
o Must be given daily to treat uncomplicated syphilis successfully
399-Penicillin:-
o The only treatment of syphilis
o Used for resistant penicillinase producers Bactria
 Used for treatment of syphilis as penicillin G
o All true
o All wrong
400-The most frequent gummatous involvement in abdominal syphilis is
o Gastric
o Intestinal
o Hepatic
o Splenic
o Pancreatic
401-Non-venereal treponematoses in comparison with venereal syphilis
o Are caused by distinct organisms
o Do not have a relapsing course
o Are best treated with aminoglycosides

187
o Tend to affect predominantly children
o Are generally less destructive
402-The TPI test will distinguish syphilis from
o Yaws
o Bejel
o Pinta
o All of the above
o None of the above
403-Yaws is caused by Treponema pallidum subsp
o Endemicum
o Pertenue
o Carateum
o Vincenti
o None of the above
404-Yaws what causative organism
o T. Palleduim
 Ssp pertenue
405-Pinta is unique among the treponematoses as it
o Has only cutaneous manifestations
o Is found on the Eastern hemisphere
o Affects mainly the elderly
o Is not curable
o All of the above
406-Pinta causative organism
o T. Carateum
407-Endemic syphilis (bejel) (Endemicum)
o Is prevalent in dry arid climates
o Was eradicated from Africa in 1963
o Affects males more than females
o Tends to affect children between 2 months to 2 years of age
o Can be transmitted through breast milk
408-Mixed chancre on the penis may be due to syphilis and:-
o Staphylococcal infection
o Streptococcal infection
o Haemophilus ducreyi
o Pseudomonas infection
409-The incubation period for chancroid is:-
o 1-2 days
o 1-5 days (2-4)
o 5-10 days
o 7-14 days
o 14-28 days
410-LGV may show
o Multiple painfull ulcers
o painful erosion
 painless erosion last for days
o painless erosion last for 1-2 week
411-Gonorrhea most common site for discharges
o Cervix
o Urethra
o Post vaginal fornix

188
412-The most common site for taking swab to diagnose female gonorrhea is:-
o Urethra
o Vagina
o Cervix
o Labia minora
413-The most suitable schedule for treatment of gonococcemia is:-
o Procaine penicillin G 4gm
o Ceftriaxone 1gm/day IV for 48h after improvement + ciprofloxacin 500mg orally twice daily for 7
days
o Doxycycline 100mg twice daily for 15 days
o Erythromycin 500mg 4 times daily foe 10 days
414-The most suitable schedule for genocaceiemia
o 4 g procaine
o Cefitroxeno 1gm/7days
o Tetracycline 2g/day -15 days
o Doxycyline 100mg twice /day – 15days
o Erythromycin 1g/day – 10days
415-Gonococcemia can be most definitely distinguished clinically from chronic meningococcemia on the
basis of
 Systemic toxicity and fever
o Hemorrhagic cutaneous lesions around joints
o Arthralgia
o Tenosynovitis
o None of the above
416-A culture of N. Gonorrhea can be most clearly distinguished from one of N. Meningitis on the basis
of
o Growth on Thayer-Martin medium
o Growth upon incubation in an elevated CO2 atmosphere
o Oxidase test
o Fermentation of sugar
o All of the above
417-Reverse transcriptase is characteristic to one of the following:-
o HIV infection
o HSV
o EHV
418-In HIV-infected patients, one of the following is NOT a clinical subtype of anal intraepithelial
neoplasm
o Bowenoid
o Verrucous
o Porokeratotic
o Leukoplakic
o Erythroplakic
419-In general, immunodeficiency should be suspected when a patient has:-
o Recurrent infections of increased duration and severity
o Recurrent infections with unusual organisms
o Incomplete clearing of infections of poor response to antibiotics
o All of the above
o None of the above
420-HIV patient develop lipotrophy of cheeks what the filling material to use
o Hyaluronic acid
o Human collagen

189
o Poly-L-lactic acid (Sculptra).

892-Match the following:


 Syphilis →penicillin g
 Lvg →doxycline
 Rectal gonorrhea →cefitrioxon
 Chancroid →ciprofloxacin
 Chlamydia urethritis →amoxycillin

904-Lesion basic cardiovascular syphilisis


 Aortic

925-1 month urethritis after extramarietal sex then vesicles and pustules on side of foot and sole:
o Palmoplantar pustulosis
 Reiter syndrome
o Syphilis

940-Best ttt for Juvenil Rieter


 The arthritis was treated with acetaminophen and naproxen

977-Oral syphilis is the form of


 Mucous patches
o Erythema
Vesicles

989-patient with painless ulcer on the glans what is the best early diagnostic tool
o VRDL
o TEPHA
 FTA ABS

1026-AIDS IP
3 m/10y

1028-Balanitis caused by all except:


o Not feature of bowen
o Nourethritis
 Foliculitis

1067-Patient back erythematous lesions on back and genital area with blasma cells in biobsy
 Plasma cell balanitis
o Seyphles

190
2019-Vulvar syndrome (vulvovaginitis) vulvo vaginal syndrome caused by candida
The vulvo vaginal- gingival syndrome is occurring in erosive
 Lichen planus

1141-Male patient with painful papulovesicular eruption of genitalia then resolved and after that
developed unilateral enlarged inguinal lymph nodes and sinuses occurred (LGV). Best treatment:
o Penicillin benzathin
o Oflaxacin
 Tetracycline
o Acyclovir

1176-Male patient with painless ulcer on the shaft of the penis and had sexual intercourse since (9 < ? <
90) day, what is the most confirmatory test:
 Dark ground microscopy

1200-Patient with recurrent oral and genital ulcers and recurrent episodes of conjunctivitis With papules
spreading over trunk and extremites Lab analysis done twice with WBCS 18000 TO 20000,over six
months on treatment…what is the best option?( behcet disease)
o NSAIDS
 Steroids

1220-Patient with papular pruritic eruption (Pruritic papular eruption of HIV)TTT is:
 PUVA
o Nifedipine
o Ivermectin
o Permethrin
1266-Oral and genital ulcer, bilateral conjunctivitis, CBC showed leukocytosis18000to20000 and
perphral eosinophilia on 2 occasions 6 monthes apart, biopsy showed +ve mutation gene TTT ( leukemia
AML) :
 Imatinib
o systemic steroids
o malphan
1365-Sores and color smear gram sno image gram-negative bacilli what is the appropriate treatment
(chancroid)
 Siprofloxacin 2
o Penicillin
 Azethromycin 1

191
1365-Figure of multiple painful genital ulcers and after few days developed enlarged inguinal mass
choose the right answer:
o LGV
o Granulomainguinale
 Chancroid
o Chancre

1366-Pt can do which treatment for wart (figure penis) by himself:


 Imiqimod
o Podiphilin
o Cryo
o Electrocautery

1367-Photo of scrotum brownish papules (angiokeratoma) what is the best treatment:


 Liquid nitrogen

1368-Recurrent penile eruption


 HS
o Chancre
o Chancroid
o Lgv

192
1634-In LGV:- bubos, inflammatory adenitis.

1645- Match the following:-

Chancroid 1-5 days Haemophilus ducreyi Amoxicilline + clavulanic acid


LGV 3-30 days Chlamudia (L1 , L2 , L3) Ciprofloxacin
Syphilis 9-90 days Treponema pallidum Penicillin G
Rectal Gonorrhea 2-5 days Neisseria gonorrhoeae Ceftrixone
Chlamydia urithritis 2-3 weeks Chlamedia trachomatis Doxacycline
Granuloma inguinal 8-80 days Donovani bodies Doxacycline or TMP-SMZ or
Ciprofloxacin

From the questions in page 78 :

35-Syphilis; early congenital; early manifestation appear during:- 2nd to 10th weeks (6th weeks).
36-Syphilis;transmission to the fetus occurs in:- benging of the 2nd trimester.
37-Syphilis; early congenital; early manifestation ; rhinitis-nasal snuffles
38-Syphilis; early congenital; late manifestation ; condylome lata
39-Syphilis; secondary; palms-soles (most common site of rash)
40-Syphilis; mucous patches (most common presentation)
41-Syphilis; Painless ulcer
42-Syphilis; FTA-abs test 1 to turn positive
43-Syphilis; mixed chancre; T. Pallidum +H ducrei
45-Gonorrhea; cervix (most common site in females)
46-LGV: pubo

From the questions in page 83 :

47) Most common cause of acute paronychia- staph


48) Most common cause of chronic paronychia-candida
49) Most common syphilis transmitted by sexual route- primary
50) Most common syphilis transmitted from infected mother- secondary
51) Most infectious lesion in syphilis- mucous patches
52) Most sensitive test in syphilis- Enzyme Immunoassay> TPPA > FTA-abs
53) Most specific test in syphilis- TPPA> TPHA
54) DOC for chancroid- azithro
55) DOC for LGV and donovanosis- doxy
56) DOC for syphilis in pregnancy- penicillin
57) DOC for urethral discharge and cervical discharge (syndromic management)- azithro+
cefixime

193
58) DOC for vaginal disch (syndromic management)- fluconazole + secnidazole/metro/tinidazole
59) DOC for bubo (syndromic management)- azithro+ doxy
60) DOC for genital ulcer (syndromic management)- if vesicle - acyclovir, if not azithro+
benzathine penicillin
61) DOC for neurosyphilis- crystalline aqueous penicillin
62) DOC for penicillin allergy in syphilis- doxy
63) DOC for penicillin allergy in syphilis in pregnancy- desensitization
64) DOC for penicillin allergy in neurosyphilis- desensitization
65) DOC for Impetigo herpetiformis- steroids
66) Investigation of choice in primary syphilis- dark ground illumination
From the questions in page 84 :

101) Investigation of choice for neurosyphilis- CSF-VDRL

110) Commonest extra genital site for primary chancre- lips

111) Commonest cause of recurrent blisters on genitals (healing with hyperpigmentation- FDE) (
if not then herpes genitalis)

116) Commonest site for primary syphilis chancre- Coronal sulcus


117) First test to become positive in primary syphilis- FTA-Abs

194
Surgery of the Skin & Local Aneasthesia
Dr.Ashraf Abo
✂ ✂ ✂Suture removal
---------------------------
✂Head 5-7 days
✂Face 3-5 days
✂Eyelid & eyebrow 3-5 days
✂Trunk 5-7 days
✂Extremities 7-10 days
✂Surface of joint 10-14 days
✂Hand 7 days
✂Sole of foot 7-10 days

36-Least pressure ulcer point:


o Scapula
o Heel
o Ischeal tuberosities
o Greater trochanter
o Secrum
62-The tip of the nose is innervated by:-
o Ophthalmic nerve
o Occulomotor nerve
o Trigeminal nerve
63-Nerve supply of the alae nasi by
o Branch of supratrochlear nerve
o Branch of infratrochlear nerve
o Branch of the infraorbital nerve(from the maxillary division of the trigeminal nerve)
o External nasal nerve

88-Infection in cutaneous surgery usually occurs after


o 24 hrs
o 24 – 48 hrs
o 48 – 72 hrs
 4-8 days
89-The appearance of infection after surgery
o After 12-24 h
o After 24-48 h
o After 48-72 h
o 4-8 days
86-For prophylactic from infection , before procedure , the antibiotic should be given
o 6 hours
o 14 hours
o 24 hours

195
87-High risk patient for cardiac diseases scheduled for operation (cosmotic) should recive antibiotic
(prphylactic).To avoid skin infection:-
o 1 hour before surgery
o At time of surgery
o 6 hours after surgery
o 1 day after surgery

90-To prevent cutaneous complications after surgery , the antibiotics is given:


 Before surgery
o Day of surgery
o 6 hrs after surgery
o 24 hrs after surgery
91-Sutures near the eye can be removed after
o 5-7 d
o 1w
o 7-9d
o 10-14d
92-Single or multiple bands in the
o Vicryl
o Prolein
o Silk
o Nylon
93-Monofilament or single sture
o Vicryl
o Dexon
o Chromic
o Prolene
94-Type of suture most suitable for cutaneous surgery
o Single nylon suture
o Double nylon suture
o Braided silk
o Unbraided silk
95-The most prominent side effects of cryosurgery is:-
 Hypopigmentation
o Hyperpigmentation
o Scarring
o Pain
o Never damage
670-Depressed area with postinflamatory hyperpigmentation after repeated injection what's better
o Reassurance
o Topical steroid
o Dapsone
o Topical retinoid

1061-70 kg patient will do excisional surgery, maximum amount of 1 % lidocaine should not exceed
o 10 ml
 30 ml
o 100 ml
o 250 ml

196
Tongue and Mucous Membranes
Disorders affecting the tongue :
· Furred Tongue Poor dental hygiene/Excessive smoking/Topical or systemic antibiotic
· Atrophy of the Tongue Anemia: iron deficiency/Ariboflavinosis/Pellagra
· Scrotal Tongue :Hypothyroidosis/Edema/Symphiliticgumma
· Bifid Tongue : Oral facial digital syndrome/Cleft palate/Widely spaced teeth
· Geographical Tongue -/Psoriasis/Atopic dermatitis/Seb. Dermatitis
· Median rhomboid glossitis - develop mental defect/C h. Oral candidiasis
· Glossodynia : burning tongue/psychological disorder
166-Black hairy tongue what is the treatment:-
o Antibacterial
o Antiviral
o Antifungal
o Brushing the tongue
558-Percentage of oral lesion in Behcet's disease is:-
o 10%
o 30%
o 50%
o 90%
559-The most classic ocular involvement in Behcet's disease is
o Posterior uveitis
o Conjunctivitis
o Retinal detachment
o Optic atrophy
o Vitreous hemorrhage
549-Each of the following distinguishes Churg-Strauss syndrome from WG except
o Myocardial involvement
o Eosinophilia
o Long history of asthma
o High incidence of anti-mpo antibodies
o Granulomatous inflammation
560-In Behcet's disease all the following can occur EXCEPT
o Uvietis
o Optic neuritis
o Keratoconjuctivitis
 Urithritis
561-Patient comes to ER having fever & history of recurrent ulcers on tongue , penis and sometimes
recurrent bilateral conjunctivitis what is the best treatment: behcet disease
o Steroid
o Dapsone
893-Geographic tongue
 Psoriasis
944-Gingival hypertrophy occurs most commonly with
o Burkett's lymphoma (rare not common)
994-Lower lip presistant edema +( Melkersson-Rosenthal syndrome) ln what treatment is not needed
o Thalidomide
 Isotretinion
o Ketotefen

197
o Cs
 Clofazimine
1032-Commenst cause of angular cheilitis
 Mechanical
1142-Case of old man hospitalized prolong developed over month with red glossy tong
o Zinc
o Scurvy
 Vit B2
1363-5 y eodema of lips and gingival biopsy give epitheliod granuloma doesn'respond to antituberculous
(Picture of mouth &teeth,lip hypertrophy +facial palsy),biopsy showed granuloma
o TB
o Leprosy
 Melkersson rosenthal syndrome

1364-melkersson rosenthal syndrome treatment


 Corticosteroid, NSAI and antibiotics.

1373-Photo of tongue ( black tongue): due to :


 Antibiotic
o Antihypertensive
o Antiviral
o Hypoglycemic

From the questions in page 80 :

154-Inflammatory bowel disease; skin affection: PG, SCPD,EED


155-Crohn’s disease :oral lesions (the more common ones)
157-Tongue, Black Hairy: bacteria, antibiotic
157-Tongue; Smooth, sprue, pellagra (beefy red)
175-Tongue, Geographic: psoriasis(pustular),Seborrhea, Reiter’s
175-Tongue; strawberry: measles
165-Tongue; scrotal: congenital/acquired (Melkerson-Rosenthal)

198
Vascular

530-Leucocytoclastic vasculitis is due to:-


o Delayed hypersensitivity reaction
o Circulating immune complexes
o Immediate hypersensitivity reaction
531-What not associated with vasculitis
o Erythema elevatum duntenum
o Erythema induratum
o Gainuloma fascula
o Reticular histiocytosis
532-All are vasculitis disorder EXCEPT
o EED
o Granuloma faciale
o Multicent reticulo histocyte
533-Leukocytocloclastic vasculitis can be caused by the following drugs EXCEPT
o Insulin
o Tamoxifen
o Sulphonamides
o Oral contraceptives
o Cyclosporine
534-All of the following characteristics urticarial vasculitis lesions except
o They favor the trunk over acral surfaces
o Posinflammatory hyperpigmentation
o Itching
o Duration longer than 24 hours
o Purpura
535-What in urticarial vasculitis
o B cells
o T cells
o Complement deficiency
o Ige
536-Purpura is seldom thrombocytopenic in origin if the platelet count is below
o 3000
o 7000
o 10,000
o 20,000
o 50,000
537-Henoch-schönlein purpura all EXEPT
 Iga deposits are specific but not sensitive (not effect diagnosis or prognosis)
o Early steroid treatment decreases clinical manifestations
o Relapse occurs in one third of patients
o Third cases of patients will remit
o CNS is affected
538-What is TRUE concerning renal involvement in Henoch-Schonlein purpura ?
o It is second after skin in frequency of appearance
o It tends to occur early in the course
o It never progresses to renal failure if the only presenting sign is microscopic hematuria

199
 It is unlikely to develop later if renal manifestation are obsent in the first episode of HSP
o It is always associated with linear deposits of IgA in kidney biopsy specimens
539-Which of the following statements is true regarding Henoch-Schonlein purpura and renal disease?
o End-stage renal disease is more common occurs in 30% of patients
 Renal disease is more common in patients with purpura above the waist
o Azathioprine is not an effective treatment of nephritis
o ESRD is more likely to occur in children than in adults
o ESRD is more common in the setting of an iga-ANCA
540-Child with abdominal pain , purpuric lesions and joint pain , what the diagnosis
o Rheumatic fever
o Henoch-Schenolen Purpura
o Kawasaki syndrome
541-Hench schohlein purpura what is TRUE
o Recurrent 1/3-2/3 patient
o Renal affection
o IgA perivascular deposition is specific
542-Child with palpable purpura on leg with out abdominal complication , urine analysis normal , what
treatment (henoch–schönlein purpura history)
o NSAID (first line is symptomatic therapy)
o Topical steroid
o Dapson
o Systemic steroid

543-Which treatment seems most effective in treating the rash of Hench-Schlloein purpura?
o Corticosteroids
o Nsaids
o Factor XIII
o Dapsone
o Azathioprine

545-Which of the following statements is NOT TRUE of Polyarteritis nodosa


o Rarely involves the skin except as a benign form limited to skin and subcutaneous tissue
 Involves mainly arterioles
o Exhibits necrosis, inflammation, granulation and fibrosis of all vascular wall layers
o Is a focal disease of vessels
o None of the above
546-Which organ is classically spared by SPAN?
 Lungs
o Kidney
o Genitourinary system
o Gastrointestinal tract
o Central nervous system
547-43 year old patient presented with multiple, bright red, friable papules of the gingival as well as
palpable purpura of the lower extremities. The diagnosis most probably is
o Cryoglobulinemia secondary to HCV infection
o Polyarteritis nodosa
o Wegener granulomatosis
o Churg-Strauss syndrome
o Microscopic polyarteritis
548-Typical features of Waardenburg include all EXCEPT

200
o Dystopia canthorum
o White forelock
o Confluent eyebrows
o Low-set ears
o Congenital deafness

550-The pathogenic process in Degos' disease is


o Ischemic infarction
o Necrobiosis
o Hemorrhagic necrosis
o Malignant infarction
o None of the above

565-What is the common association these 3 disease EED, PG, SCPD


o Inflammatory bowel disease
o Sweet’s syndrome
o HCV
o DM

576-Cyanotic areas that appear on sites of dense subcutaneous fat as the thigh & lower leg of adult female
that during winter is due to –
o Acrocyanosis
o Erythrocyanosis
o Livedo reticularis
o Chilblains
o Perniosis
577-Physiological livedo reticularis means:-
o Acrocyanosis
o Erythrocyanosis
o Perniosis
o Cutis marmorata
578-The most common cause of livedo reticularis is
o Connective tissue disease
o Polyarteritis nodosa
o Lymphosarcoma
 Physiologic reaction to cold
o Arteriosclerosis
679-Periungual telangiectasia are diagnostic of
o SLE
o Dermatomyositis
o Scleroderma
o Glomous tumor
o None of the above

833-Which of the following is the best treatment option for frostbite?


o Removal from the environment , immersion in water at 40-42 C and pain control +
o Removal from the environment , immersion in water at 30-35 C
o Immersion in water at 45-50 and pain control
o Removal from the environment and pain control

201
834-Frostbite treatment implies
o Removal from site, immersion in water 45 _ 50 , pain control
o Removal from site, immersion in water 30 _ 35 , pain control
o Removal from site, immersion in water 42 _ 45 , pain control
o Removal from site, pain control

835-Ulcer on leg stump due to:-


o Diabetic
o Vascular pressure
o Friction of arm prosthesis
 All of the above
836-A child with depressed skin lesion on buttocks due to past injections of unknown drug best
management
o Topical steroids
o Topical calcineurin inhibitors
o No treatment
837-Varicose veins maybe complicated by:-
o Hyperpigmentation
o Ulceration
o Painful
o Thrombophlebitis
 All of above
838- 47 years old patient presented with an ulcer on the big toe, foot & lower leg are cold & arterial pulse
is not felt, the diagnosis is:-
o Diabetic ulcer
o Stasis ulcer
o Arterial ulcer
o Venous ulcer.
o Traumatic ulcer
839-Treatment of choice in Pyoderma faciale is:-
o Tetracycline
o Prednisolone
o Isotretenoin
o Combination of prednisolone & Isotretenoin
o Doxycycline
840- Oral lesion in chronic GVHD:-
o 100%
 80%
o 50%
o 10%
o 0%
841-Commonest cause of graft versus host disease
o Bone marrow transplant
o Blood transfusion
o Kidney transplant

859-6 hours old baby suddenly develop palpable purpura surrounded by erythema at elbow,knee and
buttocks and his urine analysis is normal but the lesions are tender what is his diagnosis:
o Acute heamorrhagic pupura of childhood
o Anaphlactoid p

202
o Idiopathic thrombocytopenic purpura
o It develop without renal changes and tends to affect extremities and it is painful

876-Child develop erythematous rash after sun exposure, CBC reveal increased rbcs and porphrins what
is your ttt:
o Phlebotomy
o Strict sun protection
o Steroid

884-Urticarial vasculitis can develop with low:


o Ig E
 Hypocomplementia

886-What not assocaited vasculitis


o Erythema elevatum duntenum
o Erythema induratum
o Granuloma faciale
 Reticular histiocytosis

888-Not a vasculitis:
o Granuloma faciale
o Erythema multiform
 Reticular histiocytosis

894-Graft host
 Immunocompetent donor cells against tissue of immucomopetent host

919-What is true about protein C?


 Has antithrombotic effect
o Inhibit protein S
920-Heterogenous protein c deficiency result in:
o Thrombocytopenia
o Low ability to coagulate
 coumarin skin lesion
o act as 10 % as normal
 Venous thrombosis
 AD
 Inactivate Va,VIIa
921-Protein c deficiency
 Venous thrombosis
924-20 years old female treated for depression, suffering from ecchymosis and purpura of the lower leg
which investigation(Gardener diamond syndrom)?
 Intradermal injection of autolgous RBC,s

203
o Platelet count
o Skin biopsy
o Bleeding and clotting time

1008-Hemangioma best described as


o aabnormal vascular development and proliferation
 Benigin non proliferative condition
o Enotheilial proliferation
o Affection of the veins, capillaries

1087-Genetic angioedema best TTT


o Anti-histamine
o Steroids
 Danazol
o Ephedrine

1095-New born baby see the doctor eczema lesions found on the face arm and groin. Pustule on areas of
the body with a cold abcess and bronchitis have ige caliber of more than 3000 units what is the diagnosis
 Hyperimmunoglobulinemia E syndrome
o AD

1120-Describing a case of acute GVHD ( a patient with leukemia and do bone marrow transplantation he
develop skin rash) .which is not indicated in treatment
 Acitretin
o Cyclosporine
o Infleximab
o Steroid

1188-Hypocomplementemic urticarial vasculitis is present with


 SLE
o Wegner granulomatosis
o Churg strauss
o Systemic sclerosis
o Esinophilic fasciitis

1189-A case of female patient when exposed to cold she develops cyanosis of finger and when rewarm
her fingers return red , is associated with which disease
 Raynaud's disease
o Rheumatoid arthritis
1199-Child complain of rash for the first time in his life over lower lig with no systemic
complain…biobsy shows vasculitis (HSP) what is the treatment?
 NSAID
o Steroids
1210-Girl with erythematous and cold toe finger every winter since 7 years is due to
 Perniosis

204
o Acrocyanosis
o Raynaud's phenomenon

1219-Not a type of vasculitis:


Multicentric reticulohistiocytosis

1231-Devil’s punch is
 Purpura simplex on the thigh of young women

1243-What factor increases the risk of haemangioma?


 Prematurity

1290- Regarding targetoid hemosiderotic hemangioma, what is true:


o Multiple vascular lesions on legs of eldery people
o Closely related to HIV
 HP shows dilated vascular channels surrounded by hemosidren &extravasated RBC's
o May be mistaken by lymphatoid papulosis

1259-Haemangioma affecting:
 Veins and capillaries

1301-Klippel trenaunay syndrome (congenital - ubnormal,blood vessels and lymph vessels)


 Doppler duplex

1303-13 year old female child presented with recurrent sinusitis fever, arthralgia, respiratory
distress, Haematuria and hypertension. Renal Biopsy showed necrotizing granuloma. The anti proteinase-
3 ANCA was positive. The most likely diagnosis is(friable red papules on gingiva + palpable purpura on
extremities)
o Polyarteritis Nodosa
 Wegener’s Granulomatosis
o Microscopic Polyangitis
o Churg-Strauss

1304-Child with purpura (Acute heamorrhagic pupura of childhood),what is the least cusative organism:
 HSV
o CMV

1350-Purple papules on the face of baby remission after:


 Blue muffin baby

205
1396-Hereditary Hemorrhagic Telangiectasia: (Osler–Weber–Rendu)
• AD, HHT1 and HHT2
• Multiple mucocutaneous and GI telangiectasias: epistaxis, telangiectasis(skin/mucosa),

1421-Telangiectasia are the hallmark of perioral dermatitis:-


o True
o False

1486-Purpura is the commonest unwanted drug reaction in the skin


o True
o False
1535-Pateint take a drug at what time leukocytoclastic vasculitis appear:- 1-3 week
1557- In hereditary angioedema the following is right:- AD, serum C4 is reduced, hereditary deficiency
of C1 estrase
1574-Porphyria cutanea tarda has a relation with:- hepatitis C.
1589- Which of the following is not characteristic of puepura hyperglobulinemia (waldenstrom
syndrome):- occurs predominantly in middle age men.
1607-Phototherapy of neonatal hyperbilirubinaemia is accompanied with light having a wave length of:-
400-500 nm.
1612-Best treatment of hypocomplemenic urtecarial vasculitis:- sedative antihistaminic
1617-Thermoregulatory vasomotor activity occurs in:- arterovenous anastomoses. Metarterioles,
arterioles.
1633 -Ulcer of big toe with cold limb and intermittent clouducation is:- Arterial ulcer.
1636- Hereditary angioedema all false except:- AD
From the questions in page 81 :

166-Heriditary-Haemorrhagic telangiectasis: autosomal dominant AD

171-Spider telangclectaisa; chronic liver, children

206
‫‪Thank you‬‬
‫اللهم اجعل هذا الكتاب شفيعا لنا يوم القيامه‬
‫نروجو الدعاء لكن من قام بجمع األسئلة و األجوبة‬

‫‪207‬‬

You might also like